Download as docx, pdf, or txt
Download as docx, pdf, or txt
You are on page 1of 129

TESTATE ESTATE OF EDWARD E. CHRISTENSEN v.

HELEN CHRISTENSEN
GARCIA, GR No. L-16749, 1963-01-31

Facts:

This is an appeal from a decision of the Court of First Instance of Davao,


Hon. Vicente N. Cusi, Jr., presiding, in Special Proceeding No. 622 of said
court, dated September 14, 1949, approving among other things the final
accounts of the executor, directing... the executor to reimburse Maria
Lucy Christensen the amount of P3,600 paid by her to Helen Christensen
Garcia as her legacy, and declaring Maria Lucy Christensen entitled to
the residue of the property to be enjoyed during her lifetime, and in case
of death without issue,... one-half of said residue to be payable to Mrs.
Carrie Louise C. Borton, etc., in accordance with the provisions of the will
of the testator Edward E. Christensen.

"'12. I hereby give, devise and bequeath, unto my well-beloved daughter,


the said Maria Lucy Christensen Daney (Mrs. Bernard Daney), now
residing as aforesaid at No. 665 Rodger Young Village, Los Angeles,
California, U.S.A., all the Income from the rest, remainder, and... residue
of my property and estate, real, personal and/ or mixed, of whatsoever
kind or character, and wheresoever situated, of which I may he
possessed at my death and which may have come to me from any source
whatsoever, during her lifetime: * * *"... the executor in his final account
and project partition ratified the payment of only P3,600 to Helen
Christensen Garcia and proposed that the residue of the estate be
transferred to his daughter, Maria Lucy

Christensen.

The legal grounds of opposition are (a) that the distribution should be
governed by the laws of the Philippines , and (b) that said order of
distribution is contrary thereto insofar as it denies to Helen Christensen,
one of two acknowledged... natural children, one-half of the estate in full
ownership.

In amplification of the above grounds it was alleged that the law that
should govern the estate of the deceased Christensen should not be the
internal law of California alone, but the entire law thereof because
several... foreign elements are involved, that the forum is the Philippines
and even if the case were decided in California, Section 946 of the
California Civil Code, which requires that the domicile of the decedent
apply, should be applicable.
The court below ruled that as Edward E. Christensen was a citizen of the
United States and of the State of California at the time of his death, the
successional rights and intrinsic validity of the provisions in his will are
to be governed by the law of California, in... accordance with which a
testator has the right to dispose of his property in the way he desires,
because the right of absolute dominion over his property is sacred and
inviolable (In re McDaniel's Estate, 77 Cal. Appl. 2d 877, 176 P. 2d 952,
and In re Kaufman, 117 Cal. 286, 49

Pac. 192, cited in page 179, Record on Appeal). Oppositor Maria Helen
Christensen, through counsel, filed various motions for reconsideration,
but these were denied. Hence this appeal.

Issues:

The lower court erred in failing to recognize that under international law,
particularly under the Renvoi Doctrine, the intrinsic validity of the
testamentary disposition or the distribution of the estate o£ the deceased
Edward E. Christensen should be governed by the laws of... the
Philippines.

Ruling:

There is no question that Edward E. Christensen was a citizen of the


United States and of the State of California at... the time of his death. But
there is also no question that at the time of his death he was domiciled in
the Philippines

In arriving at the conclusion that the domicile of the deceased is the


Philippines, we are persuaded by the fact that he was born in Now York,
migrated to California and resided there for nine years, and since he came
to the Philippines in 1913 he returned to California very... rarely and only
for short visits (perhaps to relatives), and considering that he appears
never to have owned or acquired a home or properties in that state, which
would indicate that he would ultimately abandon the Philippines and
make home in the State of California.

As to his citizenship, however, we find that the citizenship that he


acquired in California when he resided in Sacramento, California from
1904 to 1913, was never lost by his stay in the Philippines, for the latter
was a territory of the United States (not a state) until 1946... and the
deceased appears to have considered himself as a citizen of California by
the fact that when he executed his will in 1951 he declared that he was a
citizen of that Estate; so that he appears never to have intended to
abandon his California citizenship by acquiring... another.
'Residence simply requires bodily presence as an inhabitant in a given
places while domicile requires bodily presence in that place and also an
intention to make it one's domicile.'

The law that governs the Validity of his testamentary dispositions is


defined in Article 16 of the Civil Code of the Philippines , which is as
follows:

"Art. 16. Real property as well as personal property is subject to the law
of the country where it is situated.

"However, intestate, and testamentary successions, both with respect to


the order of succession and to the amount of successional rights and to
the intrinsic validity of testamentary provisions, shall be regulated by the
national law of the person whose succession is under... consideration,
whatever may he the nature of the property and regardless of the country
wherein said property may be found."

The application of this article in the case at bar requires the


determination of the meaning of the term "national law" as used therein.

The "national law" indicated in Article 16 of the

Civil Code above quoted can not, therefore, possibly mean or apply to any
general American law. So it can refer to no other than the private law of
the state of which the decedent is a citizen, in the case at bar, the private
law of the State of California .

under the California Probate Code, a testator may dispose of his property
by will in the... form and manner he desires, citing the case of Estate of
Mc Daniel, 77 Cal. Appl. 2d 877, 176 P. 2d 952. But appellant invokes the
provisions of Article 946 of the Civil Code of California, which is as
follows:

"If there is no law to the contrary, in the place where personal property is
situated, it is deemed to follow the person of its owner, and is governed
by the law of his domicile."

Appellees argue that what Article 16 of the Civil Code of the Philippines
pointed out as the national law is the internal law of California . But as
above explained, the laws of California have prescribed two sets of laws
for its citizens, one for residents therein... and. another for those
domiciled in other jurisdictions. Reason demands that We should enforce
the California , internal law prescribed for its citizens residing therein,
and enforce the conflict of law rules law for the citizens domiciled
abroad. If we must enforce the law of
California as in comity we are bound to do, as so declared in Article 16 of
our Civil Code, then we must enforce the law of California in accordance
with the express mandate thereof and as above explained, i.e., apply the
internal law for residents therein, and its conflict of... laws rule for those
domiciled abroad.

As explained in the various authorities cited above the national law


mentioned in Article 16 of our Civil Code is the law on conflict... of laws in
the California Civil Code, i.e., Article 946, which authorizes the reference
or return of the question to the law of the testator's domicile.

The Philippine court must apply its own law as directed in the conflict of
law rule of the state of the decedent, if the question has... to be decided,
especially as the application of the internal law of California provides no
legitime for children while the Philippine law, Arts. 887 (4) and 894, Civil
Code of the Philippines , makes natural children legally acknowledged
forced heirs of the parent recognizing... them.

We therefore find that as the domicile of the deceased Christensen, a


citizen of California , is the Philippines , the validity of the provisions of
his will depriving his acknowledged natural child, the appellant, should be
governed by the Philippine law, the domicile,... pursuant to Art. 946 of the
Civil Code of California, not by the internal law of California .

the decision appealed from is hereby reversed

G.R. No. L-23678 – 20 SCRA 358 – Civil Law – Application of Laws – Nationality
Principle – Renvoi Doctrine – Conflict of Laws

Succession – Nationality of the Decedent – Legitime

Amos Bellis was a citizen of the State of Texas, and of the United States.
By his first wife whom he divorced he had five legitimate children (Edward
Bellis et al), by his second wife, who survived him, he had three legitimate
children. He, however, also had three illegitimate children in the
Philippines (Maria Cristina Bellis et al). Before he died, he executed a will
here in the Philippines which disposed of his Philippine properties (Bellis
actually executed another will disposing his US properties. In said will, he
still gave nothing to his illegitimate children). His illegitimate children were
not given anything. Amos Bellis died in 1958 and at the time of his death,
he was a resident and a domicile of Texas, USA. The executor of the will
then caused the admission of the will to probate. The illegitimate children
opposed the will on the ground that they have been deprived of their
legitimes to which they should be entitled, if Philippine law were to be
applied. Maria Cristina et al invoked Par. 3, Art. 17 of the New Civil Code
which provides that Philippines Laws on property relations should not be
rendered ineffective by foreign laws and/or agreements.

ISSUE: Whether or not the national law of the deceased should determine
the successional rights of the illegitimate children.

HELD: Yes. The Supreme Court held that the said children, Maria Kristina
et al, are not entitled to any legitime. Under the Texas Law, being the
national law of the deceased and which is the law applicable, there are no
legitimes. The SC did not give credence to the argument of Maria Cristina
et al. Par. 3, Art. 17 of the New Civil Code is not an exception to the
Nationality Principle (par. 2, Article 16 of the Civil Code). Rather, it is the
other way around. This is highlighted by the fact that Art. 1039 of the
same law clearly states that capacity to succeed is governed by the law of
the nation of the decedent. Thus, the intent of the lawmakers is clear.

The SC also took time to discuss the renvoi Doctrine though not invoked
by Maria Cristina et al: It is not disputed that the decedent was both a
national of Texas and a domicile thereof at the time of his death. So that
even assuming Texas has a conflict of law rule providing that the
domiciliary system (law of the domicile) should govern, the same would
not result in a reference back (renvoi) to Philippine law, but would still
refer to Texas law. Nonetheless, if Texas has a conflicts rule adopting the
situs theory (lex rei sitae) calling for the application of the law of the place
where the properties are situated, renvoi would arise, since the properties
here involved are found in the Philippines. In the absence, however, of
proof as to the conflict of law rule of Texas, it should not be presumed
different from ours.
Succession defined (book)

NACAR vs. NISTAL G.R. No. L-33006 December 8, 1982

FACTS: Sometime in the year 1968 petitioner’s stepfather, Isabelo Nacar, incurred an indebtedness with
the respondent in the amount P2,791.00. Despite repeated demands by the respondent, Isabelo was not
able to pay. Thus, prompting the respondent to file a civil action for the collection of money against
Isabelo. In the year 1970, Isabelo died. nistal then filed a complaint in Civil Case and entitled it "Claim
Against the Estate Nicanor Nacar the Late Isabelo Nacar With Preliminary Attachment". Petitioner filed a
motion to dismiss, to dissolve writ of preliminary attachment, and to order the return of the carabaos. In
his motion to dismiss, the petitioner raised the issue of lack of jurisdiction and absence of a cause of
action. Mr. Nacar averred that the indebtedness mentioned in the complaint was alleged to have been
incurred by the late Isabelo Nacar and not by Nicanor Nacar. There was, therefore, no cause of action
against him. Private respondent Japitana filed an opposition to this motion while intervenor Antonio
Doloricon filed a complaint in intervention asserting that he was the owner of the attached carabaos and
that the certificates of ownership of large cattle were in his name. The respondent Judge denied the
motion to dismiss prompting Mr. Nacar to come to the Supreme Court.

ISSUE: Whether or not a complaint against an heir of a decedent, who incurred indebtedness, is the
correct action for the collection of money?

HELD: The Court held in the negative. Under the circumstances of this case, respondent Japitana has no
cause of action against petitioner Nacar. The Court said that, a cause of action is an act or omission of
one party in violation of the legal right of the other. Its essential elements are, namely: (1) the existence
of a legal right in the plaintiff, (2) a correlative legal duty in the defendant, and (3) an act or omission of
the defendant in violation of plaintiff's right with consequential injury or damage to the plaintiff for
which he may maintain an action for the recovery of damages or other appropriate relief. Indeed,
although respondent Japitana may have a legal right to recover an indebtedness due him, petitioner
Nicanor Nacar has no correlative legal duty to pay the debt for the simple reason that there is nothing in
the complaint to show that he incurred the debt or had anything to do with the creation of the liability.
As far as the debt is concerned, there is no allegation or showing that the petitioner had acted in
violation of Mr. Japitana's rights with consequential injury or damage to the latter as would create a
cause of action against the former.
34 – Gloria and Yduan v. Builders Savings & Loan Assoc., Inc. G.R. No. 202324, 4 June 2018 THESIS
STATEMENT  Petitioners filed before the RTC a Complaint against Respondent for declaration of null
and void real estate mortgage, promissory note, and cancellation of notation in the TCT.

FACTS  Petitioners are registered owners of a parcel of land in Kamuning, QC covered by TCT No.
35814.  Subject land was used as a mortgaged for a loan and later it was foreclosed and sold at an
auction to Respondents. CONCHITA GLORIA & MA. LOURDES GLORIA-PAYDUAN  Claims that Benildo
Biag deceived them into surrendering TCT No. 35814 and was fraudulently made to sign loan and
mortgage documents.  Respondents conspired with Benildo Biag in the execution of the forged loan &
mortgaged documents. BUILDERS SAVINGS & LOAN ASSOC., INC.  Lourdes had neither capacity to sue
nor authority and interest to file the case a quo.  Conchita was a retired public-school teacher who
could not be cajoled to execute a real estate mortgage on her property against her will.

RULING OF THE LOWER COURT  RTC ruled in favor of the Petitioners.  It declared the real estate
mortgage as null and void.

RULING OF THE APPELLATE COURT  CA reversed and set aside the decision of the RTC.  It held that
Conchita voluntarily executed the real estate mortgage and Lourdes’ testimony was merely hearsay.

RULING OF THE SUPREME COURT As regards the supposed defective verification occasioned by
Conchita's failure to sign the amended complaint with its concomitant verification and certification
against forum shopping, the Court has repeatedly held that in a case involving co-owners of property
where said property is the subject matter of the suit, the failure of the other co-owners to sign the
verification and certification against forum shopping is not fatal, as the signing by only one or some of
them constitutes substantial compliance with the rule. The court also found no merit in respondents'
argument that the present petition should be dismissed for failure of the other co-heirs/co-petitioners
to sign the verification and certification against forum-shopping as required by Sections 4 and 5, Rule 7
of the 1997 Rules of Civil Procedure. xxxxxx "As such co-owners, each of the heirs may properly bring an
action for ejectment, forcible entry and detainer, or any kind of action for the recovery of possession of
the subject properties. Thus, a co-owner may bring such an action, even without joining all the other co-
owners as co-plaintiffs, because the suit is deemed to be instituted for the benefit of all." Finally, the
Court finds the trial court to be correct in granting petitioners' motion for reconsideration and declaring
the mortgage and promissory note as null and void. The evidence indicates that the documents were
indeed simulated.

Spouses Salitico v. Heirs of Resurreccion Martinez Felix


G.R. No. 240199
April 10, 2019
FACTS:

By virtue of a document called Huling Habilin ni Amanda H. Burgos, a parcel of land located in Bambang,
Bulacan was inherited by Resurreccion, Amanda’s niece, as devisee. Resurreccion then executed a document
entitled Bilihang Tuluyan ng Lupa which transferred ownership over the parcel of land in favor of the Spouses
Salitico, who then took physical possession of the subject property. The will, or the Huling Habilin, was
approved for probate. The Heirs of Resurreccion then sent a demand letter to the Spouses Salitico to vacate
the subject property and surrender possession to the former. For their part, the Spouses Salitico executed an
Affidavit of Adverse Claim which was denied registration by the Register of Deeds. They then sought the
delivery and return of the owner’s duplicate copy and execution of the Deed of Absolute Sale. They also prayed
that the original title be cancelled and a new one be issued in their names, and demanded payment of
attorney’s fees, moral and exemplary damages, and reimbursement for litigation expenses.

The RTC denied the Spouses’ Motion for Summary Judgment but partially granted their Motion for
Reconsideration. The RTC then issued a partial summary judgment in their favor ordering the Register of
Deeds to register their Affidavit of Adverse Claim. The RTC found that there was a valid sale but it held that the
action filed by the Spouses is premature on the ground that their cause of action has not yet accrued as
Amanda’s estate had not yet been fully settled by the Probate Court.

The CA dismissed the appeal of the Spouses due to the pendency of the probate proceedings and cited Rule
75, Section 1 of the Rules of Court, which states that no will shall pass either real or personal estate unless it is
proved and allowed in the proper court. The CA also cited Rule 90, Section 1, which states that no distribution
shall be allowed until the payment of debts, funeral charges, and expenses of administration, allowance to the
widow, and inheritance tax have been made, unless the distributees or any of them file a bond in a sum fixed
by the court conditioned on the payment of the said obligations.

ISSUE:

W/N the contention of the Spouses Salitico is meritorious

RULING:

The contention is partially meritorious. Art. 777 of the Civil Code provides that the rights of the inheritance are
transmitted from the moment of the death of the decedent. The said provision operates at the very moment of
the decedent’s death. The heir is then deemed to have acquired ownership of his share in the inheritance at
that moment and not at the time of the declaration of the heirs, partition, or distribution.

Upon Amanda’s death, Resurreccion became the absolute owner, subject to the resolutory condition that upon
the settlement of Amanda’s estate, the devise is not declared inofficious or excessive. Resurreccion now has
the obligation to deliver, pursuant to the valid sale to the Spouses Salitico. However, in consonance with PD
1529 on the transfer without an order of final distribution, the court held that before a transferee may compel
issuance of a new certificate of title, a final order of distribution of estate or an order in anticipation of such,
issued by a testate or intestate court, is necessary. Here, there is a valid sale, but the final settlement of
Amanda’s Estate is pending and an order in anticipation of such is valid; hence, the Register of Deeds cannot
yet be compelled to cancel the Original Certificate of Title and issue a new one in favor of the Spouses Salitico.

Treyes vs. Larlar (G.R. No. 232579)


2022-08-23administrator
DR. NIXON L. TREYES, PETITIONER, V. ANTONIO L. LARLAR, REV. FR. EMILIO L.
LARLAR, HEDDY L. LARLAR, ET AL., RESPONDENTS.

G.R. No. 232579, September 08, 2020

CAGUIOA, J.:

FACTS:

Rosie Larlar Treyes died intestate and without a child. Rosie’s siblings sent a letter to Dr. Nixon
Treyes, the husband of Rosie, the letter states that they are inviting him for a conference for the
settlement of estate of Rosie but Treyes ignored the letter and executed two affidavits of self-
adjudication which he registered with the Registry of Deeds of Marikina, Rizal, and San Carlos,
Negros Occidental, transferring unto himself 14 properties, as sole heir of his decedent-spouse.
After sending a second letter, it was found out by the siblings that the properties of Rosie were
already transferred to Treyes.

The siblings Larlar filed an action for annulment of the Affidavits, cancellation of TCTs,
reconveyance of ownership
and possession, partition, and damages, before the RTC of Negros Occidental.

A first service of summons was served on Treyes, which he filed a motion to dismiss on the
ground of lack of jurisdiction over the person of petitioner. So another service of summons was
served to Treyes which then he filed another Motion to Dismiss arguing that the private
respondents’ Complaint should be dismissed on the following grounds: (1) improper venue; (2)
prescription; and (3) lack of jurisdiction over the subject matter.

Treyes filed a Motion to Dismiss on the ground, among others, of lack of jurisdiction over the
subject matter and lack of real parties in interest, arguing that the petitioners have not established
their right to succession and so lacking the personality to file the complaint; prescription on the
ground that the action for recovery of properties of the heirs has prescribed, and improper venue
for the complaint was filed in the court ( before the RTC of San Carlos, Negros Occidental.)

The RTC denied the Omnibus Motion, prompting Treyes to file before the Court of Appeals
(CA) a petition for Certiorari under Rule 65. The CA, however, denied the same.

ISSUE and HOLDING:

3) Whether there is a need for prior determination of heirship in a special proceeding prior
to filing an action for recovery of ownership and possession of property
No. Treyes contends that the petitioners have not established their right as legal heirs and is a
prerequisite to an ordinary suit; hence, their action for reconveyance should be dismissed.

The court held that the establishment of right of the heirs is conferred by law and there is no need
for judicial confirmation to establish petitioners as heirs. It was already established by the
petitioners that they are heirs ipso facto jure (by the law itself) , thus there is no need for any
judicial confirmation. The complaint alleges that their rights over the properties is by virtue of
their being siblings of the decedent.

In Article 777 of the Civil Code, it substantially provides that rights of succession is transmitted
upon the moment of death of the decedent. This much to say that the title or rights is
immediately passed to the heirs upon death. Thus, the heirs have legally been deemed to have
acquired ownership over the estate of the decedent, without need of any declaration.

In partition, even before a property is judicially partitioned, heirs are already deemed owners and
without need for prior separate judicial declaration of their heirship.

In a summary settlement of estates, heirs may undertake the extrajudicial settlement of estate of
decedent amongst themselves through execution of a public instrument without prior declaration
in separate judicial proceeding that they are heirs of the decedent.

Article 1001 likewise provides that brothers and sisters or their children who survive with the
widow or widower are entitled to one-half of the inheritance, the other half to the surviving
spouse.

1) Whether the action should be dismissed for improper venue

In this case, in the first motion to dismiss, Treyes only raised lack of jurisdiction over the
petitioner. The defense for improper venue was very much available at the time of filing. Thus,
raising the defense of improper venue although would not have been prejudicial to the petitioner,
there is no valid justification for the failure to invoke such defense.

2) Whether the action has prescribed

No, the defense of prescription of the complaint has no merit. Treyes invoked prescription on the
basis of Rule 74 is inconsistent with his main theory that the complaint is an ordinary civil action
and not a special proceeding. The provisions of Rule 74, Section 4 barring distributees or heirs
from objecting to an extrajudicial partition after the expiration of two years from such
extrajudicial partition is applicable only:

1. to persons who have participated or taken part or had notice of the extrajudicial partition, and
2. when the provisions of Section 1 of Rule 74 have been strictly complied with, i.e., that all the
persons or heirs of the decedent have taken part in the extrajudicial settlement or are represented
by themselves or through guardians.

The court held that both requirements are absent in this case; thus, the prescriptive period on
constructive trust under the Civil Code, particularly under implied constructive trust, applies, and
not the prescription on Special Proceedings.

The Civil Code identifies 2 kinds of Trusts:

1. Implied – by operation of law


2. Express – by intention of the parties

Under Implied Trust, there are two more categories:

1. Resulting Trust
– disposition of property which raises an inference that he does not intent the person holding the
property to have any beneficial interest
2. Constructive Trust
– subject to equitable duty to convey to another, on ground that he would be unjustly enriched if
he were permitted to retain it
– The duty to acquire property arises because of fraud, duress, undue influence, mistake,
wrongful disposition, breach of fiduciary duty.

In an action for reconveyance based on implied constructive trust, the law provides a prescription
of 10 years from issuance of the torrens title over the property, which is based in article 1144 of
the civil code. This issuance of title operates as a contructive notice to the whole world, which
the discovery of fraud is deemed to have taken place at the time of such issuance.

[CASE DIGEST] Estate of K.H. Hemady v. Luzon Surety (G.R.


No. L-8437)
November 28, 1956

FACTS:
Luzon Surety filed a claim against the estate of K.H. Hemady based on 20 different indemnity
agreements or counter bonds, each subscribed by a distinct principal and by the deceased K.H.
Hemady, a surety solidary (guarantor) in all of them.

Luzon Surety prayed for allowance, as a contingent claim, of the value of the twenty bonds it had
executed in consideration of the counterbonds, and further asked for judgment for the unpaid
premiums and documentary stamps affixed to the bonds, with 12% interest thereon.

Upon motion of the administratrix of Hemady’s estate, the Court of First Instance dismissed the claims
of Luzon Surety on two grounds. First, the CFI held that the premiums due and cost of documentary
stamps were not contemplated under the indemnity agreements to be a part of the undertaking of the
guarantor (Hemady), since they were not liabilities incurred after the execution of the counterbonds

And second, the CFI held that whatever losses may occur after Hemady’s death are not chargeable to
his estate, because upon his death he ceased to be guarantor.

Hence, the instant petition.

ISSUE:

Whether Hemady’s liability as a solidary guarantor is extinguished by his death. – NO.

HELD:

The Supreme Court ruled that Hemady’s liability as a solidary guarantor is not extinguished by his
death.

While in our successional system the responsibility of the heirs for the debts of their decedent cannot
exceed the value of the inheritance they receive from him, the principle remains intact that these
heirs succeed not only to the rights of the deceased but also to his obligations.

Under the Civil Code, the heirs, by virtue of the rights of succession, are subrogated to all the rights
and obligations of the deceased (Art. 661) and cannot be regarded as third parties with respect to a
contract to which the deceased was a party, touching the estate of the deceased. The heirs take such
property subject to all the obligations resting thereon in the hands of him from whom they derive their
rights.

[CASE DIGEST] Uson v. Del Rosario (G.R. No. L-4963)


December 10, 1963

FACTS:

This is an action for recovery of the ownership and possession of five parcels of land situated in
Labrador, Pangasinan, filed by Maria Uson against Maria del Rosario and her four children.
Uson alleged that she was the lawful wife of Faustino Nebreda who upon his death in 1945 left the
lands involved in this litigation. Nebreda left no other heir except his widow Uson. However, she
claimed that when Nebreda died in 1945, his common-law wife del Rosario illegally took possession of
said lands.

In their answer, defendants set up as special defense that on February 21, 1931, Maria Uson and her
husband, the late Faustino Nebreda, executed a public document whereby they agreed to separate as
husband and wife and, in consideration of their separation, Maria Uson was given a parcel of land by
way of alimony and in return she renounced her right to inherit any other property that may be left by
her husband upon his death.

The court rendered decision ordering the defendants to restore to the plaintiff the ownership and
possession of the lands in dispute. Defendants interposed the present appeal.

ISSUE:

Whether Uson is the rightful heir despite the agreement she had with her deceased husband to
separate. – YES.

HELD:

The Supreme Court held that the property belongs to the heirs at the moment of the death of the
ancestor as completely as if the ancestor had executed and delivered to them a deed for the same
before his death. From that moment, therefore, the rights of inheritance of Maria Uson over the lands
in question became vested.

The claim of the defendants that Uson had relinquished her right over the lands in question because
she expressly renounced to inherit any future property that her husband may acquire and leave upon
his death in the deed of separation they had entered into on February 21, 1931, cannot be entertained
for the simple reason that future inheritance cannot be the subject of a contract nor can it be
renounced.

De Borja vs. De Borja

FACTS: Francisco De Borja was appointed as the executer and administrator of the estate of his wife
Josefa Tangco with Jose Borja, their son, as co-administrator. After Josefa’s death, Francisco took a
second wife, Tasiana Ongsingco. Later on, Francisco died. Jose then became the sole administrator of
Josefa’s testate estate. Meanwhile, Tasiana instituted testate proceedings in the CFI Nueva Ecija upon
which she was appointed as special adminitratrix of Francisco’s estate. The relationship between the
children of the first marriage and Tasiana Ongsingco has been plagued with several court suits and
counter-suits. In order to put an end to all these litigations, a compromise agreement was entered into
between Jose and Tasiana. Under the compromise agreement, it was stipulated that Jose shall pay
Tasiana the amount of P800,000 and this shall be considered as full and complete payment and
settlement of her hereditary share in the estate of the late Francisco de Borja as well as the estate of
Josefa Tangco. Jose submitted the compromise agreement for approval tothe CFI Rizal and again to the
CFI Nueva Ecija. Tasiana opposed in both circumstances. The CFI Rizal approved the compromise
agreement but the CFI Nueva Ecija declared it void and unenforceable. Hence, this appeal by Jose for the
order of disapproval by the CFI Nueva Ecija and appeal by Tasiana for the approval by CFI Rizal. Tasiana
attacked the validity of the compromise agreement on the ground that the heirs cannot enter into such
kind of agreement without first probating the will of Francisco. In assailing its validity, Tasiana and the
CFI Nueva Ecija relied on the SC’s decision in Guevara vs. Guevara. 74 Phil. 479 wherein it was held that
the presentation of a will for probate is mandatory and that the settlement and distribution of an estate
on the basis of intestacy when the decedent left a will, is against the law and public policy.

ISSUE: Whether or not the compromise agreement between Jose and Tasiana (heirs of Francisco) is
valid.

RULING: Yes. The doctrine of Guevara vs. Guevara, ante, is not applicable to the case at bar. This is
apparent from an examination of the terms of the agreement between Jose and Tasiana which
stipulates that that Jose shall pay Tasiana the amount of P800,000 and this shall be considered as full
and complete payment and settlement of her hereditary share in the estate of the late Francisco de
Borja as well as the estate of Josefa Tangco. This provision evidences beyond doubt that the ruling in the
Guevara case is not applicable to the cases at bar. There was here no attempt to settle or distribute the
estate of Francisco de Borja among the heirs thereto before the probate of his will. The clear object of
the contract was merely the conveyance by Tasiana Ongsingco of any and all her individual share and
interest, actual or eventual in the estate of Francisco de Borja and Josefa Tangco. There is no stipulation
as to any other claimant, creditor or legatee. And as a hereditary share in a decedent's estate is
transmitted or vested immediately from the moment of the death of such causante or predecessor in
interest (Civil Code of the Philippines, Art. 777)3 there is no legal bar to a successor (with requisite
contracting capacity) disposing of her or his hereditary share immediately after such death, even if the
actual extent of such share is not determined until the subsequent liquidation of the estate. Of course,
the effect of such alienation is to be deemed limited to what is ultimately adjudicated to the vendor
heir. However, the aleatory character of the contract does not affect the validity of the transaction;
neither does the coetaneous agreement that the numerous litigations between the parties (the
approving order of the Rizal Court enumerates fourteen of them, Rec. App. pp. 79-82) are to be
considered settled and should be dismissed, although such stipulation, as noted by the Rizal Court, gives
the contract the character of a compromise that the law favors, for obvious reasons, if only because it
serves to avoid a multiplicity of suits. It is likewise worthy of note in this connection that as the surviving
spouse of Francisco de Borja, Tasiana Ongsingco was his compulsory heir under article 995 et seq. of the
present Civil Code. Wherefore, barring unworthiness or valid disinheritance, her successional interest
existed independent of Francisco de Borja's last will and testament and would exist even if such will
were not probated at all. Thus, the prerequisite of a previous probate of the will, as established in the
Guevara and analogous cases, can not apply to the case of Tasiana Ongsingco Vda. de de Borja.
Case Digest: Bonilla v. Barcena, G.R. No. L-41715
June 18, 1976
August 31, 2023

Art. 777 | Succession, Death of


a Party

Provision:

Art. 777. The rights to the succession are transmitted from the moment
of the death of the decedent.

Facts:
On March 1975, Fortunata Barcena, mother of minors Rosalio
Bonilla and Salvacion Bonilla and wife of Ponciano Bonilla, instituted a
civil action in the Court of First Instance of Abra, to quiet title ( a legal
action that is intended to clarify ownership of a given property) over
certain parcels of land located in Abra.

On May 1975, Defendants filed a motion to dismiss the complaint.


Before the motion was heard, the plaintiff's counsel requested to amend
the complaint, which was allowed.

On July 9, 1975, Fortunata Barcena died.

Defendants filed another motion to dismiss on the ground that Fortunata


Barcena is dead and, therefore, has no legal capacity to sue.

The court immediately dismissed the case.


The court denied the motion for reconsideration filed by counsel for the
plaintiff.

Counsel for deceased plaintiff filed a written manifestation praying that


the minors Rosalio Bonilla and Salvacion Bonilla be allowed to substitute
their deceased mother, but the court denied the counsel's prayer for lack
of merit.

Nature:
Petition for Review.

Issue:
WoN the death of Fortunata Barcena extinguished the civil action to
quiet title. (NO)

Held:
Petition granted.

While it is true that a person who is dead cannot sue in court, yet he can
be substituted by his heirs in pursuing the case up to its completion. The
records of this case show that the death of Fortunata Barcena took place
on July 9, 1975 while the complaint was filed on March 31, 1975. This
means that when the complaint was filed on March 31, 1975, Fortunata
Barcena was still alive, and therefore, the court had acquired jurisdiction
over her person.

If thereafter she died, the Rules of Court prescribes the procedure


whereby a party who died during the pendency of the proceeding can be
substituted. Under Section 16, Rule 3 of the Rules of Court "whenever a
party to a pending case dies ... it shall be the duty of his attorney to
inform the court promptly of such death ... and to give the name and
residence of his executor, administrator, guardian or other legal
representatives." This duty was complied with by the counsel for the
deceased plaintiff when he manifested before the respondent Court that
Fortunata Barcena died on July 9, 1975 and asked for the proper
substitution of parties in the case. The respondent Court, however,
instead of allowing the substitution, dismissed the complaint on the
ground that a dead person has no legal personality to sue. This is a grave
error.

Article 777 of the Civil Code provides "that the rights to the succession
are transmitted from the moment of the death of the decedent." From the
moment of the death of the decedent, the heirs become the absolute
owners of his property, subject to the rights and obligations of the
decedent, and they cannot be deprived of their rights thereto except by
the methods provided for by law. The moment of death is the determining
factor when the heirs acquire a definite right to the inheritance whether
such right be pure or contingent. The right of the heirs to the property of
the deceased vests in them even before judicial declaration of their being
heirs in the testate or intestate proceedings. When Fortunata Barcena,
therefore, died her claim or right to the parcels of land in litigation in Civil
Case No. 856, was not extinguished by her death but was transmitted to
her heirs upon her death. Her heirs have thus acquired interest in the
properties in litigation and became parties in interest in the case. There
is, therefore, no reason for the respondent Court not to allow their
substitution as parties in interest for the deceased plaintiff.

The question as to whether an action survives or not depends on the


nature of the action and the damage sued for. In the causes of action
which survive the wrong complained affects primarily and principally
property and property rights, the injuries to the person being merely
incidental, while in the causes of action which do not survive the injury
complained of is to the person, the property and rights of property
affected being incidental.

Following the foregoing criterion the claim of the deceased plaintiff which
is an action to quiet title over the parcels of land in litigation affects
primarily and principally property and property rights and therefore is one
that survives even after her death. It is, therefore, the duty of the
respondent Court to order the legal representative of the deceased
plaintiff to appear and to be substituted for her.

The respondent Court refused the request for substitution on the ground
that the children were still minors and cannot sue in court. This is
another grave error because the respondent Court ought to have known
that under the same Section 17, Rule 3 of the Rules of Court, the court is
directed to appoint a guardian ad litem for the minor heirs.

Case Digest: Bonilla v. Barcena, G.R. No. L-41715 June 18, 1976
August 31, 2023

Provision:

Art. 777. The rights to the succession are transmitted from the
moment of the death of the decedent.
Facts:
Fortunata Barcena, mother of Rosalio Bonilla and Salvacion
Bonilla and wife of Ponciano Bonilla, instituted a civil action in the
Court, to quiet title (a legal action that is intended to clarify ownership
of a given property) on a certain parcels of land located in Abra.

Barcena filed a motion to dismiss the complaint. Before the motion was
heard, the Bonilla's lawyer requested to amend the complaint, which
was allowed.

But Fortunata Barcena died.

Barcena filed another motion to dismiss on the ground that Fortunata


Barcena is dead and, therefore, has no legal capacity to sue.

The court immediately dismissed the case.


The court denied the motion for reconsideration filed by counsel for
the plaintiff.

Counsel for deceased plaintiff filed a written manifestation praying


that the minors Rosalio Bonilla and Salvacion Bonilla be allowed to
substitute their deceased mother, but the court denied the counsel's
prayer for lack of merit.

Nature:
Petition for Review.

Issue:
WoN the death of Fortunata Barcena extinguished the civil action to
quiet title. (NO)

Held:
Petition granted.

While it is true that a person who is dead cannot sue in court, yet he
can be substituted by his heirs in pursuing the case up to its
completion. The records of this case show that the death of Fortunata
Barcena took place on July 9, 1975 while the complaint was filed on
March 31, 1975. This means that when the complaint was filed on
March 31, 1975, Fortunata Barcena was still alive, and therefore, the
court had acquired jurisdiction over her person.
If thereafter she died, the Rules of Court prescribes the procedure
whereby a party who died during the pendency of the proceeding can
be substituted. Under Section 16, Rule 3 of the Rules of Court
"whenever a party to a pending case dies ... it shall be the duty of his
attorney to inform the court promptly of such death ... and to give the
name and residence of his executor, administrator, guardian or other
legal representatives." This duty was complied with by the counsel for
the deceased plaintiff when he manifested before the respondent
Court that Fortunata Barcena died on July 9, 1975 and asked for the
proper substitution of parties in the case. The respondent Court,
however, instead of allowing the substitution, dismissed the complaint
on the ground that a dead person has no legal personality to sue. This
is a grave error.

Article 777 of the Civil Code provides "that the rights to the
succession are transmitted from the moment of the death of the
decedent." From the moment of the death of the decedent, the heirs
become the absolute owners of his property, subject to the rights and
obligations of the decedent, and they cannot be deprived of their rights
thereto except by the methods provided for by law. The moment of
death is the determining factor when the heirs acquire a definite right
to the inheritance whether such right be pure or contingent. The right
of the heirs to the property of the deceased vests in them even before
judicial declaration of their being heirs in the testate or intestate
proceedings. When Fortunata Barcena, therefore, died her claim or
right to the parcels of land in litigation in Civil Case No. 856, was not
extinguished by her death but was transmitted to her heirs upon her
death. Her heirs have thus acquired interest in the properties in
litigation and became parties in interest in the case. There is,
therefore, no reason for the respondent Court not to allow their
substitution as parties in interest for the deceased plaintiff.

The question as to whether an action survives or not depends on the


nature of the action and the damage sued for. In the causes of action
which survive the wrong complained affects primarily and principally
property and property rights, the injuries to the person being merely
incidental, while in the causes of action which do not survive the
injury complained of is to the person, the property and rights of
property affected being incidental.
Following the foregoing criterion the claim of the deceased plaintiff
which is an action to quiet title over the parcels of land in litigation
affects primarily and principally property and property rights and
therefore is one that survives even after her death. It is, therefore, the
duty of the respondent Court to order the legal representative of the
deceased plaintiff to appear and to be substituted for her.

The respondent Court refused the request for substitution on the


ground that the children were still minors and cannot sue in court. This
is another grave error because the respondent Court ought to have
known that under the same Section 17, Rule 3 of the Rules of
Court, the court is directed to appoint a guardian ad litem for the
minor heirs.

VITUG V. COURT OF APPEALS,


G.R. NO. 82027, [MARCH 29,
1990], 262 PHIL 830-838
FACTS: On January 13, 1985, Romarico G. Vitug filed a motion asking
for authority from the probate court to sell certain shares of stock and
real properties belonging to the estate to cover allegedly his advances
to the estate in the sum of P667,731.66, plus interests, which he
claimed were personal funds. As found by the Court of Appeals, the
alleged advances consisted of P58,147.40 spent for the payment of
estate tax, P518,834.27 as deficiency estate tax, and P90,749.99 as
“increment thereto.” According to Mr. Vitug, he withdrew the sums of
P518,834.27 and P90,749.99 from savings account No. 35342-038 of
the Bank of America, Makati, Metro Manila.

On April 12, 1985, Rowena Corona opposed the motion to sell on the
ground that the same funds withdrawn from savings account No.
35342-038 were conjugal partnership properties and part of the estate,
and hence, there was allegedly no ground for reimbursement. She also
sought his ouster for failure to include the sums in question for
inventory and for “concealment of funds belonging to the estate.”

Vitug insists that the said funds are his exclusive property having
acquired the same through a survivorship agreement executed with
his late wife and the bank on June 19, 1970. The agreement provides:

We hereby agree with each other and with the BANK OF AMERICAN
NATIONAL TRUST AND SAVINGS ASSOCIATION (hereinafter referred
to as the BANK), that all money now or hereafter deposited by us or
any or either of us with the BANK in our joint savings current account
shall be the property of all or both of us and shall be payable to and
collectible or withdrawable by either or any of us during our lifetime,
and after the death of either or any of us shall belong to and be the
sole property of the survivor or survivors, and shall be payable to and
collectible or withdrawable by such survivor or survivors.

We further agree with each other and the BANK that the receipt or
check of either, any or all of us during our lifetime, or the receipt or
check of the survivor or survivors, for any payment or withdrawal
made for our above-mentioned account shall be valid and sufficient
release and discharge of the BANK for such payment or withdrawal.

ISSUE:

A. WON A SURVIVORSHIP AGREEMENT IS NEITHER A


DONATION INTER VIVOS NOR A DONATION MORTIS CAUSA.

B. WON SPOUSES ARE ALLOWED TO ENTER INTO A


SURVIVORSHIP AGREEMENT.

HELD:

A. YES, The conveyance in question is not, first of all, one


of mortis causa, which should be embodied in a will. A will
has been defined as “a personal, solemn, revocable and free
act by which a capacitated person disposes of his property
and rights and declares or complies with duties to take effect
after his death.” In other words, the bequest or device must
pertain to the testator. In this case, the monies subject of
savings account No. 35342-038 were in the nature of conjugal
funds.|||Neither is the survivorship agreement a donation inter
vivos, for obvious reasons, because it was to take effect after
the death of one party. Secondly, it is not a donation between
the spouses because it involved no conveyance of a spouse’s
own properties to the other.

B. YES, the spouses are not prohibited by law to invest


conjugal property, say, by way of a joint and several bank
account, more commonly denominated in banking parlance as
an “and/or” account. In the case at bar, when the spouses
Vitug opened savings account No. 35342-038, they merely put
what rightfully belonged to them in a money-making venture.
They did not dispose of it in favor of the other, which would
have arguably been sanctionable as a prohibited donation. And
since the funds were conjugal, it can not be said that one
spouse could have pressured the other in placing his or her
deposits in the money pool.

The validity of the contract seems debatable by reason of its


“survivor-take-all” feature, but in reality, that contract imposed
a mere obligation with a term, the term being death. Such
agreements are permitted by the Civil Code.

REPORT THIS AD

The conclusion is accordingly unavoidable that Mrs. Vitug


having predeceased her husband, the latter has acquired upon
her death a vested right over the amounts under savings
account No. 35342-038 of the Bank of America. Insofar as the
respondent court ordered their inclusion in the inventory of
assets left by Mrs. Vitug, we hold that the court was in error.
Being the separate property of petitioner, it forms no more part
of the estate of the deceased.

Vitug vs. CA Digest


Vitug v. Court of Appeals

Facts:

1. The case is a chapter in an earlier suit involving the issue on two (2) wills of the late Dolores Vitug
who died in New York, USA in Nov 1980. She named therein private respondent Rowena Corona
(Executrix) while Nenita Alonte was co-special administrator together with petitioner Romarico
pending probate.

2. In January 1985, Romarico filed a motion asking for authorization of the probate court to sell
shares of stocks and real property of the estate as reimbursements for advances he made to the
estate. The said amount was spent for payment of estate tax from a savings account in the Bank of
America.

3. Rowena Corona opposed the motion to sell contending that from the said account are conjugal
funds, hence part of the estate. Vitug insisted saying that the said funds are his exclusive property
acquired by virtue of a survivorship agreement executed with his late wife and the bank previously.
In the said agreement, they agreed that in the event of death of either, the funds will become the
sole property of the survivor.

4. The lower court upheld the validity of the survivorship agreement and granted Romarico's motion
to sell. The Court of Appeals however held that said agreement constituted a conveyance mortis
causa which did not comply with the formalities of a valid will. Further, assuming that it is
donation inter vivos, it is a prohibited donation. Vitug petitioned to the Court contending that the said
agreement is an aleatory contract.

Issue: Whether or not the conveyance is one of mortis causa hence should conform to the
form required of wills

NO. The survivorship agreement is a contract which imposed a mere obligation with a term--being
death. Such contracts are permitted under Article 2012 on aleatory contracts. When Dolores
predeceased her husbandm the latter acquired upon her death a vested right over the funds in the
account. The conveyance is therefore not mortis causa.

Vitug vs. CA Digest


Vitug v. Court of Appeals

Facts:

1. The case is about the issue on two (2) wills of the Dolores Vitug who died in New York, USA. She
appointed Rowena Corona (Executrix) An executrix refers to a woman who has been
assigned responsibility for executing the provisions set forth in a last will and
testament. while Nenita Alonte was co-special administrator together with petitioner Romarico
pending probate.

2. Romarico filed a motion asking for authorization of the probate court to sell shares of stocks and
real property of the estate as reimbursements for advances he made to the estate. The said amount
was spent for payment of estate tax from a savings account in the Bank of America.

3. So Rowena Corona opposed the motion to sell saying that from the said account are conjugal
funds, as part of the estate. Vitug insisted saying that the said funds are his property acquired by
virtue of a survivorship agreement executed with his late wife and the bank previously. In the said
agreement, they agreed that in the event of death of either, the funds will become the sole property
of the survivor.

4. The lower court upheld the validity of the survivorship agreement and granted Romarico's motion
to sell. The Court of Appeals however held that said agreement constituted a conveyance mortis
causa which did not comply with the formalities of a valid will. Further, assuming that it is
donation inter vivos, it is a prohibited donation. Vitug petitioned to the Court contending that the said
agreement is an aleatory contract.

Issue: Whether or not the conveyance is one of mortis causa hence should conform to the
form required of wills

NO. The survivorship agreement is a contract which imposed a mere obligation with a term--being
death. Such contracts are permitted under Article 2012 on aleatory contracts. When Dolores
predeceased her husbandm the latter acquired upon her death a vested right over the funds in the
account. The conveyance is therefore not mortis causa.
EDUARDO FELIPE v. HEIRS OF MAXIMO ALDON, GR No. 60174, 1983-02-16

Facts:

Maximo Aldon married Gimena Almosara in 1936. The spouses bought


several pieces of land sometime between 1948 and 1950. In 1960-62, the
lands were divided into three lots, 1370, 1371 and 1415 of the San Jacinto
Public Land Subdivision, San Jacinto, Masbate.

In 1951, Gimena Almosara sold the lots to the spouses Eduardo Felipe
and Hermogena V. Felipe. The sale was made without the consent of her
husband, Maximo.

On April 26, 1976, the heirs of Maximo Aldon, namely his widow Gimena
and their children Sofia and Salvador Aldon, filed a complaint in the Court
of First Instance of Masbate against the Felipes. The complaint which
was docketed as Civil Case No. 2372 alleged that the... plaintiffs were the
owners of Lots 1370, 1371 and 1415; that they had orally mortgaged the
same to the defendants; and an offer to redeem the mortgage had been
refused so they filed the complaint in order to recover the three parcels of
land.

The defendants asserted that they had acquired the lots from the
plaintiffs by purchase and subsequent delivery to them. The trial court
sustained the claim of the defendants and rendered the following
judgment:

"a. declaring the defendants to be the lawful owners of the property


subject of the present litigation;... b. declaring the complaint in the preset
action to be without merit and is therefore hereby ordered dismissed;... c.
ordering the plaintiffs to pay to the defendants the amount of P2,000.00
as reasonable attorney's fees and to pay the costs of the suit."

The plaintiffs appealed the decision to the Court of Appeals which


rendered the following judgment:

"PREMISES CONSIDERED, the decision appealed from is hereby


REVERSED and SET ASIDE, and a new one is hereby RENDERED, ordering
the defendants-appellees to surrender the lots in question as well as the
plaintiffs'-appellants' muniments of title thereof to said... plaintiffs-
appellants, to make an accounting of the produce derived from the lands
including expenses incurred since 1951, and to solidarily turn over to the
plaintiffs-appellants the NET monetary value of the profits, after
deducting the sum of P1,800.00. No attorney's fees nor... moral damage
are awarded for lack of any legal justification therefor. No costs."

Issues:

The sale made by Gimena is certainly a defective contract but of what


category?

H... ave the petitioners acquired the lands by acquisitive prescription? (2)
Is the right of action of Sofia and Salvador Aldon barred by the statute of
limitations?

Have the petitioners acquired the lands by acquisitive prescription? (2) Is


the right of action of Sofia and Salvador Aldon barred by the statute of
limitations?

Ruling:

it is a voidable contract.

According to Art. 1390 of the Civil Code, among the voidable contracts
are "[T]hose where one of the parties is incapable of giving consent to the
contract." (Par. 1.) In the instant case Gimena had no capacity to give
consent to the contract of sale. The capacity to give... consent belonged
not even to the husband alone but to both spouses.

The view that the contract made by Gimena is a voidable contract is


supported by the legal provision that contracts entered by the husband
without the consent of the wife when such consent is required, are
annullable at her instance during the marriage and within ten years...
from the transaction questioned. (Art. 173, Civil Code)

Gimena's contract is not rescissible for in such a contract all the


essential elements are untainted but Gimena's consent was tainted.
Neither can the contract be classified as unenforceable because it does
not fit any of those described in Art. 1403 of the Civil Code. And... finally,
the contract cannot be void or inexistent because it is not one of those
mentioned in Art. 1409 of the Civil Code. By process of elimination, it
must perforce be a voidable contract.

The voidable contract of Gimena was subject to annulment by her


husband only during the marriage because he was the victim who had an
interest in the contract. Gimena, who was the party responsible for the
defect, could not ask for its annulment. Their children could not...
likewise seek the annulment of the contract while the marriage subsisted
because they merely had an inchoate right to the lands sold.
The termination of the marriage and the dissolution of the conjugal
partnership by the death of Maximo Aldon did not improve the situation of
Gimena. What she could not do during the marriage, she could not do
thereafter.

The case of Sofia and Salvador Aldon is different. After the death of
Maximo they acquired the right to question the defective contract insofar
as it deprived them of their hereditary rights in their father's share in the
lands. The father's share is one-half (1/2) of the... lands and their share is
two-thirds (2/3) thereof, one-third (1/3) pertaining to the widow.

SHORTHAND DIGEST

 This petition for review on a CA decision involved the wife selling portions of the lots without
the husband’s consent. The Supreme Court ruled on behalf of the defendant-heirs of the lot
in question, as the sale in dispute was a voidable contract as it was under conjugal
partnership and was sold without the consent of the husband.

UNION BANK V.
SANTIBANEZ Case Digest
ON OCTOBER 4, 2020 BY CHESKAMHEY IN UNCATEGORIZED

February 23, 2005

CALLEJO, SR., J.
DOCTRINE:

The law enjoins the probate of a will and the public requires it, because unless
a will is probated and notice thereof given to the whole world, the right of a person to
dispose of his property by will may be rendered nugatory.

FACTS:

First Countryside Credit Corporation (FCCC) and Efraim Santibañez entered into
two (2) loan agreement. The amount was intended for the payment of (2) two unit
Ford 6600 Agricultural Tractor and one (1) unit of a Rotamotor. In view thereof,
Efraim and his son, Edmund, executed a promissory note in favor of the FCCC.

However, Efraim died, leaving a holographic will. Testate proceedings commenced


before the RTC of Iloilo City. Edmund, as one of the heirs, was appointed as the
special administrator of the estate of the decedent. During the pendency of the
testate proceedings, the surviving heirs, Edmund and his sister Florence Santibañez
Ariola, executed a Joint Agreement, wherein they agreed to divide between
themselves and take possession of the three (3) tractors; that is, two (2) tractors for
Edmund and one (1) tractor for Florence. Each of them was to assume the
indebtedness of their late father to FCCC, corresponding to the tractor respectively
taken by them.

Meanwhile, Deed of Assignment with Assumption of Liabilities was executed by and


between FCCC and Union Savings and Mortgage Bank, wherein the FCCC as the
assignor, among others, assigned all its assets and liabilities to Union Savings and
Mortgage Bank.

REPORT THIS AD
Demand letters for the settlement of his account were sent by petitioner Union Bank
of the Philippines (UBP) to Edmund, but the latter failed to heed the same and
refused to pay.

UBP asserted that the obligation of the deceased had passed to his legitimate
children and heirs; the unconditional signing of the joint agreement estopped
respondent Florence S. Ariola, and that she cannot deny her liability under the said
document; as the agreement had been signed by both heirs in their personal
capacity, it was no longer necessary to present the same before the probate court
for approval; the property partitioned in the agreement was not one of those
enumerated in the holographic will made by the deceased; and the active
participation of the heirs, particularly respondent Florence S. Ariola, in the present
ordinary civil action was tantamount to a waiver to re-litigate the claim in the estate
proceedings.

Hence this petition.

ISSUE:

Whether obligations of the deceased were transmitted to the heirs as provided


in Article 774 of the Civil Code.

RULING:

No, Perusing the joint agreement, it provides that the heirs as parties thereto
“have agreed to divide between themselves and take possession and use the
above-described chattel and each of them to assume the indebtedness
corresponding to the chattel taken as herein after stated which is in favor of First
Countryside Credit Corp.” The assumption of liability was conditioned upon the
happening of an event, that is, that each heir shall take possession and use of their
respective share under the agreement. It was made dependent on the validity of the
partition, and that they were to assume the indebtedness corresponding to the
chattel that they were each to receive. The partition being invalid as earlier
discussed, the heirs in effect did not receive any such tractor. It follows then that the
assumption of liability cannot be given any force and effect.

TESTATE ESTATE OF LATE REVEREND FATHER PASCUAL RIGOR.


PARISH PRIEST OF ROMAN CATHOLIC CHURCH OF VICTORIA v. BELINA
RIGOR, GR No. L-22036, 1979-04-30

Facts:

ricelands located at Guimba, Nueva Ecija, with a total area of around


forty-four hectares. That devise was made in the will of... the late Father
Pascual Rigor, a native of Victoria, Tarlac, in favor of his nearest male
relative who would study for the priesthood.

Father Rigor, the parish priest of Pulilan, Bulacan, died on August 9, 1935,
leaving a will executed on October 29, 1933

Named as devisces in the will were the testator's nearest relatives,


namely, his three sisters:

Florencia Rigor-Escobar, Belina Rigor-Manaloto and Nestora Rigor-


Quiambao. The testator gave a devise to his cousin, Fortunato
Gamalinda.

About thirteen years after the approval of the project of partition, or on


February 19, 1954, the parish priest of Victoria filed in the pending testate
proceeding a petition praying for the appointment of a new administrator

The intestate heirs of Father Rigor countered with a petition dated March
25, 1957 praying that the bequest be declared inoperative and that they
be adjudged as the persons entitled to the said ricelands

Issues:

efficaciousness or enforceability of a devise of ricelands

Ruling:

From the... testamentary provisions, it may be deduced that the testator


intended to devise the ricelands to his nearest male relative who would
become a priest, who was forbidden to sell the ricelands, who would lose
the devise if he... discontinued his studies for the priesthood, or having
been ordained a priest, he was excommunicated, and who would be
obligated to say annually twenty masses with prayers for the repose of
the souls of the testator and his parents.

On the other hand, it is clear that the parish priest of Victoria would
administer the ricelands only in two situations: one, during the interval
of time that no nearest male relative of the testator was studying... for
the priesthood and two, in case the testator's nephew became a priest
and he was excommunicated.

We hold that the said bequest refers to the testator's nearest male
relative living at the time of his death and not to any indefinite time
thereafter.

Inasmuch as the testator was not survived by any nephew who became a
priest, the unavoidable conclusion is that the bequest in question was
ineffectual or inoperative. Therefore, the administration of the ricelands
by the parish priest... of Victoria, as envisaged in the will, was likewise
inoperative.

Principles:

The will of the testator is the first and principal law in the matter of
testaments. When his intention is clearly and precisely expressed, any
interpretation must be in accord with the plain and literal meaning of his
words, except when it may... certainly appear that his intention was
different from that literally expressed (In re Estate of Calderon, 26 Phil.
333).

One canon in the interpretation of the testamentary provisions is that


"the testator's intention is to be ascertained from the words of the will,
taking into consideration the circumstances under which it was made",
but excluding the testator's oral declarations as to... his intention (Art.
789, Civil Code of the Philippines).

"In order to be capacitated to inherit, the heir, devisee or legatee must


be... living at the moment the succession opens, except in case of
representation, when it is proper" (Art. 1025, Civil Code).

The Court of Appeals correctly ruled that this case is covered by article
888 of the old Civil Code, now article 956, which provides that if "the
bequest for any reason should be inoperative, it shall be merged into the
estate, except in cases of substitution and those... in which the right of
accretion exists"
This case is also covered by article 912(2) of the old Civil Code, now
article 960(2), which provides that legal succession takes place when the
will "does not dispose of all that belongs to the testator". There being no
substitution nor accretion as... to the said ricelands, the same should be
distributed among the testator's legal heirs. The effect is as if the
testator had made no disposition as to the said ricelands.

The Civil Code recognizes that a person may die partly testate and partly
testate, or that there may be mixed succession.

Thus, if a conditional legacy... does not take effect, there will be intestate
succession as to the property covered by the said legacy (Macrohon Ong
Ham vs. Saavedra, 51 Phil. 267).

TESTATE ESTATE OF THE LATE REVEREND FATHER PASCUAL RIGOR. THE PARISH PRIEST OF THE ROMAN
CATHOLIC CHURCH OF VICTORIA, TARLAC, petitioner-appellant, vs. BELINA RIGOR, NESTORA RIGOR,
FRANCISCA ESCOBAR DE RIGOR and JOVITA ESCOBAR DE FAUSTO, respondents-appellees. G.R. No. L-
22036 April 30, 1979 FACTS: AQUINO, J.: Father Rigor, the parish priest of Pulilan, Bulacan, died on
August 9, 1935, leaving a will executed on October 29, 1933 which was probated by the Court of First
Instance of Tarlac in its order of December 5, 1935. In addition to the devices contained therein, the will
had a provision to the effect that the testator intended to devise the ricelands to his nearest male
relative who would become a priest. It was stated therein that the parish priest of Victoria would
administer the ricelands only in two situations: one, during the interval of time that no nearest male
relative of the testator was studying for the priesthood and two, in case the testator's nephew became a
priest and he was excommunicated. ISSUE: Whether or not a device in favour of a person whose identity
at the time of the testator’s death cannot be ascertained, may be efficacious. RULING: No. The Supreme
Court held that the said bequest refers to the testator's nearest male relative living at the time of his
death and not to any indefinite time thereafter. "In order to be capacitated to inherit, the heir, devisee
or legatee must be living at the moment the succession opens, except in case of representation, when it
is proper" (Art. 1025, Civil Code). The said testamentary provisions should be sensibly or reasonably
construed. To construe them as referring to the testator's nearest male relative at anytime after his
death would render the provisions difficult to apply and create uncertainty as to the disposition of his
estate. That could not have been his intention. The reasonable view is that he was referring to a
situation whereby his nephew living at the time of his death, who would like to become a priest, was still
in grade school or in high school or was not yet in the seminary. In that case, the parish priest of Victoria
would administer the ricelands before the nephew entered the seminary. But the moment the testator's
nephew entered the seminary, then he would be entitled to enjoy and administer the ricelands and
receive the fruits thereof. In that event, the trusteeship would be terminated. Following that
interpretation of the will the inquiry would be whether at the time Father Rigor died in 1935 he had a
nephew who was studying for the priesthood or who had manifested his desire to follow the
ecclesiastical career. That query is categorically answered in paragraph 4 of appellant priest's petitions
of February 19, 1954 and January 31, 1957. He unequivocally alleged therein that "not male relative of
the late (Father) Pascual Rigor has ever studied for the priesthood." Inasmuch as the testator was not
survived by any nephew who became a priest, the unavoidable conclusion is that the bequest in
question was ineffectual or inoperative. Therefore, the administration of the ricelands by the parish
priest of Victoria, as envisaged in the wilt was likewise inoperative. It should be understood that the
parish priest of Victoria could become a trustee only when the testator's nephew living at the time of his
death, who desired to become a priest, had not yet entered the seminary or, having been ordained a
priest, he was excommunicated. Those two contingencies did not arise, and could not have arisen in this
case because no nephew of the testator manifested any intention to enter the seminary or ever became
a priest. The Court of Appeals correctly ruled that this case is covered by article 888 of the old Civil Code,
now article 956, which provides that if "the bequest for any reason should be inoperative, it shall be
merged into the estate, except in cases of substitution and those in which the right of accretion exists."
This case is also covered by article 912(2) of the old Civil Code, now article 960 (2), which provides that
legal succession takes place when the will "does not dispose of all that belongs to the testator." There
being no substitution nor accretion as to the said ricelands the same should be distributed among the
testator's legal heirs. The effect is as if the testator had made no disposition as to the said ricelands.

MARGIE SANTOS MITRA v. PERPETUA L. SABLAN-GUEVARRA, GR No.


213994, 2018-04-18

Facts:

Margie Santos Mitra (petitioner) filed a petition for the probate of the
notarial will of Remedios Legaspi y Reyes (Legaspi) with prayer for
issuance of letters testamentary before the RTC.

petitioner is the de facto adopted daughter of Legaspi

Legaspi left a notarial will, instituting the petitioner, Orlando Castro,


Perpetua Sablan Guevarra, and Remigio Legaspi Sablan, as her heirs,
legatees and devisees; that Legaspi left real and personal properties

Perpetua L. Sablan-Guevarra and Remegio L. Sablan (respondents), who


claim to be Legaspi's legal heirs, opposed the petition. They aver that the
will was not executed in accordance with the formalities required by law;
that since the last page of the will, which contained the
Acknowledgement, was not signed by Legaspi and her instrumental
witnesses, the will should be declared invalid; that the attestation clause
failed to state the number of pages upon which the will was written; and
that the will was executed under undue and improper pressure, thus,
Legaspi could not have intended the document to be her last will and
testament.
the RTC rendered a Decision[7] admitting Legaspi's will to probate.

the CA reversed the judgment of the RTC, as the CA adhered to the view
of strictly complying with the requirement of stating the number of pages
of the will in the attestation clause. Moreover, the CA detected another
supposed fatal defect in the will: the photocopy of the will submitted by
the respondents on appeal did not contain the signatures of the
instrumental witnesses on each and every page thereof.

Issues:

Whether the CA erred in finding that the instrumental witnesses to the


will failed to sign on each and every page thereof on the left margin,
except the last, as required under Article 805 of the Civil CodeWhether
the CA erred in ruling that the failure to state the number of pages
comprising the will on the attestation clause renders such will defective

Ruling:

For their part, the respondents do not deny that the original copy of the
will, as opposed to its photocopy, bore the signatures of the instrumental
witnesses on every page thereof, except the last.[27] However, they
submit that they did not cause any alteration to the photocopied version.
They explain that since the folder holding the records of the case was
bound on the left margin and the pages may not be detached therefrom,
the left portion of the will must have been unintentionally excluded or cut-
off in the process of photocopying.

In any event, it is uncontested and can be readily gleaned that the


instrumental witnesses signed on each and every page of the will, except
the last page. Such being the case, the CA erred in concluding otherwise.
There is no doubt that the requirement under the Article 805 of the Civil
Code, which calls for the signature of the testator and of the instrumental
witnesses on each and every page of the will on the left margin, except
the last, was complied with.It should also be mentioned that the
respondents take a skewed stance in insisting that the testator Legaspi
and the instrumental witnesses should have signed on the last page of
the subject will. When Article 805 of the Civil Code requires the testator
to subscribe at the end of the will, it necessarily refers to the logical end
thereof, which is where the last testamentary disposition ends.[29] As the
probate court correctly appreciated, the last page of the will does not
contain any testamentary disposition; it is but a mere continuation of the
Acknowledgment.
What is imperative for the allowance of a will despite the existence of
omissions is that such omissions must be supplied by an examination of
the will itself, without the need of resorting to extrinsic evidence.

In sum, Legaspi's last will and testament has substantially complied with
all the formalities required of a notarial will. It has been proven that
Legaspi and the instrumental witnesses signed on every page of the will,
except on the last, which refers to the Acknowledgment page. With
regard to the omission of the number of pages in the attestation clause,
this was supplied by the Acknowledgment portion of the will itself
without the need to resort to extrinsic evidence. Contrary to the CA
conclusion, such omission does not in any way serve as hindrance to
probate.WHEREFORE, premises considered, the petition is GRANTED.

20. VDA. DE VILLAFLOR VS. JUICO GR. NO-L 15737 (Mistakes and Omission) FACTS: This is a case of direct
appeal on question of law from the decision of the Court of First Instance of Rizal, dismissing plaintiff-
appellant’s complaint for the recovery of certain properties that were originally owned by the plaintiff’s
granduncle, Nicolas Villaflor, which he granted to his widow, Dona Fausta Nepomuceno, bequeathing to
her “su usoy posesion mientras viva y no se case en segundas nupcias” The facts, appear of record that;
on Octber 9, 1908, Don Nicolas Villaflor, a wealthy man of Castillejos, Zambales, executed a will in
Spanish in his own handwriting, devising and bequeathing in favor of his wife, Dona Fausta
Nepomuceno, one-half of all his real and personal properties, giving the other half to his brother Don
Fausto Villaflor. Clause 6th, containing the institution of heirs, reads as follows: . SEXTO — En virtud de
las facultades que me conceden las leyes, instituyo per mis unicos y universales herederos de todos mis
derechos y acciones a mi hermano D. Fausto Villaflor y a mi esposa Da. Fausta Nepomuceno para que
partan todos mis bienes que me pertenescan, en iguales partes, para despues de mi muerte,
exceptuando las donaciones y legados que, abajo mi mas expontanea voluntad, lo hago en la forma
siguiente: . SEPTIMO: — Lego para dispues de mi muerte a mi esposa Da. Fausta Nepomuceno, en
prueba de mi amor y carino, los bienes, alhajas y muebles que a continuacion se expresan; . OCTAVO: —
Que estos legades disfrutaria mi referida esposa Da. Fausta Nepomuceno su uso y posesion mientras
viva y no se case en segundas nupcias, de la contrario, pasara a ser propiedad estos dichos legados de mi
sobrina nieta Leonor Villaflor. The 12th clause of the will provided, however, that Clauses 6th and 7th
thereof would be deemed annulled from the moment he bore any child with Doña Fausta Nepomuceno.
Said Clause 12th reads as follows: . DUODECIMO: — Quedan anulados las parrafos 6.0 y 7.0 de este
testamento que tratan de institucion de herederos y los legados que se haran despues de mi muerte a
favor de mi esposa, en el momento que podre tener la dicha de contrar con hijo y hijos legitimos o
legitimados, pues estos, conforme a ley seran mis herederos. Don Nicolas Villaflor died on March 3,
1922, without begetting any child with his wife Doña Fausta Nepomuceno. On May 1, 1956, Doña Fausta
Nepomuceno died without having contracted a second marriage, and without having begotten any child
with the deceased Nicolas Villaflor. Her estate is now being settled in Special Proceeding No. Q-1563 in
the lower court, with the defendant Delfin N. Juico as the duly appointed and qualified judicial
administrator. The plaintiff Leonor Villaflor Vda. de Villanueva is admitted to be the same Leonor
Villaflor mentioned by Don Nicolas Villaflor in his will as his "sobrina nieta Leonor Villaflor". Plaintiff
Leonor Villaflor instituted the present action against the administrator of the estate of the widow Fausta
Nepomuceno, on February 8, 1958, contending that upon the widow's death, said plaintiff became
vested with the ownership of the real and personal properties bequeathed by the late Nicolas Villaflor to
the clause 7 of his will, pursuant to its eight (8th) clause. Defendant's position, adopted by the trial
court, is that the title to the properties, aforesaid became absolutely vested in the widow upon her
death, on account, of the fact that she never remarried. ISSUE: Whether or not Court of First Instance,
erred in dismissing the complaint of petitioner.

HELD: Yes, the lower court erred in dismissing the complaint of the herein plaintiff in construing and
adopting the argument of the herein defendant that said properties becomes an absolute property of
the widow upon her death, because, petitioner as a reversionary legatee, could succeed only if the
widow has remarried. The court discarded the expression “mientras viva” and considered the words”uso
y posesion” as equivalent to “dominio” (ownership). In so doing, the trial court violatged Article 791 of
the Civil code of the Philippines as well as section 59 of Rule 123 of the Rules of Court.. ART. 791. The
words of a will are to receive an interpretation which will give to every expression some effect, rather
than one which will render any of the expressions inoperative; and of two modes of interpreting a will,
that one is to be preferred which will prevent intestacy." . SEC. 59. Instrument construed so as to give
effect to all provisions. — In the construction of an instrument where there are several provisions or
particulars, such a construction is, if possible, to be adopted as will give effect to all." . The Civil Code, in
Article 790, p. 1 (Article 675 of the Code of 1889), expressly enjoins the following: . ART. 790. The words
of a will are to be taken in their ordinary and grammatical sense, unless a clear intention to use them in
another sense can be gathered, and that other can be ascertained." . Technical words in a will are to be
taken in their technical sense, unless the context clearly indicates a contrary intention, or unless it
satisfactorily appears that the will was drawn solely by the testator, and that he was unacquainted with
such technical sense. (675a) PREMISES CONSIDERED, the decision appealed from is reversed, and the
appellant Leonor Villaflor Vda. de VILLANUEVA is declared entitled to the ownership and fruits of the
properties described in clause 7 of the will or testament, from the date of the death of Doña Fausta
Nepomuceno. The records are ordered remanded to the court of origin for liquidation, accounting and
further proceedings conformably to this decision. Costs against the Administrator-appellee.

VICENTE URIARTE v. COURT OF FIRST INSTANCE OF NEGROS


OCCIDENTAL, GR No. L-21938-39, 1970-05-29

Facts:

It appears that on November 6, 1961 petitioner filed with the Negros


Court a petition for the settlement of the estate of the late Don Juan
Uriarte y Goite (Special Proceeding No. 6344) alleging therein, inter alia,
that,... as a natural son of the latter, he was his sole heir, and that, during
the life time of said decedent, petitioner had instituted Civil Case No.
6142 in the same Court for his compulsory acknowledgment as such
natural son.

Upon petitioner's motion the Negros Court appointed the Philippine


National Bank as special administrator on November 13, 1961 and two
days later it set the date for the hearing of the petition and ordered that
the requisite notices be... published in accordance with law. The record
discloses, however, that, for one reason or another, the Philippine
National Bank never actually qualified as special administrator.

On December 19, 1961, Higinio Uriarte, one of the two private


respondents herein, filed an opposition to the above-mentioned petition
alleging that he was a nephew of the deceased Juan Uriarte y Goite who
had "executed a Last Will and Testament in Spain, a duly... authenticated
copy whereof has been requested and which shall be submitted to this
Honorable Court upon receipt thereof", and further questioning
petitioner's capacity and interest to commence the intestate proceeding.

On August 28, 1962, Juan Uriarte Zamacona, the other private


respondent, commenced Special Proceeding No. 51396 in the Manila
Court for the probate of a document alleged to be the last will of the
deceased Juan Uriarte... y Goite, and on the same date he filed in Special
Proceeding No. 6344 of the Negros Court a motion to dismiss the same on
the following grounds: (1) that, as the deceased Juan Uriarte y

Goite had left a last will, there was no legal basis to proceed with said
intestate proceedings, and (2) that petitioner Vicente Uriarte had no legal
personality and interest to initiate said intestate proceedings, he not
being an... acknowledged natural son of the decedent.

On April 19, 1963, the Negros Court sustained Juan Uriarte Zamacona's
motion to dismiss and dismissed the Special Proceeding No. 6344
pending before it. His motion for reconsideration of said order having
been denied on July 27, 1963

Issues:

(a) whether or not the Negros Court erred in dismissing Special


Proceeding No. 6344, on the one hand, and on the other, (b) whether the
Manila Court similarly erred in... not dismissing Special Proceeding No.
51396 notwithstanding proof of the prior filing of Special Proceeding No.
6344 in the Negros Court.

Ruling:
**Ruling:**

in accordance with settled jurisprudence in this jurisdiction, testate proceedings


for the settlement of the estate of a deceased person take precedence over... in-
testate proceedings for the same purpose. Thus it has been held repeatedly that, if
in the course of intestate proceedings pending before a court of first instance it is
found that the decedent had left a last will, proceedings for the probate of the...
latter should replace the intestate proceedings even if at that stage an
administrator had already been appointed, the latter being required to render final
account and turn over the estate in his possession to the executor subsequently...
appointed. This, however, is understood to be without prejudice that should the
alleged last will be rejected or is disapproved, the proceeding shall continue as an
intestacy. As already adverted to, this is a clear indication that... proceedings
for the probate of a will enjoy priority over intestate proceedings.

The following considerations and the facts of record would seem to support the view
that he should have submitted said will for probate to the Negros Court, either in a
separate special proceeding or in an appropriate motion for said purpose filed in the
already pending

Special Proceeding No. 6344. In the first place, it is not in accord with public
policy and the orderly and inexpensive administration of justice to unnecessarily
multiply litigation, especially if several courts would be... involved. This, in
effect, was the result of the submission of the will aforesaid to the Manila Court.
In the second place, when respondent Higinio Uriarte filed an opposition to Vicente
Uriarte's petition for the issuance of... letters of administration, he had already
informed the Negros Court that the deceased Juan Uriarte y Goite had left a will in
Spain, of which a copy had been requested for submission to said court; and when the
other respondent, Juan Uriarte Zamacona, filed his motion to... dismiss Special
Proceeding No. 6344, he had submitted to the Negros Court a copy of the alleged will
of the decedent, from which fact it may be inferred that, like Higinio Uriarte, he
knew before filing the petition for probate with the Manila Court that there was
already a... special proceeding pending in the Negros Court for the settlement of the
estate of the same deceased person. As far as Higinio Uriarte is concerned, it seems
quite clear that in his opposition to petitioner's petition in Special Proceeding No.

6344, he had expressly promised to submit said will for probate to the Negros Court.

We can not accept petitioner's contention in this regard that... the latter court had
no jurisdiction to consider said petition, albeit we say that it was not the proper
venue therefor.

It is well settled in this jurisdiction that wrong venue is merely a waiveable


procedural defect, and, in the light of the circumstances obtaining in the instant
case, we are of the opinion, and so hold, that petitioner has... waived the right to
raise such objection or is precluded from doing so by laches.

All these notwithstanding, it was only on April 15, 1963 that he filed with the
Manila Court in Special Proceeding No. 51396 an Omnibus... motion asking for leave to
intervene and for the dismissal and annulment of all the proceedings had therein up
to that date; thus enabling the Manila Court not only to appoint an administrator
with the will annexed but also to admit said will to probate more than five months...
earlier, or more specifically, on October 31, 1962.

To allow him now to assail the exercise of jurisdiction over the probate of the will
by the Manila Court and the validity of all the proceedings had in Special Proceeding
No. 51396 would put a... premium on his negligence. Moreover, it must be remembered
that this Court is not inclined to annul proceedings regularly had in a lower court
even if the latter was not the proper venue therefor, if the net result would be to
have the... same proceedings repeated in some other court of similar jurisdiction;
more so in a case like the present where the objection against said proceedings is
raised too late.

```important```

If the petitioner is to be consistent with the authorities cited by him in support of


his contention,... the proper thing for him to do would be to intervene in the
certain estate proceedings entitled Special Proceedings No. 51396 in the Court of
First Instance of Manila instead of maintaining an independent action, for indeed his
supposed... interest in the estate of the decedent is of his doubtful character
pending the final decision of the action for compulsory acknowledgment".

```Important```

We believe in connection with the above matter that petitioner is entitled to


prosecute Civil Case No. 6142 until it is finally determined, or intervene in Special
Proceeding No. 51396 of the Manila Court, if it is still open, or to ask for its
reopening if it... has already been closed, so as to be able to submit for
determination the question of his acknowledgment as a natural child of the deceased
testator, said court having, in its capacity as a probate court, jurisdiction to
declare who are the heirs of the... deceased testator and whether or not a particular
party is or should be declared his acknowledged natural child (II Moran on Rules of
Court, 1957 Ed. p. 476; Conde vs. Abaya, 13 Phil. 249, Severino vs. Severino, 44
Phil. 343; Lopez vs. Lopez, 68 Phil. 227, and Jimoga-on vs.

Belmonte, 47 O. G. 1119).
IN VIEW OF THE FOREGOING CONSIDERATIONS, judgment is hereby rendered denying the
writs prayed for and, as a result, the petition for certiorari filed in G. R. No. L-
21938, as well as the supplemental petition for mandamus docketed as G. R.

No. L-21939, are hereby dismissed. The writ of preliminary injunction heretofore
issued is set aside. With costs against petitioner.

**Principles:**

Courts of First Instance have original exclusive jurisdiction over "all matters of
probate", that is, over special proceedings for the settlement of the estate of
deceased... persons - whether they died testate or intestate. While their
jurisdiction over such subject matter is beyond question, the matter of venue, or the
particular Court of First Instance where the special proceeding Should be commenced,
is... regulated by former Rule 75, Section 1 of the Rules of Court, now Section 1,
Rule 73 of the Revised Rules of Court, which provides that the estate of a decedent
inhabitant of the Philippines at the time of his death, whether a citizen or... an
alien, shall be in the court of first instance in the province in which he resided at
the time of his death, and if he is an inhabitant of a foreign country the court of
first instance of any province... in which he had estate.

It can not be denied that a special proceeding intended to effect the distribution of
the estate of a deceased person; whether in accordance with the law on intestate
succession or in accordance with his will, is a "probate matter" or a proceeding...
for the settlement of his estate. It is equally true, however, that in accordance
with settled jurisprudence in this jurisdiction, testate proceedings for the
settlement of the estate of a deceased person take precedence over... intestate
proceedings for the same purpose. Thus it has been held repeatedly that, if in the
course of intestate proceedings pending before a court of first instance it is found
that the decedent had left a last will, proceedings for the probate of the... latter
should replace the intestate proceedings even if at that stage an administrator had
already been appointed, the latter being required to render final account and turn
over the estate in his possession to the executor subsequently... appointed. This,
however, is understood to be without prejudice that should the alleged last will be
rejected or is disapproved, the proceeding shall continue as an intestacy. As
already adverted to, this is a clear indication that... proceedings for the probate
of a will enjoy priority over intestate proceedings.

Blas vs. Santos

March 20, 2016

Case Digest
G.R. No. L-14070 March 29, 1961

MARIA GERVACIO BLAS, MANUEL GERVACIO BLAS, LEONCIO GERVACIO


BLAS and LOIDA GERVACIO BLAS, Plaintiffs-Appellants,

vs.

ROSALINA SANTOS, in her capacity as Special Administratrix of the Estate of the


deceased MAXIMA SANTOS VDA. DE BLAS, in Sp. Proc. No. 2524, Court of First
Instance of Rizal, defendants-appellants. MARTA GERVACIO BLAS and DR. JOSE
CHIVI, Defendants-Appellants.

LABRADOR, J.:

FACTS: This action was instituted by plaintiffs against the administration of the estate of
Maxima Santos, to secure a judicial declaration that one-half of the properties left by
Maxima Santos Vda. de Blas, the greater bulk of which are set forth and described in
the project of partition presented in the proceedings for the administration of the estate
of the deceased Simeon Blas, had been promised by the deceased Maxima Santos to
be delivered upon her death and in her will to the plaintiffs, and requesting that the said
properties so promised be adjudicated to the plaintiffs. The complaint also prays for
actual damages in the amount of P50,000. The alleged promise of the deceased
Maxima Santos is contained in a document executed by Maxima Santos on December
26, 1936 attached to the complaint as Annex “H” and introduced at the trial as Exhibit
“A”. The complaint also alleges that the plaintiffs are entitled to inherit certain properties
enumerated in paragraph 3 thereof, situated in Malabon, Rizal and Obando, Bulacan,
but which properties have already been included in the inventory of the estate of the
deceased Simeon Blas and evidently partitioned and conveyed to his heirs in the
proceedings for the administration of his estate. Spouses Simeon Blas and Marta Cruz
have three children they also have grandchildren. One year after Marta Cruz died, Blas
married Maxima Santos but they don’t have children and the properties that he and his
former wife acquired during the first marriage were not liquidated. Simeon Blas
executed a will disposing half of his properties in favor of Maxima the other half for
payment of debts, Blas also named a few devisees and legatees therein. In lieu of this,
Maxima executed a document whereby she intimated that she understands the will of
her husband; that she promises that she’ll be giving, upon her death, one-half of the
properties she’ll be acquiring to the heirs and legatees named in the will of his husband;
that she can select or choose any of them depending upon the respect, service, and
treatment accorded to her by said heirs. On 1937 Simeon Blas died while Maxima died
on 1956 and Rosalina Santos became administrator of her estate. In the same year,
Maria Gervacio Blas, child of Simeon Blas in his first marriage, together with three other
grandchildren of Simeon Blas (heirs of Simeon Blas), learned that Maxima did not fulfill
her promise as it was learned that Maxima only disposed not even one-tenth of the
properties she acquired from Simeon Blas. The heirs are now contending that they did
not partition Simeon Blas’ property precisely because Maxima promised that they’ll be
receiving properties upon her death.

ISSUE: Whether or not the heirs can acquire the properties that Maxima promised with
them.

HELD: Yes, they can acquire the properties that Maxima promised with them because it
was stated in Art. 1347 that “No contract may be entered into upon future inheritance
except in cases expressly authorized by law.”. In this case the contract was authorized
by law because the promised made by Maxima to their heirs before she died is a valid
reason and it should be enforceable upon her death and her heirs can now acquire the
succession of the properties in issue.

LETICIA VALMONTE ORTEGA v. JOSEFINA C. VALMONTE, GR NO. 157451,


2005-12-16

Facts:

Petition for Review[1] under Rule 45 of the Rules of Court, seeking to


reverse and set aside the December 12, 2002 Decision[2] and the March
7, 2003 Resolution[3] of the Court of Appeals

Decision appealed from is REVERSED and SET ASIDE.

rendered approving and allowing probate to the said last will and
testament of Placido Valmonte

Placido toiled and lived for a long time in the United States until he finally
reached retirement. In 1980, Placido finally came home to stay in the
Philippines,... Two years after his arrival from the United States and at
the age of 80 he wed Josefina who was then 28 years old, in a ceremony
solemnized by Judge

Perfecto Laguio, Jr. on February 5, 1982. But in a little more than two
years of wedded bliss, Placido died on October 8, 1984 of a cause written
down as COR PULMONALE.

"Placido executed a notarial last will and testament written in English


and consisting of two (2) pages

"The allowance to probate of this will was opposed by Leticia on the


grounds that:
Petitioner failed to allege all assets of the testator, especially those
found in the USA;

Petitioner failed to state the names, ages, and residences of the heirs of
the testator; or to give them proper notice pursuant to law;

Will was not executed and attested as required by law and legal
solemnities and formalities were not complied with;

Testator was mentally incapable to make a will at the time of the alleged
execution he being in an advance sate of senility;

Will was executed under duress, or the influence of fear or threats;

Will was procured by undue and improper influence and pressure on the
part of the petitioner and/or her agents and/or assistants; and/or

Signature of testator was procured by fraud, or trick, and he did not


intend that the instrument should be his will at the time of affixing his
signature thereto;"

Josefina said she had no... knowledge of the existence of the last will and
testament of her husband, but just serendipitously found it in his attache
case after his death. It was only then that she learned that the testator
bequeathed to her his properties and she was named the executrix in the
said will.

"Notary Public Floro Sarmiento, the notary public who notarized the
testator's will, testified that it was in the first week of June 1983 when
the testator together with the three witnesses of the will went to his
house cum law office and requested him to prepare his last will... and
testament. After the testator instructed him on the terms and dispositions
he wanted on the will, the notary public told them to come back

He likewise explained that though it appears that the will was signed by
the testator and his witnesses on June 15, 1983, the day when it should
have been executed... had he not gone out of town, the formal execution
was actually on August 9, 1983. He reasoned that he no longer changed
the typewritten date of June 15, 1983 because he did not like the
document to appear dirty. The notary public also testified that to his
observation the... testator was physically and mentally capable at the
time he affixed his signature on the will.

"The attesting witnesses to the will corroborated the testimony of the


notary public,... "It then found these grounds extant and proven, and
accordingly disallowed probate."[... appellate court admitted the will of
Placido Valmonte to probate. The CA upheld the credibility of the notary
public and the subscribing witnesses who had acknowledged the due
execution of the will. Moreover, it held that the testator had...
testamentary capacity at the time of the execution of the will. It added
that his "sexual exhibitionism and unhygienic, crude and impolite
ways"[6] did not make him a person of unsound mind.

Issues:

Whether or not the signature of Placido Valmonte in the subject will was
procured by fraud or trickery, and that Placido Valmonte never intended
that the instrument should be his last will and testament.

In short, petitioner assails the CA's allowance of the probate of the will of
Placido Valmonte.

Ruling:

The Petition has no merit.

We stress that the party challenging the will bears the burden of proving
the existence of fraud at the time of its execution.[14] The burden to
show otherwise shifts to the proponent of the will only upon a showing of
credible evidence of fraud.[15] Unfortunately in this case, other than the
self-serving allegations of petitioner, no evidence of fraud was ever
presented.

It is a settled doctrine that the omission of some relatives does not affect
the due execution of a will.[16] That the testator was tricked into signing
it was not sufficiently established by the fact that he had instituted his
wife, who was more than fifty... years his junior, as the sole beneficiary;
and disregarded petitioner and her family, who were the ones who had
taken "the cudgels of taking care of [the testator] in his twilight years."[...
petitioner failed to substantiate her claim of a "grand conspiracy" in the
commission of a fraud. There was no showing that the witnesses of the
proponent stood to receive any benefit from the allowance of the will. The
testimonies of the three subscribing witnesses and... the notary are
credible evidence of its due execution.

the Petition is DENIED, and the assailed Decision and Resolution of the
Court of Appeals are AFFIRMED. Costs against petitioner.

Principles:

Fraud "is a trick, secret device, false statement, or pretense, by which


the subject of it is cheated. It may be of such character that the testator
is misled or deceived as to the nature or contents of the document which
he executes, or it may relate to... some extrinsic fact, in consequence of
the deception regarding which the testator is led to make a certain will
which, but for the fraud, he would not have made."[... the conflict
between the dates appearing on the will does not invalidate the
document, "because the law does not even require that a [notarial] will x
x x be executed and acknowledged on the same occasion."

(Allowance/Disallowance of Wills) Case Citation: Ortega v. Valmonte | G.R. No. 157451 Date: Dec 16,
2005 Petitioners: Leticia Valmonte Ortega Respondents: Josefina C. Valmonte Doctrine: According to
Article 799, the three things that the testator must have the ability to know to be considered of sound
mind are as follows: (1) the nature of the estate to be disposed of, (2) the proper objects of the
testator's bounty, and (3) the character of the testamentary act. Antecedent Facts: Placido toiled and
lived for a long time in the United States until he finally reached retirement. In 1980, Placido finally came
home to stay in the Philippines. Two years after his arrival from the United States and at the age of 80
he wed Josefina who was then 28 years old, on February 5, 1982. But in a little more than two years of
wedded bliss, Placido died on October 8, 1984, of a cause written down as COR PULMONALE. "Placido
executed a notarial last will and testament written in English and consisting of two (2) pages, and dated
June 15, 1983 but acknowledged only on August 9, 1983. The allowance to probate of the will was
opposed by Leticia. Respondent’s contention: The oppositor Leticia declared that Josefina should not
inherit alone because aside from her there are other children from the siblings of Placido who are just as
entitled to inherit from him. She attacked the mental capacity of the testator, declaring that at the time
of the execution of the notarial will the testator was already 83 years old and was no longer of sound
mind. She knew whereof she spoke because in 1983 Placido lived in the Makati residence and asked
Leticia's family to live with him and they took care of him. During that time, the testator's physical and
mental condition showed deterioration, aberrations and senility. This was corroborated by her daughter
Mary Jane Ortega for whom Placido took a fancy and wanted to marry. RTC Ruling: The lower court
disallowed the will. CA Ruling: Reversing the trial court, the appellate court admitted the will of Placido
Valmonte to probate. The CA upheld the credibility of the notary public and the subscribing witnesses
who had acknowledged the due execution of the will. Moreover, it held that the testator had
testamentary capacity at the time of the execution of the will. It added that his "sexual exhibitionism
and unhygienic, crude and impolite ways" did not make him a person of unsound mind. Issues:  WON
the signature of Placido Valmonte in the subject will was procured by fraud or trickery (NO)  WON
Valmonte had testamentary capacity at the time he executed the will (YES) SC Ruling: Article 839 of the
Civil Code states the instances when a will may be disallowed, as follows: (1) If the formalities required
by law have not been complied with; (2) If the testator was insane, or otherwise mentally incapable of
making a will, at the time of its execution; (3) If it was executed through force or under duress, or the
influence of fear, or threats; (4) If it was procured by undue and improper pressure and influence, on the
part of the beneficiary or of some other person; (5) If the signature of the testator was procured by
fraud; (6) If the testator acted by mistake or did not intend that the instrument he signed should be his
will at the time of affixing his signature thereto. Existence of Fraud in the Execution of a Will Petitioner
contends that it was "highly dubious for a woman at the prime of her young life [to] almost immediately
plunge into marriage with a man who [was] thrice her age x x x and who happened to be [a] Fil-
American pensionado," thus casting doubt on the intention of respondent in seeking the probate of the
will. Moreover, it supposedly "defies human reason, logic and common experience" for an old man with
a severe psychological condition to have willingly signed a last will and testament. We are not
convinced. Fraud "is a trick, secret device, false statement, or pretense, by which the subject of it is
cheated. It may be of such character that the testator is misled or deceived as to the nature or contents
of the document which he executes, or it may relate to some extrinsic fact, in consequence of the
deception regarding which the testator is led to make a certain will which, but for the fraud, he would
not have made." Unfortunately, in this case, other than the self-serving allegations of petitioner, no
evidence of fraud was ever presented. It is a settled doctrine that the omission of some relatives does
not affect the due execution of a will. Capacity to make a will According to Article 799, the three things
that the testator must have the ability to know to be considered of sound mind are as follows: (1) the
nature of the estate to be disposed of, (2) the proper objects of the testator's bounty, and (3) the
character of the testamentary act. Applying this test to the present case, we find that the appellate
court was correct in holding that Placido had testamentary capacity at the time of the execution of his
will. It must be noted that despite his advanced age, he was still able to identify accurately the kinds of
property he owned, the extent of his shares in them and even their locations. As regards the proper
objects of his bounty, it was sufficient that he identified his wife as sole beneficiary. As we have stated
earlier, the omission of some relatives from the will did not affect its formal validity. There being no
showing of fraud in its execution, intent in its disposition becomes irrelevant.

G.R. No. L-4132 March 23, 1908 In the matter of the will of MARIA SIASON Y MADRID DE LEDESMA,
Probate proceedings. Vs. Antonio Jayme for petitioner. TRACEY, J.: THE CASE Special proceedings for the
legalization of a will, the Court of First Instance refused probate on the ground that the instrument was
not subscribed by the witnesses in the presence of the testatrix and of each other as required by section
618 of the Code of Civil Procedure. FACTS: > The testatrix was ill and confined to her house, the
execution of the will taking place in the sala where she lay upon a sofa.
ISSUES RAISED: 1. As to whether the testatrix from where she lay could read what was written at the
table and the first witness, after signing, went away from the table. 2. the signature to the instrument is
defective. At the request of Señora Maria Siason. CATALINO GEVA. T. SILVERIO. FRUCTUOSO G. MORIN.
RAFAEL ESPINOS.
HELD : > the will must be signed by the testator, or by the testator's name written by some other person
in his presence, and by his express direction," The question presented in this case is, Are the words
"Señora Maria Siason" her name written by some other person? it is contended that they form a part of
the recital and not a signature, the only signature being the names of the witnesses themselves. In
Guison vs. Concepcion (5 Phil. Rep., 551) it was held that there was no signature, although the
attestation clause which followed the will contained the name of the testatrix and was thereafter signed
by the witnesses. The majority of the court, however, are of the opinion that the distinction is a tenable
one inasmuch as in the Concepcion will the name of the testatrix occurred only in the body of the
attestation clause, after the first signatures of the witnesses, whereas in this will it immediately follows
the testament itself and precedes the names of the witnesses. WHEREFORE: The decision of the court
below is reversed, without costs, and that court is directed to admit the instrument before it to probate
as the last will of the testatrix. -- CARSON, J., dissenting: > a will should not be probated unless there has
been strict compliance in its execution with all the "requisites of a will" as prescribed in section 618 of
the Code of Civil Procedure. > This section provides that the will must be signed by the testator or that it
must be signed by the testator's name written by some other person. > I think it is quite clear that the
will in question was signed by the name of the witness written at the request of the testatrix, but that it
was not signed by the name of the testatrix written by the witness. > The name of the testatrix appeared
in the attestation clause at the end of the will in the case of Guison vs. Concepcion (5 Phil. Rep., 551),
and it was placed there for precisely the same purpose as it appears in the will under consideration; that
is, in a recital of fact that the testatrix had requested some one to attach her signature to the will. We
declared in that case, that the will could not be probated because the name of the testatrix was not
subscribed to the will in accordance with the provisions of section 618 of the Code of Civil Procedure. > I
recognize that in the case under consideration a holding that the name of the testatrix is not signed to
the will in the manner prescribed by law would appear to defeat the intent of the testatrix, and to
invalidate the instrument for a failure to comply with a mere technical formality. But the same reasons
of public policy which dictated the provisions of law prescribing certain requisites without which no will
is valid,

CASEDIGEST:NOBLEV.ABAJA(450SCRA265) Published by paul on June 25, 2013 | Leave a response Noble


v. Abaja 450 SCRA 265 FACTS: The case is about the probate of the will of Alipio Abada (Not respondent
Abaja). Petitioner Belinda Noble is the administratrix of the estate of Abada. Respondent Alipio Abaja
filed a petition for the probate of Abada’s will. Petitioner Noble moved for dismissal of the petition for
probate. Caponong-Noble points out that nowhere in the will can one discern that Abada knew the
Spanish language. She alleges that such defect is fatal and must result in the disallowance of the will.
ISSUE: Should it be expressly stated in the will that it (the will) was in a language known by the testator?
HELD: No. There is no statutory requirement to state in the will itself that the testator knew the
language or dialect used in the will.[25] This is a matter that a party may establish by proof aliunde. In
this case, Alipio testified that Abada used to gather Spanish-speaking people in their place. In these
gatherings, Abada and his companions would talk in the Spanish language. This sufficiently proves that
Abada speaks the Spanish language.

In re will of Ana Abangan (Case Digest)

G.R. No. L-13431 November 12, 1919

In re will of Ana Abangan.


GERTRUDIS ABANGAN, executrix-appellee,
vs.
ANASTACIA ABANGAN, ET AL., opponents-appellants.

FACTS:
Ana Abangan’s will was admitted to probate. Opponents appealed. Ana’s will consisted of two
sheets. The first contained all the dispositions duly signed at the bottom bottom by Martin
Montalban (in the name and under the direction of the testatrix) and by three witnesses. The
following sheet contains only the attestation clause duly signed at the bottom by the three
instrumental witnesses. Neither of these sheets is signed on the left margin by the testatrix and
the three witnesses, nor numbered by letters; and these omissions, according to appellants’
contention, are defects whereby the probate of the will should have been denied.

ISSUES:

1. WON the probate of the will should have been denied because neither of the two sheets
were signed on the left margin by the testatrix and the three witnesses, nor numbered by
letters.

2. WON the testatrix knew the language in which the will was written.

RULING:

1.) NO. The will was duly admitted to probate. According to the SC:

ATTESTATION. — In a will consisting of two sheets the first of which contains all the
testamentary dispositions and is signed at the bottom by the testator and three witnesses and
the second contains only the attestation clause and is signed also at the bottom by the three
witnesses, it is not necessary that both sheets be further signed on their margins by the testator
and the witnesses, or be paged.

TESTATOR’S SIGNATURE. — The testator’s signature is not necessary in the attestation clause
because this, as its name implies, appertains only to the witnesses.

The SC emphasized that the object of the solemnities surrounding the execution of wills is
to close the door against bad faith and fraud, to avoid substitution of wills and testaments and
to guaranty their truth and authenticity. Therefore the laws on this subject should be
interpreted in such a way as to attain these primordial ends. But, on the other hand, also one
must not lose sight of the fact that it is not the object of the law to restrain and curtail the
exercise of the right to make a will. So when an interpretation already given assures such ends,
any other interpretation whatsoever, that adds nothing but demands more requisites entirely
unnecessary, useless and frustrative of the testator’s last will, must be disregarded.

2.) Yes. The testatrix knew the language in which the will was written.

DIALECT IN WHICH WRITTEN; PRESUMPTION. — The circumstance appearing in the will itself
that same was executed in the city of Cebu and in the dialect of this locality where the testatrix
was a neighbor is enough, in the absence of any proof to the contrary, to presume that she
knew this dialect in which her will is written.

The judgment appealed from was affirmed.

source
start here
Testate Estate of Cagro v. Cagro Digest
Testate Estate of Cagro vs. Cagro

G.R. L-5826

Facts:

1. The case is an appeal interposed by the oppositors from a decision of the CFI of Samar which
admitted to probate a will allegedly executed by Vicente Cagro who died in Pambujan, Samar on
Feb. 14, 1949.

2. The appellants insisted that the will is defective because the attestation was not signed by the
witnesses at the bottom although the page containing the same was signed by the witnesses on the
left hand margin.

3. Petitioner contended that the signatures of the 3 witnesses on the left hand margin conform
substantially to law and may be deemed as their signatures to the attestation clause.
Issue: Whether or not the will is valid

HELD: Will is not valid. The attestation clause is a memorandum of the facts attending the execution
of the will. It is required by law to be made by the attesting witnesses and it must necessarily bear
their signatures.

An unsigned attestation clause cannot be considered as an act of the witnesses since the omission
of their signatures at the bottom negatives their participation.

Moreover, the signatures affixed on the let hand margin is not substantial conformance to the law.
The said signatures were merely in conformance with the requirement that the will must be signed
on the left-hand margin of all its pages. If the attestation clause is unsigned by the 3 witnesses at the
bottom, it would be easier to add clauses to a will on a subsequent occasion and in the absence of
the testator and any or all of the witnesses.

The probate of the will is denied.

Testate estate of the late VICENTE CAGRO. JESUSA CAGRO vs. PELAGIO CAGRO

G.R. L-5826

Facts:

1. The case is an appeal intervened by the Cagro from a decision of the Court of First Instance
of Samar which admitted to check the authenticity of the will left behind, so to prove that the will
executed by Vicente Cagro to be valid who died in Samar. (1949)

It is a probate proceeding.

2. PELAGIO CAGRO and company insisted that the will is defective because the attestation
(evidence or proof ) was not signed by the witnesses at the bottom although the page
containing the attestation was signed by the witnesses on the left hand margin.

3. JESUSA CAGRO said that the signatures of the 3 witnesses on the left hand margin is valid
and may be considered as their signatures to the attestation clause (A formal attestation
clause itself can serve as prima facie evidence of the facts within the instrument.)

Issue: Whether or not the will is valid


So the court rules that the Will is not valid. An attestation clause is a provision at the end of an
instrument, for example in a will, that is signed by witnesses and it contains the formalities
required to make the will effective.

An unsigned attestation clause cannot be considered as an act of the witnesses since the
absence of their signatures at the bottom means they did not participate.

So the signatures affixed on the left hand margin is not enough to prove the validity of the
will. The said signatures were merely in accordance with the requirement that the will must
be signed on the left-hand margin of all its pages. If the attestation clause is no signed by the
3 witnesses at the bottom, it would be easier to add clauses to a will in the absence of the
testator and all of the witnesses.

So because of this The probate of the will is denied by the court.not valid.

NENITA DE VERA SUROZA, complainant, v. JUDGE REYNALDO P. HONRADO of the Court of First
Instance of Rizal, Pasig Branch 25 and EVANGELINE S. YUIPCO, Deputy Clerk of Court, respondents. A.M.
No. 2026-CFI, 19 December 1981 AQUINO, J.:

FACTS: Mauro Suroza, a corporal in U.S. Army, married Marcelina Salvador in 1923. They were childless.
They reared a boy named Agapito who used the surname Suroza and who considered them as his
parents as shown in his 1945 marriage contract with Nenita de Vera. Mauro died in 1942. Marcelina, as
a veteran's widow, became a pensioner of the Federal Government. That explains why on her death she
had accumulated some cash in two banks. Agapito and Nenita begot a child named Lilia who became a
medical technologist and went abroad. Agapito also became a soldier. He was disabled and his wife
Nenita was appointed as his guardian in 1953 when he was declared an incompetent by the Court of
First Instance. On a date not indicated in the record, the spouses Antonio Sy and Hermogena Talan
begot a child named Marilyn Sy, who, when a few days old, was entrusted to Arsenia de la Cruz
(apparently a girl friend of Agapito) and who was later delivered to Marcelina Salvador Suroza who
brought her up as a supposed daughter of Agapito and as her granddaughter. Marilyn used the surname
Suroza. She stayed with Marcelina but was not legally adopted by Agapito. She married Oscar Medrano.
Marcelina supposedly executed a notarial will in Manila on July 23, 1973, when she was 73 years old.
That will which is in English was thumbmarked by her. She was illiterate. Her letters in English to the
Veterans Administration were also thumbmarked by her. In that will, Marcelina bequeathed all her
estate to her supposed granddaughter Marilyn. ISSUE: Whether or not a notarial will, which on its face is
void because it is written in English, a language not known to the illiterate testatrix, and which is
probably a forged will because she and the attesting witnesses did not appear before the notary as
admitted by the notary himself, be admitted for probate. RULING: No. In this case, respondent judge, on
perusing the will and noting that it was written in English and was thumbmarked by an obviously
illiterate testatrix, could have readily perceived that the will is void. In the opening paragraph of the will,
it was stated that English was a language "understood and known" to the testatrix. But in its concluding
paragraph, it was stated that the will was read to the testatrix "and translated into Filipino language".
That could only mean that the will was written in a language not known to the illiterate testatrix and,
therefore, it is void because of the mandatory provision of article 804 of the Civil Code that every will
must be executed in a language or dialect known to the testator. The hasty preparation of the will is
shown in the attestation clause and notarial acknowledgment where Marcelina Salvador Suroza is
repeatedly referred to as the "testator" instead of "testatrix". Had respondent judge been careful and
observant, he could have noted not only the anomaly as to the language of the will but also that there
was something wrong in instituting the supposed granddaughter as sole heiress and giving nothing at all
to her supposed father who was still alive. Furthermore, after the hearing conducted by respondent
deputy clerk of court, respondent judge could have noticed that the notary was not presented as a
witness. In spite of the absence of an opposition, respondent judge should have personally conducted
the hearing on the probate of the will so that he could have ascertained whether the will was validly
executed.

NENITA DE VERA SUROZA, complainant, v. JUDGE REYNALDO P. HONRADO of the Court of First
Instance of Rizal, Pasig Branch 25 and EVANGELINE S. YUIPCO, Deputy Clerk of Court, respondents.
A.M. No. 2026-CFI, 19 December 1981 AQUINO, J.:

FACTS: Mauro Suroza is a corporal in U.S. Army. His wife is Marcelina Salvador. They do not have
children but they adopted Agapito Suroza and who considered them as his parents as shown in his
marriage contract with Nenita de Vera. When Mauro died, Marcelina became a pensioner of the
Federal Government since That explains why on her death she had accumulated some cash in two
banks. Agapito and Nenita begot a child named Lilia who became a medical technologist and went
abroad. Agapito also became a soldier. He was disabled and his wife Nenita was appointed as his
guardian when he was declared an incompetent by the Court. The spouses Antonio Sy and Hermogena
Talan had a child named Marilyn Sy , she was an infant, few days old only, was entrusted to Arsenia
de la Cruz (she is the girl friend of Agapito) and who was later delivered to Marcelina Salvador Suroza
who brought her up as a supposed daughter of Agapito and as her granddaughter. Marilyn used the
surname Suroza. She stayed with Marcelina but was not legally adopted by Agapito. She married
Oscar Medrano. Marcelina supposedly executed a notarial will in Manila on July 23, 1973, when she
was 73 years old. That will which is in English was thumbmarked by her. She was illiterate. Her letters
in English to the Veterans Administration were also thumbmarked by her. In that will, Marcelina
bequeathed all her estate to her supposed granddaughter Marilyn. ISSUE: Whether or not a notarial
will, which on its face is void because it is written in English, a language not known to the illiterate
testatrix, and which is probably a forged will because she and the attesting witnesses did not appear
before the notary as admitted by the notary himself, be admitted for probate. RULING: No. In this
case, respondent judge, on perusing the will and noting that it was written in English and was
thumbmarked by an obviously illiterate testatrix, could have readily perceived that the will is void. In
the opening paragraph of the will, it was stated that English was a language "understood and known"
to the testatrix. But in its concluding paragraph, it was stated that the will was read to the testatrix
"and translated into Filipino language". That could only mean that the will was written in a language
not known to the illiterate testatrix and, therefore, it is void because of the mandatory provision of
article 804 of the Civil Code that every will must be executed in a language or dialect known to the
testator. The hasty preparation of the will is shown in the attestation clause and notarial
acknowledgment where Marcelina Salvador Suroza is repeatedly referred to as the "testator" instead
of "testatrix". Had respondent judge been careful and observant, he could have noted not only the
anomaly as to the language of the will but also that there was something wrong in instituting the
supposed granddaughter as sole heiress and giving nothing at all to her supposed father who was still
alive. Furthermore, after the hearing conducted by respondent deputy clerk of court, respondent
judge could have noticed that the notary was not presented as a witness. In spite of the absence of an
opposition, respondent judge should have personally conducted the hearing on the probate of the will
so that he could have ascertained whether the will was validly executed.

GREGORIO D. CANEDA v. CA, GR No. 81322, 1990-02-05

Facts:

It appears on record that sometime on November 8, 1977, Buenaventura


Gueson for value received, executed a promissory note for the sum of
P18,960.00 in favor of Gregorio Caneda, Jr. promising to pay a monthly
installment of P790.00 for 24 months with 14%... to secure the obligation
Gueson executed a chattel mortgage and used a Toyota Jiffy jeep as a
collateral; that it is expressly stipulated in the promissory note and
chattel mortgage that default in the payment of any installment will make
the entire obligation due and... demandable.

promissory note and chattel mortgage was assigned by Gregorio Caneda


in favor of Investors Finance Corporation (FNCB). Defendant Gueson
defaulted in his obligation and as of September 24, 1980 had an
outstanding balance of P11,230.00 exclusive of interest and... other
charges. Despite repeated demands defendant Gueson allegedly failed
and refused to pay the entire obligation. Hence, FNCB on December, 1980
filed a complaint for replevin and/or sum of money against Buenaventura
Gueson and John Doe. As relief, FNCB prayed for the seizure... of the
Toyota Jiffy jeep and its delivery to it, the payment of 25% of the total
amount due as attorney's fees plus 10% thereof as liquidated damages
and costs. In the alternative FNCB also prayed for the payment of the sum
of P11,230,00 with interest at 15% per annum to be... computed from
September 25, 1978 until fully paid

January 2, 1981, Buenaventura Gueson filed his answer with third party
complaint. In his answer Gueson interposed the defense that he did not
receive any value for the promissory note he executed as he merely
accommodated the real debtor Gregorio Caneda

March 18, 1981, Gregorio D. Caneda, Jr. filed his answer to the third party
complaint. He denied that he is the real debtor or the party
accommodated. He alleged that he had not incurred any monetary
obligation in favor of FNCB. He pointed out that Gueson agreed to buy his

Jiffy jeep, but since he has no cash, they agreed that Gueson will apply
for financing with FNCB; that he executed a deed of sale on the condition
that if the financing will not be approved the sale shall not materialize
and Gueson shall deed back the jeep to him

June 18, 1981, the pre-trial conference was terminated as no settlement


could be reached by the parties (Rollo, p. 134). Trial ensued thereafter

February 19, 1982 hearing, the John Doe named on the complaint was
identified as Gregorio Caneda, Jr. Upon FNCB's motion the third party
complaint was treated as a cross-claim and the pleading filed by Caneda,
Jr. was considered as an answer to the complaint and... cross-claim

Because of Caneda's failure to attend the hearing, who instead filed an


ex-parte motion for postponement despite the previous warning of the
court that the October 22, 1982 hearing could not be moved as the
previous scheduled hearing for the reception of Caneda,... Jr.'s evidence
was postponed at his instance, the trial court in its order dated October
22, 1982 declared that Caneda waived his right to present evidence and
the case would be decided on the evidence on record. Caneda filed a
motion for reconsideration, but it was denied in... the order of the trial
court dated November 22, 1982.

incident was elevated to the Court of Appeals. But for lack of merit the
petition for certiorari and prohibition filed by Caneda, Jr. was dismissed
on March 15, 1983 by the Appellate Court

November 26, 1983, the trial court rendered its decision[*] on the main
case, finding that Buenaventura Gueson was merely an accommodation
party for the benefit of Caneda,... Finally, said defendant/cross-defendant
Gregorio D. Caneda, Jr., is hereby ordered to pay the costs of the suit.

IT IS SO ORDERED
June 18, 1987, Caneda Jr. filed with the Court of Appeals a motion to
cancel entry of judgment alleging that the appellate court's decision is
not yet final and executory as he has not received a copy of the said
decision. In its resolution dated July 23, 1987, the Court... of Appeals
denied said motion.

Hence, this petition.

Petitioner claimed among others that the Court of Appeals arbitrarily


denied his motion to cancel entry of judgment, despite the fact that on
June 3, 1987, he learned for the first time that a decision dated November
28, 1986 was rendered by the Court of Appeals because he... was not
furnished a copy of said decision... which was delivered instead by letter
carrier Anastacio Arbizo of the Post Office of Davao City on December 11,
1986 at about 12:10 noon to a certain Boy Reyes, petitioner's neighbor,
living just in front of his office

Petitioner argued that Boy Reyes is not authorized to receive his mails so
that the negligence of Reyes is not binding on him.

he claimed that the Court of Appeals' denial of his motion to cancel entry
of judgment is tantamount to a denial of... his fundamental right to due
process of law... respondents pointed out that previous summons and
other pleadings were duly served in petitioner's office, they were not able
to show that the copy of the decision in question was properly served...
respondents resorted to suppositions and surmises claiming that it is
unthinkable that Boy Reyes, petitioner's neighbor, "living just right in
front of the office" which office also serve as petitioner's residence would
not deliver the mail

Issues:

whether or not a copy of the November 28, 1986 decision of the Court of
Appeals has been properly served on herein petitioner and therefore has
become final and executory.

Ruling:

service was made at an address which was neither the "residence" nor
the "dwelling house" of the petitioner nor his office or regular place of
business at the time of service and served on a person who is not the...
proper person to whom the papers should be left, the same is not the
service contemplated by the Rules.

The principal question that arises is the effect of the assignment on the
obligations of Gueson and Caneda to FNCB.
As between Gueson and Caneda, it is obvious that whatever private
agreement or understanding transpired between them is binding on them
alone and not on FNCB whose only concern in the whole transaction is
the repayment of the loan it has extended.

ppeal of petitioner is hereby ALLOWED, but considering this case on the


merits, the assailed decision of the Court of Appeals of November 28,
1986 making Caneda, Jr. liable to FNCB, is hereby AFFIRMED

Magsaysay-Labrador, et. al v. CA
September 8, 2021 legalassassins 0 Comments case digest, commercial law, corporation law

Share this post!

0
Shares

CONCEPCION MAGSAYSAY-LABRADOR, SOLEDAD MAGSAYSAY-CABRERA,


LUISA MAGSAYSAY-CORPUZ, assisted be her husband, Dr. Jose Corpuz, FELICIDAD
P. MAGSAYSAY, and MERCEDES MAGSAYSAY-DIAZ, petitioners,
vs.
THE COURT OF APPEALS and ADELAIDA RODRIGUEZ-MAGSAYSAY, Special
Administratrix of the Estate of the late Genaro F. Magsaysay respondents.
G.R. No. 58168
19 December 1989

Facts:
On 9 February 1979, Adelaida Rodriguez-Magsaysay, widow and special administratix of the
estate of the late Senator Genaro Magsaysay, brought before the then Court of First Instance of
Olongapo an action against Artemio Panganiban, Subic Land Corporation (SUBIC), Filipinas
Manufacturer’s Bank (FILMANBANK) and the Register of Deeds of Zambales, for the
annulment of the Deed of Assignment executed by the late Senator in favor of SUBIC (as a
result of which TCT 3258 was cancelled and TCT 22431 issued in the name of SUBIC), for the
annulment of the Deed of Mortgage executed by SUBIC in favor of FILMANBANK (dated 28
April 1977 in the amount of P 2,700,000.00), and cancellation of TCT 22431 by the Register of
Deeds, and for the latter to issue a new title in her favor. On 7 March 1979, Concepcion
Magsaysay-Labrador, Soledad Magsaysay-Cabrera, Luisa Magsaysay-Corpuz, Felicidad
Magsaysay, and Mercedes Magsaysay-Diaz, sisters of the late senator, filed a motion for
intervention on the ground that on 20 June 1978, their brother conveyed to them 1/2 of his
shareholdings in SUBIC or a total of 416,566.6 shares and as assignees of around 41 % of the
total outstanding shares of such stocks of SUBIC, they have a substantial and legal interest in the
subject matter of litigation and that they have a legal interest in the success of the suit with
respect to SUBIC. On 26 July 1979, the trial court denied the motion for intervention, and ruled
that petitioners have no legal interest whatsoever in the matter in litigation and their being
alleged assignees or transferees of certain shares in SUBIC cannot legally entitle them to
intervene because SUBIC has a personality separate and distinct from its stockholders.On appeal,
the Court of Appeals found no factual or legal justification to disturb the findings of the lower
court. The appellate court further stated that whatever claims the Magsaysay sisters have against
the late Senator or against SUBIC for that matter can be ventilated in a separate proceeding. The
motion for reconsideration of the Magsaysay sisters was denied. Hence, the petition for review
on certiorari.

Issue:
Whether the Magsaysay sister, allegedly stockholders of SUBIC, are interested parties in a case
where corporate properties are in dispute.

Held:
Viewed in the light of Section 2, Rule 12 of the Revised Rules of Court, the Magsaysay sisters
have no legal interest in the subject matter in litigation so as to entitle them to intervene in the
proceedings. To be permitted to intervene in a pending action, the party must have a legal
interest in the matter in litigation, or in the success of either of the parties or an interest against
both, or he must be so situated as to be adversely affected by a distribution or other disposition of
the property in the custody of the court or an officer thereof . Here, the interest, if it exists at all,
of the Magsaysay sisters is indirect, contingent, remote, conjectural, consequential and collateral.
At the very least, their interest is purely inchoate, or in sheer expectancy of a right in the
management of the corporation and to share in the profits thereof and in the properties and assets
thereof on dissolution, after payment of the corporate debts and obligations. While a share of
stock represents a proportionate or aliquot interest in the property of the corporation, it does not
vest the owner thereof with any legal right or title to any of the property, his interest in the
corporate property being equitable or beneficial in nature. Shareholders are in no legal sense the
owners of corporate property, which is owned by the corporation as a distinct legal person.

Codoy vs. Calugay Case Digest G.R. No. 123486 August 12,
1999 312 SCRA 333
Codoy vs. Calugay Case Digest
G.R. No. 123486 ; August 12, 1999 ; 312 SCRA 333
PRINCIPLE/S:

a) Art. 811 of the Civil Code is MANDATORY if the HOLOGRAPHIC will is CONTESTED

Reason: The possibility of a false document being adjudged as the Will of the testator cannot be
eliminated

b) Azaola vs. Singson case vis-a-vis Codoy vs. Calugay case

1. Azaola case - will was not contested.

Effect: Article 811 if permissive

- Witnesses may not be needed for probate of the holographic will

2. Codoy case - will was contested (ground: forgery)

Effect: Article 811 if mandatory

- At least 3 witnesses needed for probate of the holographic will

Holographic will is NOT contested = Art. 811 permissive (Azaola case)

Holographic will is contested = Art. 811 mandatory (Codoy case)

FACTS: Respondent Calugay et al., devisees and legatees of the holographic will of the
deceased Matilde, filed a petition for probate of the said will with the RTC. They attested to the
genuineness and due execution of the will. Petitioner Codoy et al. filed an opposition claiming
that the will was a forgery and illegible. Calugay presented 6 witnesses and various
documentary evidence. Petitioners instead of presenting their evidence, filed a demurrer to
evidence. RTC granted the Demurrer to evidence and denied the petition for probate of
respondents. CA reversed the RTC’s ruling citing the decision in the case of Azaola
vs. Singson which essentially states that witnesses are not needed in probate of holographic
wills.

ISSUE/S: WON Article 811 of the Civil Code is mandatory or directory for probate of a
holographic will.
HELD: Mandatory if the holographic will is contested. Article 811 of the Civil Code is mandatory
since the word “shall” connotes a mandatory order. We have ruled that "shall" in a statute
commonly denotes an imperative obligation and is inconsistent with the idea of discretion and
that the presumption is that the word "shall," when used in a statute is mandatory.

The reason is that we cannot eliminate the possibility of a false document being adjudged as the
will of the testator. Which is why if the holographic will is contested, the law requires three
witnesses to declare that the will was in the handwriting of the deceased.

Rodelas v. Aranza Digest


Rodelas v. Aranza

G.R. No. L-58509 December 7, 1982

Relova, J. (Ponente)

Facts:

1. The appellant filed a petition for the probate of the holographic will of Ricardo Bonilla in 1977. The
petition was opposed by the appellees on the ground that the deceased did not leave any will,
holographic or otherwise.

2. The lower court dismissed the petition for probate and held that since the original will was lost, a
photostatic copy cannot stand in the place of the original.

Issue: Whether or not a holographic will can be proved by means of a photocopy

RULING: Yes. A photocopy of the lost or destroyed holographic will may be admitted because the
authenticity of the handwriting of the deceased can be determined by the probate court with the
standard writings of the testator.
Roxas vs. De Jesus Jr
Kalaw v. Relova Digest
Kalaw v. Relova

G.R. No. L-40207 September 28, 1984

Melencio-Herrera, J. (Ponente)

Facts:

1. Gregorio Kalaw, the private respondent, claiming to be the sole heir of sister Natividad, filed a
peition for probate of the latter's holographic will in 1968. The will contained 2 alterations: a) Rosa's
name, designated as the sole heir was crossed out and instead "Rosario" was written above it. Such
was not initialed, b) Rosa's name was crossed out as sole executrix and Gregorio's ma,e was written
above it. This alteration was initialed by the testator.
2. Rosa contended that the will as first written should be given effect so that she would be the sole
heir. The lower court denied the probate due to the unauthenticated alterations and additions.

Issue: Whether or not the will is valid

RULING: No, the will is voided or revoked since nothing remains in the will which could remain valid
as there was only one disposition in it. Such was altered by the substitution of the original heir with
another. To rule that the first will should be given effect is to disregard the testatrix' change of mind.
However, this change of mind cannot be given effect either as she failed to authenticate it in
accordance with Art. 814, or by affixing her full signature.

Ajero vs. CA

G.R. No. 106720 September 15, 1994

SPOUSES ROBERTO AND THELMA AJERO, petitioners,


vs.
THE COURT OF APPEALS AND CLEMENTE SAND, respondents.

FACTS:

Annie Sand named as devisees petitioners Roberto and Thelma Ajero, private respondent
Clemente Sand, Meriam S. Arong, Leah Sand, Lilia Sand, Edgar Sand, Fe Sand, Lisa S. Sand, and
Dr. Jose Ajero, Sr., and their children.
Petition for allowance of decedent's holographic will was opposed by Private respondent on the
grounds that: neither the testament's body nor the signature therein was in decedent's
handwriting; it contained alterations and corrections which were not duly signed by decedent;
and, the will was procured by petitioners through improper pressure and undue influence. The
petition was likewise opposed by Dr. Jose Ajero. He contested the disposition in the will of a
house and lot located in Cabadbaran, Agusan Del Norte. He claimed that said property could not
be conveyed by decedent in its entirety, as she was not its sole owner.

The trial court admitted the decedent's holographic will to probate.

On appeal, the petition for probate of decedent's will was dismissed. The Court of Appeals
found that, "the holographic will failed to meet the requirements for its validity." It held that
the decedent did not comply with Articles 813 and 814 of the New Civil Code, which read, as
follows:

Art. 813: When a number of dispositions appearing in a holographic will are


signed without being dated, and the last disposition has a signature and date,
such date validates the dispositions preceding it, whatever be the time of prior
dispositions.

Art. 814: In case of insertion, cancellation, erasure or alteration in a holographic


will, the testator must authenticate the same by his full signature.

ISSUE:

Whether the holographic will should be admitted to probate.

RULING:
Yes. The holographic will should be admitted to probate.

The SC held that the lists for disallowing wills are exclusive and that no other grounds can serve
to disallow a will.

It cited the following:

Section 9, Rule 76 of the Rules of Court provides that will shall be disallowed in any of the
following cases:

(a) If not executed and attested as required by law;

(b) If the testator was insane, or otherwise mentally incapable to make a will, at
the time of its execution;

(c) If it was executed under duress, or the influence of fear, or threats;

(d) If it was procured by undue and improper pressure and influence, on the part
of the beneficiary, or of some other person for his benefit;

(e) If the signature of the testator was procured by fraud or trick, and he did not
intend that the instrument should be his will at the time of fixing his signature
thereto.

Article 839 of the New Civil Code:

Art. 839: The will shall be disallowed in any of the following cases;

(1) If the formalities required by law have not been complied with;

(2) If the testator was insane, or otherwise mentally incapable of


making a will, at the time of its execution;

(3) If it was executed through force or under duress, or the


influence of fear, or threats;

(4) If it was procured by undue and improper pressure and


influence, on the part of the beneficiary or of some other person;
(5) If the signature of the testator was procured by fraud;

(6) If the testator acted by mistake or did not intend that the
instrument he signed should be his will at the time of affixing his
signature thereto.

In the case at bench, the SC said that the holographic will complied with the requirements of
Articles 813 and 814 of the Civil Code and reiterated its ruling in Abangan vs. Abangan:

The object of the solemnities surrounding the execution of wills is to close the
door against bad faith and fraud, to avoid substitution of wills and testaments
and to guaranty their truth and authenticity. Therefore, the laws on this subject
should be interpreted in such a way as to attain these primordial ends. But, on
the other hand, also one must not lose sight of the fact that it is not the object of
the law to restrain and curtail the exercise of the right to make a will. So when
an interpretation already given assures such ends, any other interpretation
whatsoever, that adds nothing but demands more requisites entirely
unnecessary, useless and frustrative of the testator's last will, must be
disregarded.

For purposes of probating non-holographic wills, these formal solemnities include the
subscription, attestation, and acknowledgment requirements under Articles 805 and 806 of the
New Civil Code.

In the case of holographic wills, on the other hand, what assures authenticity is the requirement
that they be totally autographic or handwritten by the testator himself, as provided under
Article 810 of the New Civil Code, thus:

A person may execute a holographic will which must be entirely written, dated,
and signed by the hand of the testator himself. It is subject to no other form, and
may be made in or out of the Philippines, and need not be witnessed.
A reading of Article 813 of the New Civil Code shows that its requirement affects the validity of
the dispositions contained in the holographic will, but not its probate. If the testator fails to
sign and date some of the dispositions, the result is that these dispositions cannot be
effectuated. Such failure, however, does not render the whole testament void.

Likewise, a holographic will can still be admitted to probate, notwithstanding non-compliance


with the provisions of Article 814. In the case of Kalaw vs. Relova the Supreme Court held:

Ordinarily, when a number of erasures, corrections, and interlineations made by


the testator in a holographic Will have not been noted under his signature, . . .
the Will is not thereby invalidated as a whole, but at most only as respects the
particular words erased, corrected or interlined.

Thus, unless the unauthenticated alterations, cancellations or insertions were made on the date
of the holographic will or on testator's signature, their presence does not invalidate the will
itself. The lack of authentication will only result in disallowance of such changes.

This separation and distinction adds support to the interpretation that only the requirements of
Article 810 of the New Civil Code — and not those found in Articles 813 and 814 of the same
Code — are essential to the probate of a holographic will.

The SC affirmed the ruling of the CA that decedent Annie Sand could not validly dispose of the
house and lot located in Cabadbaran, Agusan del Norte, in its entirety. The testatrix cannot
validly dispose of the whole property, which she shares with her father's other heirs.

The holographic will of decedent Annie Sand was admitted to probate.


In Re: Estate of Johnson G.R. No. 12767. November 16,
1918
Facts: On February 4, 1916, Emil H. Johnson, a native of Sweden and a naturalized citizen of the United
States, died in the city of Manila. He left a will disposing an estate with an estimated amount of P231,800.
The will was written in the testator’s own handwriting, and is signed by himself and two witnesses only,
instead of three witnesses required by section 618 of the Code of Civil Procedure. This will, therefore, was
not executed in conformity with the provisions of law generally applicable to wills executed by inhabitants
of these Islands, and hence could not have been proved under section 618. On February 9, 1916, however,
a petition was presented in the Court of First Instance of the city of Manila for the probate of this will, on
the ground that 1) Johnson was, at the time of his death, a citizen of the State of Illinois, United States of
America; 2) that the will was duly executed in accordance with the laws of that State; and hence could
properly be probated here pursuant to section 636 of the Code of Civil Procedure. Petitioner alleged that
the law is inapplicable to his father’s will

Issue: Whether or not there was deprivation of due process on the part of the petition

Held: No.

Ratio: Due publication was made pursuant to this order of the court through the three-week publication of
the notice in Manila Daily Bulletin. The Supreme Court also asserted that in view of the statute concerned
which reads as “A will made within the Philippine Islands by a citizen or subject of another state or
country, which is executed in accordance with the law of the state or country of which he is a citizen or
subject, and which might be proved and allowed by the law of his own state or country, may be proved,
allowed, and recorded in the Philippine Islands, and shall have the same effect as if executed according to
the laws of these Islands” the “state”, being not capitalized, does not mean that United States is excluded
from the phrase (because during this time, Philippines was still a territory of the US).

Matias v. Salud Digest


Matias vs. Salud

G.R. L-10907 June 29, 1957


Ponente: Concepcion, J.

Facts:

1. This case is an appeal from a CFI Cavite order denying the probate of the will of Gabina Raquel.
The document consist of 3 pages and it seems that after the attestation clause, there appears the
siganture of the testatrix 'Gabina Raquel', alongside is a smudged in violet ink claimed by the
proponents as the thumbmark allegedly affixed by the tetratrix. On the third page at the end of the
attestation clause appears signatures on the left margin of each page, and also on the upper part of
each left margin appears the same violet ink smudge accompanied by the written words 'Gabina
Raquel' with 'by Lourdes Samonte' underneath it.

2. The proponent's evidence is to the effect that the decedent allegedly instructed Atty. Agbunag to
drat her will and brought to her on January 1950. With all the witnesses with her and the lawyer, the
decedent affixed her thumbmark at the foot of the document and the left margin of each page. It was
also alleged that she attempted to sign using a sign pen but was only able to do so on the lower half
of page 2 due to the pain in her right shoulder. The lawyer, seeing Gabina unable to proceed
instructed Lourdes Samonte to write 'Gabina Raquel by Lourdes Samonte' next to each thumbmark,
after which the witnesses signed at the foot of the attestation clause and the left hand margin of
each page.

3. The probate was opposed by Basilia Salud, the niece of the decedent.

4. The CFI of cavite denied the probate on the ground that the attestation clause did not state that
the testatrix and the witnesses signed each and every page nor did it express that Lourdes was
specially directed to sign after the testatrix.

Issue: Whether or not the thumbprint was sufficient compliance with the law despite the
absence of a description of such in the attestation clause

HELD: YES

The absence of the description on the attestation clause that another person wrote the testatrix'
name at her request is not a fatal defect, The legal requirement only ask that it be signed by the
testator, a requirement satisfied by a thumbprint or other mark affixed by him.
As to the issue on the clarity of the ridge impression, it is held to be dependent on the aleatory
circumstances. Where a testator employs an unfamiliar way of signing and that both the attestation
clause and the will are silent on the matter, such silence is a factor to be considered against the
authenticity of the testament. However, the failure to describe the signature itself alone is not
sufficient to refuse probate when evidence fully satisfied that the will was executed and witnessed in
accordance with law.

In the Matter of the will of ANTERO MERCADO, deceased. ROSARIO GARCIA, petitioner, vs. JULIANA
LACUESTA, ET AL., respondents 90 Phil 489 November 29, 1951 Facts: Antero Mercado left a will dated
January 3, 1943. The will is written in the Ilocano dialect which is spoken and understood by the
testator. The will also contained an attestation clause which is signed by three witnesses. The attestation
clause states: “We, the undersigned, by these presents to declare that the foregoing testament of
Antero Mercado was signed by himself and also by us below his name and of this attestation clause and
that of the left margin of the three pages thereof. Page three the continuation of this attestation clause;
this will is written in Ilocano dialect which is spoken and understood by the testator, and it bears the
corresponding number in letter which compose of three pages and all them were signed in the presence
of the testator and witnesses, and the witnesses in the presence of the testator and all and each and
every one of us witnesses.” The will appears to have been signed by Atty. Florentino Javier who wrote
the name of Antero Mercado, followed below by "A ruego del testador" and the name of Florentino
Javier. Antero Mercado is alleged also to have written a cross immediately after his name. Issue:
Whether or not the attestation clause in the will is valid. Ruling: No. The attestation clause is fatally
defective for failing to state that Antero Mercado caused Atty. Florentino Javier to write the testator's
name under his express direction, as required by section 618 of the Code of Civil Procedure. When the
testator expressly caused another to sign the former’s name, this fact must be recited in the attestation
clause. Otherwise, the will is fatally defective. Moreover, the cross appearing on the will is not the usual
signature of Antero Mercado nor is it even one of the ways by which he signed his name. After mature
reflection, the Court is not prepared to liken the mere sign of the cross to a thumbmark, and the reason
is obvious. The cross cannot and does not have the trustworthiness of a thumbmark. Thus, the cross
cannot be considered a valid signature.

Gonzales v. Court of Appeals (CA) Digest


Gonzales v. CA

G.R. No. L-37453 May 25, 1979


Guerrero, J. (Ponente)
Facts:

1. Petitioner Rizalina Gonzales and Lutgarda Santiago (Private respondent) are the nieces of the
deceased Isabel Gabriel who died a widow. A will was thereafter submitted to probate. The said will
was typewritten, in Tagalog and appeared to have been executed in April 1961 or two months prior
to the death of Isabel. It consisted of 5 pages including the attestation and acknowledgment, with the
signature of testatrix on page 4 and the left margin of all the pages.

2. Lutgarda was named as the universal heir and executor. The petitioner opposed the probate.

3. The lower court denied the probate on the ground that the will was not executed and attested in
accordance with law on the issue of the competency and credibility of the witnesses.

Issue: Whether or not the credibility of the subscribing witnesses is material to the validity of a will

RULING: No. The law requires only that witnesses posses the qualifications under Art. 820 (NCC)
and none of the disqualifications of Art. 802. There is no requirement that they are of good standing
or reputation in the community, for trustworthiness, honesty and uprightness in order that his
testimony is believed and accepted in court. For the testimony to be credible, it is not mandatory that
evidence be established on record that the witnesses have good standing in the the community.
Competency is distinguished from credibility, the former being determined by Art. 820 while the latter
does not require evidence of such good standing. Credibility depends on the convincing weight of his
testimony in court.
Gonzales v. Court of Appeals (CA) Digest
Gonzales v. CA

G.R. No. L-37453 May 25, 1979


Guerrero, J. (Ponente)

Facts:

The typewritten will of Isabel Gabriel who died as a widow, it was written in Tagalog and was
to have been executed two months prior to the death of Isabel . The said will was probated.
The will has 5 pages including the attestation and acknowledgment,and also the signature of
testatrix on page 4 and the left margin of all the pages.

There is this certain Lutgarda, Lutgarda Santiago (Private respondent) a niece of the deceased
Isabel Gabriel, she was chosen as the universal heir and executor. The petitioner opposed the
probate.

3. The honorable (lower) court denied the probate. The honorable court said that the will was
not executed and attested according to the law to show the credibility of the witnesses.

Issue: Whether or not the credibility of the subscribing witnesses is material to the validity of
a will
The Honorable court ruled that: No. the credibility of the subscribing witnesses is not
material to the validity of a will. According to the law, only that witnesses posses the
qualifications under Art. 820 (NCC) and none of the disqualifications of Art. 802. It is not
stated on the law that is a requirement that they must be in good reputation in the community
for the honorable court that in order that his testimony will be accepted in court. It is not
mandatory that evidence be established on record, for the testimony to be credible,

Competency is distinguished from credibility, the former being determined by Art. 820 while
the latter does not require evidence of such good standing. Credibility depends on the
convincing weight of his testimony in court.

Icasiano v. Icasiano Digest


Icasiano vs. Icasiano
G.R. No. L-18979 June 30, 1964

Facts:
1. Celso Icasiano, filed a petition for the probate of the will of Josefa Villacorte and for his
appointment as executor thereof. It appears from the evidence that the testatrix died on September
12, 1958. She executed a will in Tagalog, and through the help of her lawyer, it was prepared in
duplicates, an original and a carbon copy.

2. On the day that it was subscribed and attested, the lawyer only brought the original copy of the
will while the carbon duplicate (unsigned) was left in Bulacan. One of the witnesses failed to sign
one of the pages in the original copy but admitted he may have lifted 2 pages simultaneously instead
when he signed the will. Nevertheless, he affirmed that the will was signed by the testator and other
witnesses in his presence.

Issue: Whether or not the failure of one of the subscribing witnesses to affix his signature to
a page is sufficient to deny probate of the will

RULING: No, the failure to sign was entirely through pure oversight or mere inadvertence. Since the
duplicated bore the required signatures, this proves that the omission was not intentional. Even if the
original is in existence, a duplicate may still be admitted to probate since the original is deemed to be
defective, then in law, there is no other will bu the duly signed carbon duplicate and the same can be
probated.

The law should not be strictly and literally interpreted as to penalize the testatrix on account of the
inadvertence of a single witness over whose conduct she has no control of. Where the purpose of
the law is to guarantee the identity of the testament and its component pages, and there is no
intentional or deliberate deviation existed.

Note that this ruling should not be taken as a departure from the rules that the will should be signed
by the witnesses on every page. The carbon copy duplicate was regular in all respects.

Case Digest: Cruz vs. Villasor, G.R. No. L-32213,


November 16, 1973
October 05, 2023

Art. 806 | Succession, Notary Public, Witness

Provision:

Art. 806. Every will must be acknowledged before a notary public by the
testator and the witnesses. The notary public shall not be required to
retain a copy of the will or file another with the office of the Clerk of
Court.

Ponente:
Esguerra, J.,
Petitioner: Agapita Cruz

Respondents: Hon. Judge Guillermo P. Villasor and Manuel Lugay

Recit Version:

Agapita N. Cruz, the surviving spouse of the deceased Valente Z. Cruz,


opposed the probate of his will. Of the three witnesses, one of them, the
notary public Angel H. Teves, Jr., was also the one before whom the will
was supposed to have been acknowledged.

The court held that the notary public could not act as both the attesting
and acknowledging witness since he cannot acknowledge before himself
having signed the will. Allowing the notary public to be the third witness
would mean only two witnesses appeared before him, contrary to the
legal requirement of having at least three credible witnesses.

Facts:

Valente Z. Cruz left a last will and testament before he died.

Manuel B. Lugay is the supposed executor of the will.

Agapita N. Cruz, the surviving spouse of the deceased, opposed the


probate. She claimed the will was executed through fraud, deceit,
misrepresentation, and undue influence. She argued that Valente was not
fully informed of the content of the will, especially regarding the
properties he was disposing of, and that the will wasn't executed in
accordance with the law.

Of the three instrumental witnesses thereto, namely Deogracias T.


Jamaloas Jr., Dr. Francisco Pañares and Atty. Angel H. Teves, Jr., one of
them, the last named, is at the same time the Notary Public before whom
the will was supposed to have been acknowledged.

Petitioner argues that the result is that only two witnesses appeared
before the notary public to acknowledge the will.

CFI-Cebu: Admitted the will to probate.

Issue:

WoN the supposed last will and testament of Valente Z. Cruz was
executed in accordance with law. NO

Held:

Private respondent-appellee, Manuel B. Lugay, following the reasoning of


the trial court, maintains that there is substantial compliance with the
legal requirement of having at least three attesting witnesses even if the
notary public acted as one of them, bolstering up his stand
with American Jurisprudence, p. 227 which, insofar as pertinent, reads
as follows:
It is said that there are, practical reasons for upholding a will as against
the purely technical reason that one of the witnesses required by law
signed as certifying to an acknowledgment of the testator's signature
under oath rather than as attesting the execution of the instrument.

After weighing the merits of the conflicting claims of the parties, We are
inclined to sustain that of the appellant that the last will and testament in
question was not executed in accordance with law. The notary public
before whom the will was acknowledged cannot be considered as the
third instrumental witness since he cannot acknowledge before himself
his having signed the will. To acknowledge before means to avow; to
own as genuine, to assent, to admit; and "before" means in front or
preceding in space or ahead of.

Consequently, if the third witness were the notary public himself, he


would have to avow assent, or admit his having signed the will in front of
himself. This cannot be done because he cannot split his personality into
two so that one will appear before the other to acknowledge his
participation in the making of the will. To permit such a situation to
obtain would be sanctioning a sheer absurdity.

Furthermore, the function of a notary public is, among others, to guard


against any illegal or immoral arrangement. That function would defeated
if the notary public were one of the attesting instrumental witnesses. For
them he would be interested sustaining the validity of the will as it
directly involves him and the validity of his own act. It would place him in
inconsistent position and the very purpose of acknowledgment, which is
to minimize fraud, would be thwarted.
Admittedly, there are American precedents holding that notary public
may, in addition, act as a witness to the executive of the document he
has notarized. There are others holding that his signing merely as notary
in a will nonetheless makes him a witness thereon. But these authorities
do not serve the purpose of the law in this jurisdiction or are not decisive
of the issue herein because the notaries public and witnesses referred to
aforecited cases merely acted as instrumental, subscribing attesting
witnesses, and not as acknowledging witnesses. He the notary public
acted not only as attesting witness but also acknowledging witness, a
situation not envisaged by Article 806 of the Civil Code which reads:

ART. 806. Every will must be acknowledged before a notary public by the
testator and the witnesses. The notary public shall not be required to
retain a copy of the will or file another with the office of the Clerk of
Court.

To allow the notary public to act as third witness, or one the attesting
and acknowledging witnesses, would have the effect of having only two
attesting witnesses to the will which would be in contravention of the
provisions of Article 805 be requiring at least three credible witnesses to
act as such and of Article 806 which requires that the testator and the
required number of witnesses must appear before the notary public to
acknowledge the will. The result would be, as has been said, that only
two witnesses appeared before the notary public for or that purpose. In
the circumstances, the law would not be duly in observed.

FOR ALL THE FOREGOING, the judgment appealed from is hereby


reversed and the probate of the last will and testament of Valente Z. Cruz
is declared not valid and hereby set aside.
Javellana v. Ledesma Digest
Javellana vs. Ledesma

G.R. No. L-7179

Facts:

1. The CFI of Iloilo admitted to probate a will and codicil executed by the deceased Apolinaria
Ledesma in July 1953. This testament was deemed executed on May 1950 and May 1952. The
contestant was the sister and nearest surviving relative of the deceased. She appealed from this
decision alleging that the will were not executed in accordance with law.

2. The testament was executed at the house of the testatrix. One the other hand, the codicil was
executed after the enactment of the New Civil Code (NCC), and therefore had to be acknowledged
before a notary public. Now, the contestant, who happens to be one of the instrumental witnesses
asserted that after the codicil was signed and attested at the San Pablo hospital, that Gimotea (the
notary) signed and sealed it on the same occasion. Gimotea, however, said that he did not do so,
and that the act of signing and sealing was done afterwards.

2. One of the allegations was that the certificate of acknowledgement to the codicil was signed
somewhere else or in the office of the notary. The ix and the witnesses at the hospital, was signed
and sealed by the notary only when he brought it in his office.

Issue: Whether or not the signing and sealing of the will or codicil in the absence of the
testator and witnesses affects the validity of the will

RULING: NO. Unlike in the Old Civil Code of 1899, the NCC does not require that the signing of the
testator, the witnesses and the notary be accomplished in one single act. All that is required is that
every will must be acknowledged before a notary public by the testator and witnesses. The
subsequent signing and sealing is not part of the acknowledgement itself nor of the testamentary
act. Their separate execution out of the presence of the testator and the witnesses cannot be a
violation of the rule that testaments should be completed without interruption.
Lee v. Tambago
Digest
Home / Legal Profession /

LEGAL PROFESSION

BynicanorOctober 11, 2021

Nature of the Case: This is a complaint against Atty. Regino Tambago, charging
him with violation of the Notarial Law and the ethics of the legal profession for
notarizing a spurious last will and testament.

Facts

Complainant averred that his father, the decedent Vicente Lee, Sr., never
executed the contested will. The spurious will also contained the forged
signatures of the purported witnesses to its execution. The will was purportedly
executed and acknowledged before the respondent on June 30, 1965. However,
the resident certificate of the testator noted in the acknowledgment of the will was
dated January 5, 1962. Furthermore, the signature of the testator did not match
his signature in a deed of donation. Complainant also questioned the absence of
notation of the residence certificates of the witnesses, and that their signatures
have also been forged. Lastly, he asserted that no copy of the will was on file in
the archives division of the National Commission for Culture and the Arts
(NCCA).

In his defense, the respondent alleged that the complainant was not a son of the
decedent, and that the last will and testament was validly executed and actually
notarized by respondent. He claimed that no copy of the contested will could be
found in the archives because none was filed. He also contended that there was
no cause of action since the complainant did not first file an action for the
declaration of nullity of the will and demand his share in the inheritance, and that
the complainant was just harassing him because he lost in the criminal case he
filed against the respondent.

The investigating commissioner of the IBP found the respondent guilty of


violating the old Notarial Law and Rule 1.01 of the Code of Professional
Responsibility, and recommended the suspension of the respondent from the
practice of law for 3 months. The IBP Board of Governors adopted and approved
it but modified the penalty to 1 year of suspension and 2 years of revocation from
reappointment as Notary Public.

Issue

Whether the respondent Atty. Tambago is guilty of professional misconduct, thus


warranting his suspension from the practice of law.

Held

Yes. The respondent Atty. Tambago is found guilty of professional misconduct.


He violated the Lawyer’s Oath; Rule 138 of the Rules of Court; Canon 1 and Rule
1.01 of the Code of Professional Responsibility; Art. 806 of the Civil Code and (5)
the provisions of the old Notarial Law. As such, he is SUSPENDED from the
practice of law for 1 year, his notarial commission is revoked, and is perpetually
disqualified from reappointment as a notary public.

Reasoning

Firstly, the will is considered void because it being attested by only two witnesses
did not comply with the Notarial law which requires the attestation of three or
more credible witnesses in the presence of the testator and of one another.

Secondly, the requirement on the acknowledgment of the will was neither strictly
nor substantially complied with because of the absence of a notation of the
residence certificates of the notarial witnesses in the acknowledgment. Likewise,
by having allowed decedent to exhibit an expired residence certificate,
respondent failed to comply with the requirements of both the old Notarial Law
and the Residence Tax Act.

Thirdly, the respondent failed to make the necessary entries pertaining to the will
in his notarial register as required by the old Notarial Law. The photocopies of the
respondent’s notarial register and the certification which the respondent used to
prove his compliance with this requirement was inadmissible as evidence of the
entry of the execution of the will because it failed to comply with the rules on the
admissibility of secondary evidence, which required him to first prove the
existence and cause of the unavailability of the original document.

The Court emphasized that as a notary public, the respondent was bound to
strictly observe these elementary requirements as they are mandatory and
cannot be disregarded, considering the degree of importance and evidentiary
weight attached to notarized documents and that, as in this case, the testator and
and the witnesses are no longer alive to identify the instrument. Otherwise, the
confidence of the public in the integrity of notarized deeds will be undermined. As
a consequence of the respondent’s breach of duty, the validity of the will was
seriously compromised.

The practice of law is a privilege burdened with conditions. The first and foremost
duty of a lawyer is to obey the laws of the land. For a lawyer is the servant of the
law and belongs to a profession to which society has entrusted the administration
of law and the dispensation of justice. Hence, the respondent’s gross violations
of the law also made him liable for violation of his oath as a lawyer and Canon 1
and Rule 1.01 of the Code of Professional Responsibility.

IN RE WILL OF JOSEFA ZALAMEA Y ABELLA v. ANTONIO ABELLA ET AL.,


GR No. 17857, 1922-06-12

Facts:

On July 19, 1918, Doña Josefa Zalamea y Abella, single, 60 years old, who
was residing in the municipality of Pagsanjan, Province of Laguna,
executed her last will and testament with an attached inventory of her
properties, Exhibits A and A-1, in the... presence of three witnesses, who
signed with her all the pages of said documents.

The testatrix died on the 6th of January, 1921, and, as the record shows,
the executor appointed in the will, Pedro Unson, filed in the Court of First
Instance of Laguna on the 19th of January of... the same year an
application for the probate of the will and the issuance of the proper
letters of administration in his favor.

To said application an opposition was presented by Antonio Abella...


lleging that the supposed will of the deceased Zalamea was not executed
in conformity with the provisions of the law,... it was not paged...
correlatively in letters,... nor was there any attestation clause in it, nor
was it signed by the testatrix and the witnesses in the presence of each
other.

Trial having been held,... a quo overruled the opposition of the


contestants, and ordered the probate of the will,... the contestants have
appealed, and in their brief they assign three errors, which, in their
opinion, justify the reversal of the judgment appealed from.

The first error assigned by the appellants as committed by the court...


said to be the will of the deceased Josefa Zalamea, was executed with all
the solemnities required by the law.

he arguments advanced by appellants' counsel in support of the first


assignment of error tend to impeach the credibility of the witnesses for
the proponent, specially that of Eugenio Zalamea. We have made a
careful examination of the evidence... but have not found anything... that
would justify us in disturbing the finding of the court a quo.

The attesting witnesses... clearly testify that together with the other
witness to the will, Pedro de Jesus, they did sign each and every page of
the will and of the... inventory in the presence of each other and of the
testatrix, as the latter did likewise sign all the pages of the will and of the
inventory in their presence.

the appellants intimate that one of the pages of the will was not signed
by the testatrix, nor by the witnesses on the day of the execution of the
will,... their contention on the testimony of Aurelio Palileo, who says...
that on one occasion Gonzalo Abaya told him that one of the pages of the
will had not been signed by the witnesses... on the day of its execution.

Palileo's testimony is entirely contradicted by Gonzalo Abaya... in the


direct, but in the rebuttal,... evidence as well.

Palileo's testimony cannot prevail over that of the attesting witnesses,...


The appellants impeach the credibility of Eugenio Zalamea, for having
made a sworn declaration... justice of the peace of Santa
Cruz, Laguna,... before the trial of this case,... Zalamea's testimony in
connection with the dismissal of a criminal case against a... nephew of
his, in whose success he was interested,... infer from this fact the
partiality of his testimony.

allegation of little importance to impeach the credibility of the witness


Zalamea,... because his testimony is corroborated... sufficient for us to
conclude that the first assignment of error made by the appellants is
groundless.

do not think this question properly to have been raised at the trial, but in
the memorandum submitted by the attorney for the appellants to the trial
court, he contended that the will could not be admitted to probate
because one of the witnesses to the... will was not produced... that the
voluntary non-production of this witness raises a presumption against the
pretension of the proponent.

Issues:

the court below erred in admitting the will to probate notwithstanding the
omission of the proponent to produce one of the attesting witnesses.

proponent stated to the court that they had necessarily to omit the
testimony of Pedro de Jesus, one of the persons who appear to have
witnessed the execution of the will... there were reasonable grounds to
believe that said... witness was openly hostile to the proponent, inasmuch
as since the announcement of the trial of the petition for the probate of
the will, said witness has been in frequent communication with the
contestants and their attorney, and has refused to hold any conference
with the... attorneys for the proponent. In reply to this, the attorney for
the contestants, said to the court, "without discussing for the present
whether or not in view of those facts (the facts mentioned by the
attorneys for the petitioner), in the hypothesis that the same are
proven,... they are relieved from producing that witness, for while it is a
matter not decided, it is a recognized rule that the fact that a witness is
hostile does not justify a party to omit his testimony; without discussing
this, I say, I move that said statement be stricken out, and if... the
proponent wants these facts to stand in the record, let him prove them."
The court a quo ruled, saying, "there is no need."

To this ruling of the court, the attorney for the appellants did not take any
exception.

Ruling:
The trial court found that the evidence introduced by the proponent,
consisting of the testimony of the two attesting witnesses and the... other
witness who was present at the execution, and had charge of the
preparation of the will and the inventory, Exhibits A and A-1, was
sufficient.

The last error assigned by the appellants is made to consist in the


probate of the inventory, Exhibit A-1, despite the fact that this exhibit has
no attestation clause in it, and its paging is made in Arabic numerals and
not in letters.

In view of the fact that the inventory is referred to in the will as an


integral part of it, we find that the foregoing attestation clause is in
compliance with section 1 of Act No. 2645, which requires this solemnity
for the validity of a will, and makes unnecessary any... other attestation
clause at the end of the inventory

It was held that this way of numbering the pages of a will is in


compliance with the spirit of the law, inasmuch as either one of these
methods indicates the correlation of the... pages and serves to prevent
the abstraction of any of them. In the course of the decision,... We see no
reason why the same rule should not be applied where the paging is in
Arabic numerals, instead of in letters, as in the inventory in question. So
that, adhering to the view taken by this court in the case of Abangan vs.
Abangan, and followed in Aldaba vs. Roque,... with regard to the
appreciation of the solemnities of a will, we find that the judgment
appealed from should be, as is hereby, affirmed with the costs against the
appellants. So ordered.

Principles:
64 Phil. 187

AVANCENA, C.J.:
This is an action brought by the plaintiff, as executrix of the testate estate of
the deceased Leonarda Macam, for the recovery from the defendants Juana
Gatmaitan and Magno S. Gatmaitan of the ownership of the house
described in paragraph 2 of the complaint. The plaintiff appealed from the
decision of the court absolving the defendants.
On September 24, 1929, the deceased Leonarda Macam and the defendant
Juana Gatmaitan purchased the house in question for P3,000 from the
spouses Generoso Inducil and Flora Hamos (Exhibit B). It is stated in the
deed of sale that the vendors received the purchase price of the house from
the vendees, both single. However, on June 12, 1932, the deceased
Leonarda Macam and the defendant Juana Gatmaitan subscribed a
document (Exhibit C) which reads as follows:
"Know all men by these presents:
"That we, Leonarda Macam and Juana Gatmaitan, both single, of age and
residents of the municipality of Calumpit, Province of Bulacan, Philippine
Islands, by means of this document, freely and voluntarily state as follows:
"1. That during the time we lived together as friends, we have purchased a
house of strong materials built on a lot belonging to the Diocese, situated in
the municipality of Calumpit, Province of Bulacan, and declared for
taxation purposes under Tax No. 6977, one Buick automobile, and furniture
necessary for the house.
"2. That I, Juana Gatmaitan, hereby declare that the house purchased by us
was paid with my friend Leonarda Macam's own money in the sum of three
thousand pesos (P3,000) and therefore, said house truly belongs to my said
friend. The following furniture likewise truly belong to her:
"One (1) wardrobe with mirror and carved top.
"One (1) narra bed.
"One (1) small wooden wardrobe.
"One (1) small wooden table.
"One (1) narra chair.
"One (1) rattan sofa.
"One (1) dining room table.
"One (1) kitchen table.
"Two (2) dining room benches.
"Two (2) kitchen benches.
"One (1) ice box,
"3. I, Leonarda Macam also hereby declare that the Buick automobile and
most of the furniture in the house where we live, as the narra chairs,
wardrobe and bed, truly belong to my friend Juana Gatmaitan, said
automobile and most of said furniture having been bought with money
exclusively belonging to her; and she was also the one who had my house
painted.
"4. That in consideration of the friendship we mutually profess, considering
ourselves almost as sisters, we have voluntarily agreed that whoever of us
will die first shall leave to the survivor, as the latter's property, the house
and all the furniture therein together with the Buick automobile above-
stated, excluding the furniture belonging to Leonarda Macam stated in
paragraph 2 of this document, which may be taken by the heirs of said
Leonarda Macam if she will be the first to give up her soul to God, as a
remembrance to her surviving friend, and this agreement shall be
equivalent to a transfer of the rights of the one who dies first and shall be
kept by the survivor; and none of our heirs shall claim the property
mentioned in this document, left by any of us who dies first.
"In witness whereof, we affix our signature at the foot of this document as a
proof of the acceptance by each of us of this agreement, this 12th day of
July, 1932, in the municipality of Calumpit, Province of Bulacan, P. I."
It is inferred from the foregoing document that the deceased Leonarda
Macam and the defendant Juana Gatmaitan lived together as friends,
Leonarda having contributed the house and Juana the Buick automobile
and most of the furniture to such companionship, both having thereby
established between themselves a de facto joint ownership of the properties
respectively contributed by them, which, judging from their nature and
description/ are more or less of the same value. Such must be the case,
judging from the fact that, although the house was purchased with money
exclusively belonging to Leonarda, it was made to appear that both were the
purchasers.
The plaintiff contends that with respect to the house, Exhibit C, on the part
of Leonarda, constitutes a donation mortis causa in favor of Juana, and that
as it had not been executed with all the formalities required by law for a
will, it is entirely invalid and did not produce the effect of conveying the
ownership of the house to Juana.
The lower court, in absolving the defendants from the complaint,
considered the act of the deceased Leonarda as a transfer of the ownership
of the house in favor of Juana, but not in the concept of a donation. This
conclusion of the court below is supported by the literal interpretation of
Exhibit C, wherein the parties describe the act performed by them as an
agreement and a transfer.
This court is of the opinion that Exhibit C is an aleatory contract whereby,
according to article 1790 of the Civil Code, one of the parties or both
reciprocally bind themselves to give or do something as an equivalent for
that which the other party is to give or do in case of the occurrence of an
event which is uncertain or will happen at an indeterminate time. As
already stated, Leonarda was the owner of the house and Juana of the
Buick automobile and most of the furniture. By virtue of Exhibit C, Juana
would become the owner of the house in case Leonarda died first, and
Leonarda would become the owner of the automobile and the furniture if
Juana were to die first. In this manner Leonarda and Juana reciprocally
assigned their respective property to one another conditioned upon who
might die first, the time of death determining the event upon which the
acquisition of such right by the one or the other depended. This contract, as
any other contract, is binding upon the parties thereto. Inasmuch as
Leonarda had died before Juana, the latter thereupon acquired the
ownership of the house, in the same manner as Leonarda would have
acquired the ownership of the automobile and of the furniture if Juana had
died first.
In view of the foregoing considerations, the judgment appealed from is
affirmed with costs to the appellant. So ordered.
Villa-Real, Abad Santos, Imperial, Diaz, Laurel, and Concepcion,
JJ., concur.
Gago v. Mamuyac Digest
Gago vs. Mamuyac

G.R. No. L-26317 January 29, 1927

Johnson, J. (Ponente)

Facts:

1. Previously, Francisco Gago filed a petition for the probate of a will of Miguel Mamuyac executed
on July 27, 1918. The oppositors alleged that the said will was already annulled and revoked. It
appeared that on April 16, 1919, the deceased executed another will. The lower court denied the
probate of the first will on the ground of the existence of the second will.

2. Another petition was filed to seek the probate of the second will. The oppositors alleged that the
second will presented was merely a copy. According to the witnesses, the said will was allegedly
revoked as per the testimony of Jose Tenoy, one of the witnesses who typed the document. Another
witness testified that on December 1920 the original will was actually cancelled by the testator.

3. The lower court denied the probate and held that the same has been annulled and revoked.

Issue: Whether or not there was a valid revocation of the will

RULING: Yes. The will was already cancelled in 1920. This was inferred when after due search, the
original will cannot be found. When the will which cannot be found in shown to be in the possession
of the testator when last seen, the presumption is that in the absence of other competent evidence,
the same was deemed cancelled or destroyed. The same presumption applies when it is shown that
the testator has ready access to the will and it can no longer be found after his death.

MOLO vs. MOLO [No. L-2538. September 21, 1951] FACTS: Mariano Molo y Legaspi died in 1941,
without heirs in the descending and ascending lines. He was survived by his wife, petitioner Juana Molo,
and by his nieces and nephew, the oppositors-appellants. He left two wills, one executed in 1918 and
another executed in 1939. The 1939 will contained a clause which expressly revoked the 1918 will. In
1941, Juana Molo sought the probate of the 1939 will, and there being no opposition, the will was
probated. However, the testator’s nieces and nephew opposed the order of the court admitting the will
to probate; and the lower court reversed its original order and, instead, denied the probate of the said
will on the ground that the petitioner failed to prove that it was executed in accordance with law. Juana
Molo then filed another petition for the probate of the 1918 will. Again, the same oppositors filed an
opposition to the petition based on three grounds: (1) that petitioner is now estopped from seeking the
probate of the will of 1918; (2) that said will has not been executed in the manner required by law and
(3) that the will has been subsequently revoked. But before the second petition could be heard, WWII
broke out and the records of the case were destroyed. A petition for reconstitution was filed, but the
same was found to be impossible because neither petitioner nor oppositors could produce the copies
required for reconstitution. So, petitioner filed a new petition in 1946, similar to the one destroyed, but
the oppositors again filed an opposition based on the same grounds as before. After trial, the lower
court issued an order admitting the will to probate; hence, this appeal by the oppositors.

ISSUES: 1. W/N petitioner voluntarily and deliberately frustrated the probate of the 1939 will in order to
enable her to obtain the probate of the 1918 will; 2. W/N petitioner is estopped from seeking the
probate of the 1918 will; and 3. W/N the 1918 will has been subsequently revoked by the 1939 will.

RULING: 1. NO, she did not. There was no evidence which justified the insinuation that petitioner had
deliberately intended to frustrate the probate of the 1939 will of the testator to enable her to seek the
probate of another will. In fact, it was because of the oppositors’ insistent opposition that the probate of
the 1939 will was denied. 2. NO, she is not. Petitioner Juana cannot be considered guilty of estoppel
which would prevent her from seeking the probate of the 1918 will simply because in both the 1918 and
1939 wills, she was instituted by her husband as his universal heir. 3. NO, it was not. "A subsequent will,
containing a clause revoking a previous will, having been disallowed, for the reason that it was not
executed in conformity with the provisions of section 618 of the Code of Civil Procedure as to the
making of wills, cannot produce the effect of annulling the previous will, inasmuch as said revocatory
clause is void." A will is not “revoked by a defectively executed will or codicil, even though the latter
contains a clause expressly revoking the former will, in a jurisdiction where it is provided by a controlling
statute that no writing other than a testamentary instrument is sufficient to revoke a will, for the simple
reason that there is no revoking will.” Since the 1939 will was declared invalid, it means that it had no
effect whatsoever as a revoking will; and the 1918 will, therefore, remained unrevoked.

Dependent revocation will – because the validity is depedent on the second will for the revocation

GR No. 76464 Maloto vs CA


Facts: Adriana Maloto died leaving as heirs her niece and
nephews, the petitioners Aldina Maloto- Casiano and Constancio,
Maloto, and the private respondents Panfilo Maloto and Felino
Maloto. Believing that the deceased did not leave behind a last
will and testament, these four heirs commenced an intestate
proceeding for the settlement of their aunt’s estate. While the
case was still in progress, they executed an extrajudicial
settlement of Adriana’s estate dividing it into four equal parts
among themselves. They presented the same and successfully gained
court approval. Three years later, a document was discovered
entitled “KATAPUSAN NGA PAGBUBULAT-AN (Testamento),” purporting
to be the last will and testament of Adriana. Malotos oppposed
the probate of the Will stating among others that the said will
was revoked. Two witnesses were presented to prove that the will
was burned by Adriana herself.

Issue: Whether or not the will was validly revoked.

Held: No, the will was not validly revoked. A valid revocation
must be done with animus revocandi or the intention to revoke
coupled with an overt physical act of burning, tearing,
obliterating, or cancelling the will carried out by the testator
or by another person in his presence and under his express
direction. The document or papers burned by Adriana’s maid,
Guadalupe, was not satisfactorily established–that such was the
will of Adriana Maloto. And that the burning was not proven to
have been done under the express direction of Adriana. Also the
burning was not in her presence. Both witnesses stated that they
were the only ones present at the place where papers were burned.
The act done by the witnesses could not have constituted a valid
revocation of Adriana’s Will.

TESTATE ESTATE OF LATE ADRIANA MALOTO v. CA, GR No. 76464, 1988-


02-29

Facts:

Adriana Maloto died leaving as heirs her niece and nephews, the
petitioners Aldina Maloto-Casiano and Constancio Maloto, and the private
respondents Panfilo Maloto and Felino Maloto. Believing that the
deceased did not leave behind a last will and testament,... these four
heirs commenced on November 4, 1963 an intestate proceeding for the
settlement of their aunt's estate.

Three years later, or sometime in March 1967, Atty. Sulpicio Palma, a


former associate of Adriana's counsel, the late Atty. Eliseo Hervas,
discovered a document entitled "KATAPUSAN NGA PAGBULUT-AN
(Testamento)", dated January 3, 1940, and purporting to be the last will...
and testament of Adriana. Atty. Palma claimed to have found the
testament, the original copy, while he was going through some materials
inside the cabinet drawer formerly used by Atty. Hervas. The document
was submitted to the office of the clerk of the Court of First Instance... of
Iloilo on April 1, 1967. Incidentally, while Panfilo and Felino are still
named as heirs in the said will, Aldina and Constancio are bequeathed
much bigger and more valuable shares in the estate of Adriana than what
they received by virtue of the agreement of extrajudicial... settlement
they had earlier signed. The will likewise gives devises and legacies to
other parties, among them being the petitioners Asilo de Molo, the Roman
Catholic Church of Molo, and Purificacion Miraflor.

Issues:

whether or not the will was revoked by Adriana.

Ruling:

the document or papers burned by Adriana's maid, Guadalope, was not


satisfactorily established to be a will at all, much less the will of Adriana
Maloto. For another, the burning was not proven to have been done under
the express direction of

Adriana. And then, the burning was not in her presence. Both witnesses,
Guadalope and Eladio, were one in stating that they were the only ones
present at the place where the stove (presumably in the kitchen) was
located in which the papers proffered as a will were burned.

Principles:

Art. 830. No will shall be revoked except in the following cases:

(1)

By implication of law; or

(2)

By some will, codicil, or other writing executed as provided in case of


wills; or

(3)
By burning, tearing, cancelling, or obliterating the will with the intention
of revoking it, by the testator himself, or by some other person in his
presence, and by his express direction. If burned, torn, cancelled, or
obliterated by some other person,... without the express direction of the
testator, the will may still be established, and the estate distributed in
accordance therewith, if its contents, and due execution, and the fact of
its unauthorized destruction, cancellation, or obliteration are established
according to the

Rules of Court.

the physical act of destruction of a will, like burning in this case, does not
per se constitute an effective revocation, unless the destruction is
coupled with animus revocandi on the part of the testator. It is not
imperative that the physical destruction be... done by the testator
himself. It may be performed by another person but under the express
direction and in the presence of the testator. Of course, it goes without
saying that the document destroyed must be the will itself.

Animus revocandi is only one of the necessary elements for the effective
revocation of a last will and testament.

The intention to revoke must be accompanied by the overt physical act of


burning, tearing, obliterating, or cancelling the will carried out by the
testator or by another person in his presence and under his express
direction.

VII. B.

TESTATE ESTATE OF THE DECEASED MARIANO MOLO Y LEGASPI. JUANA


JUAN VDA. DE MOLO v. LUZ, GLICERIA AND CORNELIO MOLO

G.R. No. L-2538 September 21, 1951

Facts:
Mariano Molo y Legaspi died on January 24, 1941, in the municipality of
Pasay, province of Rizal, without leaving any forced heir either in the
descending or ascending line. He was survived, however, by his wife, the
herein petitioner Juana Juan Vda. de Molo, and by his nieces and nephew, the
oppositors-appellants, Luz Gliceria and Cornelio, all surnamed Molo, who
were the legitimate children of Candido Molo y Legaspi, deceased brother of
the testator. Mariano Molo y Legaspi left two wills, one executed on August 17,
1918 and another executed on June 20, 1939.

On February 7, 1941, Juana Juan Vda. de Molo, filed in the Court of First
Instance of Rizal a petition, seeking the probate of the will executed by the
deceased on June 20, 1939. There being no opposition, the will was probated.
However, upon petition filed by the herein oppositors, the order of the court
admitting the will to probate was set aside and the case was reopened. After
hearing, at which both parties presented their evidence, the court rendered
decision denying the probate of said will on the ground that the petitioner
failed to prove that the same was executed in accordance with law.

In view of the disallowance of the will executed on June 20, 1939, the widow
on February 24, 1944, filed another petition for the probate of the will
executed by the deceased on August 17, 1918, in the same court. Again, the
same oppositors filed an opposition to the petition based on three grounds: (1)
that petitioner is now estopped from seeking the probate of the will of 1918;
(2) that said will has not been executed in the manner required by law and (3)
that the will has been subsequently revoked.

Issues:

1. Was Molo’s will of 1918 subsequently revoked by his will of 1939?


2. Assuming that the destruction of the earlier will was but the necessary
consequence of the testator’s belief that the revocatory clause contained
in the subsequent will was valid and the latter would be given effect, can
the earlier will be admitted to probate?

Doctrines:

1. NO. In the case of Samson vs. Naval, the court laid down the doctrine
that “a subsequent will, containing a clause revoking a previous will,
having been disallowed, for the reason that it was not executed in
conformity with the provisions of section 618 of the Code of Civil
Procedure as to the making of wills, cannot produce the effect of
annulling the previous will, inasmuch as said revocatory clause is void.”
Although American authorities on the subject have a pool of conflicting
opinions perhaps because of the peculiar provisions contained in the statutes
adopted by each State in the subject of revocation of wills, the court is of the
impression from a review and the study of the pertinent authorities that the
doctrine laid down in the Samson case is still a good law.

1. YES. The earlier will can still be admitted to probate under the
principle of “dependent relative revocation”. The failure of a new
testamentary disposition upon whose validity the revocation depends, is
equivalent to the non-fulfillment of a suspensive condition, and hence
prevents the revocation of the original will. But a mere intent to make at
some time a will in the place of that destroyed will not render the
destruction conditional. It must appear that the revocation is dependent
upon the valid execution of a new will.

TESTATE ESTATE OF THE DECEASED MARIANO MOLO Y LEGASPI. JUANA


JUAN VDA. DE MOLO v. LUZ, GLICERIA AND CORNELIO MOLO

G.R. No. L-2538 September 21, 1951

Facts:

When Mariano Molo y Legaspi, he left without leaving any heir either in the
descending or ascending line. Juana Juan Vda. de Molo, so she is the
petitioner in this case together with mariano’s nieces and nephew.

Luz Gliceria and Cornelio Molo, they were the respondents. they were the
legitimate children of Candido Molo y Legaspi, the brother of the Mariano who
was also a deceased. Mariano before he died left two wills.

Juana, the wife of Mariano, filed a petition for the probate of the will executed
by his husband Mariano and the probate was approved but when a petition
was filed filed by Luz Gliceria and Cornelio Molo, the case was reopened in
which both parties presented their evidence, the probate was denied by the
honorable court saying that the petitioner failed to prove that the probate was
executed legally. So because of the denial of the will, Juana, the wife of
Mariano, filed another petition for the probate, so the same respondents file
an opposition due to these 3 reasons, first the petitioner is now prevented by
the honorable court from seeking the probate of the will , second the said will
has not been executed legally and (3) the will has been revoked.

Issues:

3. Assuming that the destruction of the earlier will was the consequence of
the testator’s belief that the revocatory clause contained in the
subsequent will was valid and the latter would be given effect, can the
earlier will be admitted to probate?

Doctrines:

2. NO. In the case of Samson vs. Naval, the court laid down the doctrine
that “a subsequent will, containing a clause revoking a previous will,
having been disallowed, for the reason that it was not executed in
conformity with the provisions of section 618 of the Code of Civil
Procedure as to the making of wills, cannot produce the effect of
annulling the previous will, inasmuch as said revocatory clause is void.”

Although American authorities on the subject have a pool of conflicting


opinions perhaps because of the peculiar provisions contained in the statutes
adopted by each State in the subject of revocation of wills, the court is of the
impression from a review and the study of the pertinent authorities that the
doctrine laid down in the Samson case is still a good law.

2. YES. The previous will can still be admitted to probate under the
principle of “dependent relative revocation”.

When we talk about dependent relative revocation this is usually applied


where the testator cancels or destroys a will or executes an instrument intended to revoke a
will with a present intention to make a new testamentary disposition as a substitute for the
old, and the new disposition is not made or, if made, fails of effect for same reason. The
failure of a new testamentary disposition upon whose validity the
revocation depends, is equivalent to the non-fulfillment of a suspensive
condition, so this prevents the revocation of the original will. It must
appear that the revocation is dependent upon the valid execution of a
new will. However, the intent to make a will in the place of that
destroyed will not render the destruction conditional.
Rodelas v. Aranza Digest
Rodelas v. Aranza

G.R. No. L-58509 December 7, 1982

Relova, J. (Ponente)

Facts:

1. The appellant filed a petition for the probate of the holographic will of Ricardo Bonilla in 1977. The
petition was opposed by the appellees on the ground that the deceased did not leave any will,
holographic or otherwise.

2. The lower court dismissed the petition for probate and held that since the original will was lost, a
photostatic copy cannot stand in the place of the original.

Issue: Whether or not a holographic will can be proved by means of a photocopy

RULING: Yes. A photocopy of the lost or destroyed holographic will may be admitted because the
authenticity of the handwriting of the deceased can be determined by the probate court with the
standard writings of the testator.

Rodelas v. Aranza Digest


Rodelas v. Aranza

G.R. No. L-58509 December 7, 1982

Relova, J. (Ponente)
Facts:

Ricardo Bonilla had a holographic will. So in this case, the appellant filed a case for the probate of
the will. The petition was opposed since according to the appellees, the deceased did not leave any
holographic will or any kind of will.The honorable court dismissed the petition for probate since the
original will was lost and a photocopied wil cannot replace the original.

Issue: Whether or not a holographic will can be proved by means of a photocopy

RULING of the honorable court is that :

The Holographic will can be proved by means of a photocopy. A photocopy of a lost or destroyed
holographic will can be admitted as long as the authenticity of the handwriting of the testator can be
determined by the probate court with the standard writings of the testator.

Sebial v. Sebial Digest

G.R. No. L-23419


Facts of the Case:

Gelacio Sebial died in 1943, he had 3 children with this 1st wife Reoncia (Roberta's mother) and 6
other children with his 2nd wife Dolores, (Benjamina's mother). In 1960, BEjamina filed for the
settlement of her father's estate and her appointment as administrator. Thisd petition was oppsed by
Roberta on the ground that said estate had already been apportioned and that she should be the
one appointed as administrator and not Benjamina. The Court appointed Benjamina and found that
alleged partition was invalid and ineffective. So the letters of administration were issued and a notice
to the creditors was issue don the same date. The oppositors motion for reconsideration was denied.
For the possibility of an amicale settlement, the court ordered both sides to give a complete list of
the porperties of te decedent with segregation for each marriage.

On Nov. 1961, the lower court approved the administrator's inventory (second one) or six months
from the appointment. Roberta them moved for the motion reocnsideration alleging as ground that
the court has no jusridiction to approve the inventory as it was files beyiind the 3-month period. The
Court of Appeals certified the case to the Supreme Court.

Issue: Did the court lose jurisdiction to approve the inventory which was made 6 months
after the appointment?

Ruling. NO. Under section 1 of Rule 83 of the Rules of Court, the prescribed three-month period is
not mandatory. Once a petition for the issuance of letters of administration is filed with the proper
court and the publication of the notice of hearing is complied with, said court acquires jurisdiction
over the estate and retains such until the probate proceedings is closed. Hence, even if the inventory
was filed only after the three-month period, this delay will not deprive the probate court of its
jurisdiction to approve it. However, under section 2 of Rule 82 of the Rules of Court, such
unexplained delay can be a ground for an administrator's removal.

Sebial v. Sebial Digest

G.R. No. L-23419


Facts of the Case:

Gelacio Sebial, a deceased, married his first wife and he had also a second wife named Dolores,
Gelacio had 3 children with his first wife Reoncia and on his 2nd wife Dolores, he had 6 children.
Benjamina, one of the children of Gelacio to his 2nd wife, Dolores filed for the settlement of her
father's estate and also so she can be appointed as an administrator. The petition was opposed by
Roberta who is one of the daughters of Gelacio to his first wife Reoncia on the ground that the estate
of Gelacio had been divided or allocated and that Roberta wanted to be appointed as administrator
instead of Benjamina. So the Honorable Court had chosen Benjamina and said that allocation or
division of the esate was invalid. So the letters of administration were issued. The motion for
reconsideration of the respondents was denied. So to have an amicable settlement, there was an
order by the honorable court to present a complete list of the properties of the decendants
separating the properties for their spouses.So the honorable court approved the administrator's
inventory (second one) or six months from the appointment. Roberta them moved for the motion
reocnsideration alleging as ground that the court has no jusridiction to approve the inventory as it
was files beyiind the 3-month period. The Court of Appeals certified the case to the Supreme Court.

Issue: Did the court lose jurisdiction to approve the inventory which was made 6 months
after the appointment?

Ruling. NO. Under section 1 of Rule 83 of the Rules of Court, the prescribed three-month period is
not mandatory. Once a petition for the issuance of letters of administration is filed with the proper
court and the publication of the notice of hearing is complied with, said court acquires jurisdiction
over the estate and retains such until the probate proceedings is closed. Hence, even if the inventory
was filed only after the three-month period, this delay will not deprive the probate court of its
jurisdiction to approve it. However, under section 2 of Rule 82 of the Rules of Court, such
unexplained delay can be a ground for an administrator's removal.

ROSA CAYETANO CUENCO v. CA, GR No. L-24742, 1973-10-26

Facts:

On 25 February 1964 Senator Mariano Jesus Cuenco died at the Manila


Doctors' Hospital, Manila. He was survived by his widow, the herein
petitioner, and their two (2) minor sons, Mariano Jesus, Jr. and Jesus
Salvador, both surnamed Cuenco, all residing at 69 Pi y Margal
St., Sta. Mesa Heights, Quezon City, and by his children of the first
marriage, respondents herein, namely, Manuel Cuenco, Lourdes Cuenco,
Concepcion Cuenco Manguera, Carmen Cuenco, Consuelo Cuenco Reyes
and Teresita Cuenco Gonzalez, all of legal age and residing in Cebu.

On 5 March 1964, (the 9th day after the death of the late Senator)[1]
respondent Lourdes Cuenco filed a Petition for Letters of Administration
with the court of first instance of Cebu (Sp. Proc. No. 2433-R), alleging
among other things, that the late... senator died intestate in Manila on 25
February 1964; that he was a resident of Cebu at the time of his death;
and that he left real and personal properties in Cebu and Quezon City.

In the meantime, or specifically on 12 March 1964, (a week after the filing


of the Cebu petition) herein petitioner Rosa Cayetano Cuenco filed a
petition with the court of first instance of Rizal (Quezon City) for the
probate of the deceased's last will and... testament and for the issuance
of letters testamentary in her favor, as the surviving widow and executrix
in the said last will and testament. The said proceeding was docketed as
Special Proceeding No. Q-7898.

Having learned of the intestate proceeding in the Cebu court, petitioner


Rosa Cayetano Cuenco filed in said Cebu court an Opposition and Motion
to Dismiss, dated 30 March 1964, as well as an Opposition to Petition for
Appointment of Special Administrator, dated 8 April

1964. On 10 April 1964, the Cebu court issued an order holding in


abeyance its resolution on petitioner's motion to dismiss "until after
the Court of First Instance of Quezon City shall have acted on the
petition for probate of that document purporting to be... the last will
and testament of the deceased Don Mariano Jesus Cuenco."

Instead, respondents filed in the Quezon City court an Opposition and


Motion to Dismiss, dated 10 April 1964, opposing probate of the will and
assailing the jurisdiction of the said Quezon City court to entertain
petitioner's petition for probate and for appointment as... executrix in Sp.
Proc. No. Q-7898 in view of the alleged exclusive jurisdiction vested by
her petition in the Cebu court in Sp. Proc. No. 2433 - R. Said respondent
prayed that Sp. Proc. No. Q-7898 be dismissed for lack of jurisdiction
and/or improper venue.

In its order of 11 April 1964, the Quezon City court denied the motion to
dismiss, giving as a principal reason the "precedence of probate
proceeding over an intestate proceeding."[4] The said court further found
in said order that the... residence of the late senator at the time of his
death was at No. 69 Pi y Margal, Sta. Mesa Heights, Quezon City.

Instead of appealing from the Quezon City court's said order admitting the
will to probate and naming petitioner-widow as executrix thereof,
respondents filed a special civil action of certiorari and prohibition with
preliminary injunction with respondent Court of

Appeals (docketed as case CA-G.R. No. 34104-R) to bar the Rizal court
from proceeding with case No. Q-7898.

On 21 November 1964, the Court of Appeals rendered a decision in favor


of respondents (petitioners therein)

Issues:

whether the Quezon City court acted without jurisdiction or with grave
abuse of discretion in taking cognizance and assuming exclusive
jurisdiction over the probate proceedings filed with it, in pursuance of the
Cebu court's order of 10

April 1964 expressly consenting in deference to the precedence of


probate over intestate proceedings

Ruling:

The Court finds under the above-cited facts that the appellate court erred
in law in issuing the writ of prohibition against the Quezon City court from
proceeding with the testate proceedings and annulling and setting aside
all its orders and actions, particularly its admission... to probate of the
deceased's last will and testament and appointing petitioner-widow as
executrix thereof without bond pursuant to the deceased testator's
express wish

Conversely, such court, may upon learning that a petition for probate of
the decedent's last will has been presented in another court where the
decedent obviously had his conjugal domicile and resided with his
surviving widow and their minor children, and that the allegation of... the
intestate petition before it stating that the decedent died intestate may
be actually false, may decline to take cognizance of the petition and hold
the petition before it in abeyance, and instead defer to the second court
which has before it the petition for probate of the... decedent's alleged
last will.

This is exactly what the Cebu court did. Upon petitioner-widow's filing
with it a motion to dismiss Lourdes' intestate petition, it issued its order
holding in abeyance its action on the dismissal motion and deferred to the
Quezon City court, awaiting its action on the... petition for probate before
that court. Implicit in the Cebu court's order was that if the will was duly
admitted to probate by the Quezon City court, then it would definitely
decline to take cognizance of Lourdes' intestate petition which would
thereby be shown to be false and... improper, and leave the exercise of
jurisdiction to the Quezon City court, to the exclusion of all other courts.
Likewise by its act of deference, the Cebu court left it to the Quezon City
court to resolve the question between the parties whether the decedent's
residence at the... time of his death was in Quezon City where he had his
conjugal domicile rather than in Cebu City as claimed by respondents.
The Cebu court thus indicated that it would decline to take cognizance of
the intestate petition before it and instead defer to the Quezon City
court,... unless the latter would make a negative finding as to the probate
petition and the residence of the decedent within its territory and venue.

3. Under these facts, the Cebu court could not be held to have acted
without jurisdiction or with grave abuse of jurisdiction in declining
to take cognizance of the intestate petition and deferring to the
Quezon City court.

Necessarily, neither could the Quezon City court be deemed to have


acted without jurisdiction in taking cognizance of and acting on the
probate petition since under Rule 73, section 1, the Cebu court must first
take cognizance over the estate of the decedent and must exercise...
jurisdiction to exclude all other courts, which the Cebu court declined to
do. Furthermore, as is undisputed, said rule only lays down a rule of venue
and the Quezon City court indisputably had at least equal and co-ordinate
jurisdiction over the estate.

Since the Quezon City court took cognizance over the probate petition
before it and assumed jurisdiction over the estate, with the consent and
deference of the Cebu court, the Quezon City court should be left now, by
the same rule of venue of said Rule 73, to exercise... jurisdiction to the
exclusion of all other courts.

For the same reasons, neither could the Quezon City court be held to have
acted without jurisdiction nor with grave abuse of discretion in admitting
the decedent's will to probate and appointing petitioner as executrix in
accordance with its testamentary disposition, in... the light of the settled
doctrine that the provisions of Rule 73, section 1 lay down only a rule of
venue, not of jurisdiction.

Principles:
Vda. De Baluyut vs. Luciano
Gallanosa v. Arcangel Digest
Gallanosa v. Arcangel

Facts:

1. Florentino Gallanosa executed a will in 1938 when he was 80 years old. He owned 61 parcels of
and at that time. He died in 1939 childless and survived by his brother Leon. In his will, he
bequethed his 1/2 share of the conjugal estate to his second wife Tecla and if she predecease him
(as what occurred), the said share shall be assigned to the spouses Gallanosa (Pedro & Corazon).
Pedro is Tecla's son by her 1st marriage. He also gave 3 parcels of land to Adolfo, his protege.

2. The said will was admitted to probate with Gallanosa as executor. In 1952, thjhe legal heirs filed
an action for the recovery of said 61 parcels of land. The action was dismissed on the ground of res
judicata. Then, 28 years after probate, another acton agaisnt Gallanosa for annulment of the will,
recovery of the lands alleging fraud and deceit, was filed. As a result, the lower court set aide the
1939 decree of probate.
Issue: Whether or not a will which has been probated may still be annulled

RULING: No. A final decree of probate is conclusive as to the due execution of the will. Due
execution means that the testator was of sound and disposing mind at the time of the execution and
that he was not acting under duress, menace, fraud or undue influence. Finally, that it was executed
in accordance with the formalities provided by law.

The period for seeking relief under Rule 38 has already expired, hence the judgment may only be
set aside on the grounds of, 1) lack of jurisdiction or lack of due process of law, and 2) the judgment
was obtained by means of extrinsic collateral fraud (which must be filed within 4 years from the
discovery). Finally, Art. 1410 cannot apply to wills and testament.

Gallanosa v. Arcangel Digest


Gallanosa v. Arcangel

Facts:

Florentino Gallanosa made a will on 1938. Florentino owned 61 parcels of land at the age of 80. He
had no children but he has a brother named Leon. So Florentino had stated in his will that half of his
share of his conjugal property with his second wife Tecla, the 1/2 share must be given to the Pedro
& Corazon Gallanosa. Pedro is Tecla's son to her 1st marriage. He also gave 3 parcels of land to
Adolfo, his protege. (kanang kamay). The will of Florentino was admitted to probate with Pedro and
Corazon Gallanosa as executor. Years later, in 1952, other legal heirs wanted to recover the 61
parcels of land so they filed a petition but the action was dismissed on the ground of res judicata.
(known as claim preclusion, is the Latin term for matter decided and refers to
either of two concepts in both civil law and common law legal systems). So
pagkatapos ng 28 years, another action against Gallanosa for annulment of the will of Florentino,
was again filed and the recovery of the lands. So the honorable lower court set aside the 1939
decree of probate.

Issue: Whether or not a will which has been probated may still be annulled
The honorable court’s RULING is that the will which has been probated must not be annulled. The
honorable court stated that a final decree of probate is conclusive to the due execution of the will.
When we say due execution, it means that the testator was of sound mind when he made the will
that he was not acting under fraud or undue influence during the execution and also that the will was
executed in accordance with the formalities provided by law.

The period for seeking relief under Rule 38 A petition for relief is governed by Section 1, Rule 38 of
the Rules of Court. It provides: SEC. 1. Petition for relief from judgment, order, or other proceedings.
- When a judgment or final order is entered, or any other proceeding is thereafter taken against a
party in any court through fraud, accident, mistake, or excusable negligence, he may file a petition in
such court and in the same case praying that the judgment, order or proceeding be set aside.

has already expired so the judgment may only be set aside on the grounds of lack of jurisdiction or
lack of due process of law, and the judgment was obtained by means of extrinsic collateral fraud (so
this can be filed within 4 years from the discovery). So Art. 1410 cannot apply to wills and testament.

363 Phil. 304

PURISIMA, J.:
The pivot of inquiry in this special civil action for Certiorari under Rule 65
is: whether or not the Court of Appeals acted with grave abuse of discretion
in denying petitioner's Motion To Strike Off the Record OSG's Comment
dated June 4, 1996 and in denying her motion to reconsider its December
15, 1995 Decision.

The antecedent facts that matter are, as follows:

On December 15, 1995, the Court of Appeals promulgated its decision in CA


GR No. 15386, entitled People of the Philippines v. Reynaldo Cartalla y
Abasolo.

On January 12, 1996, the petitioner, Dra. Josefa E. Nepomuceno, moved for
reconsideration. Acting thereupon on January 23,1996, the Court of
Appeals required the Office of the Solicitor General to comment within
ten (10) days from notice.

On June 4, 1996, or after more than four (4) months, the Solicitor General
sent in the Comment called for.

On June 19, 1996, while the motion for reconsideration was pending
resolution, petitioner presented a "Motion To Strike Off The Record the
OSG's Comment dated June 4, 1996", contending that subject Comment
was filed beyond the time fixed by the Court of Appeals in its Resolution of
May 17, 1996.

On September 20, 1996, the Court of Appeals denied both the "Motion to
Strike Off the Record OSG's Comment dated June 4, 1996", as well as the
Motion for Reconsideration; ratiocinating, thus:

"The motion to strike off the record the OSG's Comment on the present
motion for reconsideration should be denied considering that after the
issuance of our Resolution dated May 17, 1996, we issued another
resolution dated June 10, 1996 granting plaintiff -appellee a final
extension of fifteen days from May 24, 1996 or until June 8, 1996 within
which to file its comment. The said comment was timely filed on June 6,
1996.

As to the motion for reconsideration filed by the accused, the same is


bereft of merit as the grounds raised therein have already been considered
and passed upon in the Decision of the Court promulgated on December
15, 1995, and to new ground or compelling reason exists as to justify a
reconsideration of the decision.

WHEREFORE, the motion to strike off OSG's Comment and the motion for
reconsideration are DENIED for lack of merit.[1]
Dissatisfied with the aforesaid resolution of the Court of Appeals, petitioner
has come to this Court for relief.

The Petition is not impressed with merit.

Contrary to petitioner's stance, subject Comment of the Solicitor General


filed on June 4, 1996 was seasonable because, as can be gleaned from the
records on hand, the Court of Appeals gave the Solicitor General several
extensions, to wit:
1. On February 27, 1996, the Fifth Division granted an extension of
thirty (30) days from February 9 to March 10, 1996.

2. On March 20, 1996, the same division granted another thirty (30)
days from March 10 to April 9, 1996.

3. On May 3, 1996, the Former Fifth Division granted in the interest of


justice thirty (30) days from April 9 to May 9, 1996.

4. On May 17, 1996, the Former Fifth Division granted fifteen (15) days
from May 9 to May 24, 1996 but definitely for the last time.

5. On June 10, 1996, the Former Fifth Division granted fifteen (15) days
from May 24 to June 8, 1996 but definitely for the last time.

In Esguerra v. Court of Appeals, 335 Phil 60, citing Alafriz v.


Nable, 72 Phil 278 (1941), it was held, that:

"xxx Grave abuse of discretion implies such capricious and a


whimsical exercise of judgment as is equivalent to lack of jurisdiction, or
in other words where the power is exercised in an arbitrary or despotic
manner by reason of passion or personal hostility, and it must be so
patent and gross as to amount to an invasion of positive duty or to a
virtual refusal to perform the duty enjoined or to act at all in
contemplation of the law. "

"There is excess of jurisdiction where the respondent being clothed with


the power to determine the case, oversteps his authority as determined by
law" (Rocha v. Crossfield, 6 Phil 355).

"There is grave abuse of discretion where the respondent acts in a


capricious, whimsical, arbitrary or despotic manner in the exercise of his
judgment as to be said to be equivalent to lack of jurisdiction." (Abad
Santos v. Province of Tarlac, 66 Phil 480, Alafriz v. Nable, 62
Phil 278)
Grave abuse of discretion is indeed a relative term. In the case under
consideration, it is decisively clear that substantial justice would be better
served by allowing the Solicitor General enough time and opportunity to
comment on the motion for reconsideration of petitioner.
"xxx Laws and rules should be interpreted and applied not in a vacuum or
in isolated abstraction but in the light of surrounding circumstances and
attendant facts in order to afford justice to all. xxx

xxx xxx xxx

"xxx technical rules barring a full hearing on the merits should be relaxed,
again in the interest of justice to all." (Magsaysay Lines, Inc. vs.
Honorable Court of Appeals, 329 Phil 312, 313)
WHEREFORE, the Petition is DISMISSED for lack of merit. No
pronouncement as to costs.

[CASE DIGEST] LOURDES L. DOROTHEO v. COURT OF


APPEALS, NILDA D. QUINTANA, for Herself and as Attorney-
in-Fact of VICENTE DOROTHEO and JOSE DOROTHEO
December 8, 1999

Ynares-Santiago, J

FACTS:

 Aniceta Reyes died in 1969 without her estate being settled. Her husband, Alejandro Dorotheo, died
thereafter.

 In 1977, Lourdes Legaspi (aka as Lourdes Dorotheo, the live-in partner of Alejandro) filed a special
proceeding for the probate of Alejandro's last will and testament, alleging that she took care of
Alejandro before he died. In 1981, the lower court admitted said will to probate. This was not opposed
by herein respondents, the legitimate children of Aniceta and Alejandro.
 In 1983, Aniceta and Alejandro's legitimate children filed a Motion to Declare the Will Intrinsically Void
on account of the fact that Lourdes was not married to Alejandro and was therefore not considered an
heir. The trial court granted the motion.

 Lourdes filed a motion for reconsideration, arguing that she was entitled to some compensation since
she took care of Alejandro prior to his death although she admitted that they were not married to each
other. The lower court denied her motion. She then appealed before the CA, but the same was
dismissed for her failure to file appellant's brief within the extended period granted. Subsequently, the
CA order became final and executory. This was followed by a writ of execution issued by the lower
court to implement the final and executory order. But herein respondents still had to file several
motions including a motion to compel Lourdes to surrender to them the Transfer Certificates of Titles
(TCT) covering the properties of the late Alejandro. When Lourdes refused to surrender the TCTs, the
respondents filed a motion for cancellation of said titles and for issuance of new titles in their names.
Lourdes opposed the motion.

 In 1990, the lower court presided by Judge Angas issued an order to set aside the final and executory
CA order and the attendant writ of execution. The judge argued that the order was merely
interlocutory, hence not final in character.

 Expectedly, the respondents filed a motion for reconsideration, which Judge Angas denied. They then
elevated their appeal before the CA, which nullified the orders of Judge Angas and upheld the validity
of the prior final and executory CA order.

 Hence, this petition for review by Lourdes..

RULING:

Petition denied. CA ruling affirmed.

Whether a judgment on a probated will that has attained finality still subject to reversal? – NO.

 A final and executory decision or order can no longer be disturbed or reopened no matter how
erroneous it may be. In setting aside the 1986 Order that has attained finality, Judge Angas of the trial
court in effect nullified the entry of judgment made by the CA. It is well settled that a lower court
cannot reverse or set aside decisions or orders of a superior court, for to do so would be to negate the
hierarchy of courts and nullify the essence of review. It has been ruled that a final judgment on
probated will, albeit erroneous, is binding on the whole world.

 If no appeal is taken in due time from a judgment or order of the trial court, the same attains finality
by mere lapse of time. Thus, the order allowing the will became final and the question determined by
the court in such order can no longer be raised anew, either in the same proceedings or in a different
motion. The matters of due execution of the will and the capacity of the testator acquired the
character of res judicata and cannot again be brought into question, all juridical questions in
connection therewith being for once and forever closed. Such final order makes the will conclusive
against the whole world as to its extrinsic validity and due execution.

On extrinsic and intrinsic validity of wills

 Probate proceedings deals generally with the extrinsic validity of the will sought to be probated,[7]
particularly on four aspects: (a) whether the will submitted is indeed, the decedents last will and
testament; (b) compliance with the prescribed formalities for the execution of wills; (c) the
testamentary capacity of the testator; (d) and the due execution of the last will and testament.

 The intrinsic validity is another matter and questions regarding the same may still be raised even after
the will has been authenticated. Thus, it does not necessarily follow that an extrinsically valid last will
and testament is always intrinsically valid. Even if the will was validly executed, if the testator
provides for dispositions that deprives or impairs the lawful heirs of their legitime or rightful
inheritance according to the laws on succession, the unlawful provisions/dispositions thereof cannot be
given effect. This is specially so when the courts had already determined in a final and executory
decision that the will is intrinsically void. Such determination having attained that character of finality
is binding on this Court which will no longer be disturbed.

Whether the final and executory CA order is erroneous. – THE SC CANNOT REEVALUATE THE MERITS OF
A CASE THAT HAS ATTAINED FINALITY.

 The SC seems to be saying that Lourdes has a point and that the subject will may in fact be
intrinsically valid, but it can't make a final determination because: A final and executory decision of
which the party had the opportunity to challenge before the higher tribunals must stand and should no
longer be reevaluated. Failure to avail of the remedies provided by law constitutes waiver. And if the
party does not avail of other remedies despite its belief that it was aggrieved by a decision or court
action, then it is deemed to have fully agreed and is satisfied with the decision or order.
 Public policy and sound practice demand that, at the risk of occasional errors, judgments of courts
must at some point of time fixed by law become final otherwise there will be no end to litigation.
Interes rei publicae ut finis sit litium - the very object of which the courts were constituted was to put
an end to controversies. To fulfill this purpose and to do so speedily, certain time limits, more or less
arbitrary, have to be set up to spur on the slothful. The only instance where a party interested in a
probate proceeding may have a final liquidation set aside is when he is left out by reason of
circumstances beyond his control or through mistake or inadvertence not imputable to negligence,
which circumstances do not concur herein.

Whether the order declaring Alejandro's will to be intrisically invalid merely interlocutory. – NO.

 Lourdes posits that said order is merely interlocutory, therefore it can still be set aside by the trial
court. She argues that an order merely declaring who are heirs and the shares to which set of heirs is
entitled cannot be the basis of execution to require delivery of shares from one person to another
particularly when no project of partition has been filed.

 But the SC noted that in the same Order, the trial court also said that the estate of the late spouses be
distributed according to the laws of intestacy, given that Lourdes is not the legal wife. Accordingly, it
has no option but to implement that order of intestate distribution and not to reopen and again re-
examine the intrinsic provisions of the same will.

 No intestate distribution of the estate can be done until and unless the will had failed to pass both its
extrinsic and intrinsic validity. If the will is extrinsically void, the rules of intestacy apply regardless of
the intrinsic validity thereof. If it is extrinsically valid, the next test is to determine its intrinsic validity
that is whether the provisions of the will are valid according to the laws of succession. In this case, the
court had ruled that the will of Alejandro was extrinsically valid but the intrinsic provisions thereof
were void. Thus, the rules of intestacy apply as correctly held by the trial court. Therefore, it is
Alejandro's children, not Lourdes (who, legally, is a mere stranger to Alejandro), who should be
considered heirs.

[CASE DIGEST] LOURDES L. DOROTHEO v. COURT OF


APPEALS, NILDA D. QUINTANA, for Herself and as Attorney-
in-Fact of VICENTE DOROTHEO and JOSE DOROTHEO
December 8, 1999

Ynares-Santiago, J

FACTS:
 Aniceta Reyes and Alejandro Dorotheo died without estate being settled. Lourdes Legaspi, a live-in
partner of Alejandro but uses the family name Dorotheo filed a special proceeding for the probate of
Alejandro's last will and testament. According to Lourdes she was the one who took care of Alejandro
before his death. The honorable lower court admitted said will to probate. This was not opposed by the
the legitimate children of Aniceta and Alejandro. So, the mentioned legitimate children filed a Motion
to Declare the Will Intrinsically Void on account of the fact that Lourdes just a live in partner and she is
not married to Alejandro so she cannot be considered an heir. The trial court granted the motion.

 Lourdes filed a motion for reconsideration. According to her, she was entitled to some compensation
since she took care of Alejandro prior to his death although she admitted that they are just live in
partner. The lower court denied her motion. She then appealed before the CA, but the same was
dismissed for her failure to file appellant's brief within the extended period granted. Subsequently, the
CA order became final and executory. This was followed by a writ of execution issued by the lower
court to implement the final and executory order. But herein respondents still had to file several
motions including a motion to compel Lourdes to surrender to them the Transfer Certificates of Titles
(TCT) covering the properties of the late Alejandro. When Lourdes refused to surrender the TCTs, the
respondents filed a motion for cancellation of said titles and for issuance of new titles in their names.
Lourdes opposed the motion.

 In 1990, the lower court presided by Judge Angas issued an order to set aside the final and executory
CA order and the attendant writ of execution. The judge argued that the order was merely
interlocutory, hence not final in character.

 Expectedly, the respondents filed a motion for reconsideration, which Judge Angas denied. They then
elevated their appeal before the CA, which nullified the orders of Judge Angas and upheld the validity
of the prior final and executory CA order.

 Hence, this petition for review by Lourdes..

RULING:

Petition denied. CA ruling affirmed.

Whether a judgment on a probated will that has attained finality still subject to reversal? – NO.
 A final and executory decision or order can no longer be disturbed or reopened no matter how
erroneous it may be. In setting aside the 1986 Order that has attained finality, Judge Angas of the trial
court in effect nullified the entry of judgment made by the CA. It is well settled that a lower court
cannot reverse or set aside decisions or orders of a superior court, for to do so would be to negate the
hierarchy of courts and nullify the essence of review. It has been ruled that a final judgment on
probated will, albeit erroneous, is binding on the whole world.

 If no appeal is taken in due time from a judgment or order of the trial court, the same attains finality
by mere lapse of time. Thus, the order allowing the will became final and the question determined by
the court in such order can no longer be raised anew, either in the same proceedings or in a different
motion. The matters of due execution of the will and the capacity of the testator acquired the
character of res judicata and cannot again be brought into question, all juridical questions in
connection therewith being for once and forever closed. Such final order makes the will conclusive
against the whole world as to its extrinsic validity and due execution.

On extrinsic and intrinsic validity of wills

 Probate proceedings deals generally with the extrinsic validity of the will sought to be probated,[7]
particularly on four aspects: (a) whether the will submitted is indeed, the decedents last will and
testament; (b) compliance with the prescribed formalities for the execution of wills; (c) the
testamentary capacity of the testator; (d) and the due execution of the last will and testament.

 The intrinsic validity is another matter and questions regarding the same may still be raised even after
the will has been authenticated. Thus, it does not necessarily follow that an extrinsically valid last will
and testament is always intrinsically valid. Even if the will was validly executed, if the testator
provides for dispositions that deprives or impairs the lawful heirs of their legitime or rightful
inheritance according to the laws on succession, the unlawful provisions/dispositions thereof cannot be
given effect. This is specially so when the courts had already determined in a final and executory
decision that the will is intrinsically void. Such determination having attained that character of finality
is binding on this Court which will no longer be disturbed.

Whether the final and executory CA order is erroneous. – THE SC CANNOT REEVALUATE THE MERITS OF
A CASE THAT HAS ATTAINED FINALITY.

 The SC seems to be saying that Lourdes has a point and that the subject will may in fact be
intrinsically valid, but it can't make a final determination because: A final and executory decision of
which the party had the opportunity to challenge before the higher tribunals must stand and should no
longer be reevaluated. Failure to avail of the remedies provided by law constitutes waiver. And if the
party does not avail of other remedies despite its belief that it was aggrieved by a decision or court
action, then it is deemed to have fully agreed and is satisfied with the decision or order.

 Public policy and sound practice demand that, at the risk of occasional errors, judgments of courts
must at some point of time fixed by law become final otherwise there will be no end to litigation.
Interes rei publicae ut finis sit litium - the very object of which the courts were constituted was to put
an end to controversies. To fulfill this purpose and to do so speedily, certain time limits, more or less
arbitrary, have to be set up to spur on the slothful. The only instance where a party interested in a
probate proceeding may have a final liquidation set aside is when he is left out by reason of
circumstances beyond his control or through mistake or inadvertence not imputable to negligence,
which circumstances do not concur herein.

Whether the order declaring Alejandro's will to be intrisically invalid merely interlocutory. – NO.

 Lourdes posits that said order is merely interlocutory, therefore it can still be set aside by the trial
court. She argues that an order merely declaring who are heirs and the shares to which set of heirs is
entitled cannot be the basis of execution to require delivery of shares from one person to another
particularly when no project of partition has been filed.

 But the SC noted that in the same Order, the trial court also said that the estate of the late spouses be
distributed according to the laws of intestacy, given that Lourdes is not the legal wife. Accordingly, it
has no option but to implement that order of intestate distribution and not to reopen and again re-
examine the intrinsic provisions of the same will.

 No intestate distribution of the estate can be done until and unless the will had failed to pass both its
extrinsic and intrinsic validity. If the will is extrinsically void, the rules of intestacy apply regardless of
the intrinsic validity thereof. If it is extrinsically valid, the next test is to determine its intrinsic validity
that is whether the provisions of the will are valid according to the laws of succession. In this case, the
court had ruled that the will of Alejandro was extrinsically valid but the intrinsic provisions thereof
were void. Thus, the rules of intestacy apply as correctly held by the trial court. Therefore, it is
Alejandro's children, not Lourdes (who, legally, is a mere stranger to Alejandro), who should be
considered heirs.

LETICIA VALMONTE ORTEGA v. JOSEFINA C. VALMONTE, GR NO. 157451,


2005-12-16

Facts:
Petition for Review[1] under Rule 45 of the Rules of Court, seeking to
reverse and set aside the December 12, 2002 Decision[2] and the March
7, 2003 Resolution[3] of the Court of Appeals

Decision appealed from is REVERSED and SET ASIDE.

rendered approving and allowing probate to the said last will and
testament of Placido Valmonte

Placido toiled and lived for a long time in the United States until he finally
reached retirement. In 1980, Placido finally came home to stay in the
Philippines,... Two years after his arrival from the United States and at
the age of 80 he wed Josefina who was then 28 years old, in a ceremony
solemnized by Judge

Perfecto Laguio, Jr. on February 5, 1982. But in a little more than two
years of wedded bliss, Placido died on October 8, 1984 of a cause written
down as COR PULMONALE.

"Placido executed a notarial last will and testament written in English


and consisting of two (2) pages

"The allowance to probate of this will was opposed by Leticia on the


grounds that:

Petitioner failed to allege all assets of the testator, especially those


found in the USA;

Petitioner failed to state the names, ages, and residences of the heirs of
the testator; or to give them proper notice pursuant to law;

Will was not executed and attested as required by law and legal
solemnities and formalities were not complied with;

Testator was mentally incapable to make a will at the time of the alleged
execution he being in an advance sate of senility;

Will was executed under duress, or the influence of fear or threats;

Will was procured by undue and improper influence and pressure on the
part of the petitioner and/or her agents and/or assistants; and/or

Signature of testator was procured by fraud, or trick, and he did not


intend that the instrument should be his will at the time of affixing his
signature thereto;"

Josefina said she had no... knowledge of the existence of the last will and
testament of her husband, but just serendipitously found it in his attache
case after his death. It was only then that she learned that the testator
bequeathed to her his properties and she was named the executrix in the
said will.

"Notary Public Floro Sarmiento, the notary public who notarized the
testator's will, testified that it was in the first week of June 1983 when
the testator together with the three witnesses of the will went to his
house cum law office and requested him to prepare his last will... and
testament. After the testator instructed him on the terms and dispositions
he wanted on the will, the notary public told them to come back

He likewise explained that though it appears that the will was signed by
the testator and his witnesses on June 15, 1983, the day when it should
have been executed... had he not gone out of town, the formal execution
was actually on August 9, 1983. He reasoned that he no longer changed
the typewritten date of June 15, 1983 because he did not like the
document to appear dirty. The notary public also testified that to his
observation the... testator was physically and mentally capable at the
time he affixed his signature on the will.

"The attesting witnesses to the will corroborated the testimony of the


notary public,... "It then found these grounds extant and proven, and
accordingly disallowed probate."[... appellate court admitted the will of
Placido Valmonte to probate. The CA upheld the credibility of the notary
public and the subscribing witnesses who had acknowledged the due
execution of the will. Moreover, it held that the testator had...
testamentary capacity at the time of the execution of the will. It added
that his "sexual exhibitionism and unhygienic, crude and impolite
ways"[6] did not make him a person of unsound mind.

Issues:

Whether or not the signature of Placido Valmonte in the subject will was
procured by fraud or trickery, and that Placido Valmonte never intended
that the instrument should be his last will and testament.

In short, petitioner assails the CA's allowance of the probate of the will of
Placido Valmonte.

Ruling:

The Petition has no merit.

We stress that the party challenging the will bears the burden of proving
the existence of fraud at the time of its execution.[14] The burden to
show otherwise shifts to the proponent of the will only upon a showing of
credible evidence of fraud.[15] Unfortunately in this case, other than the
self-serving allegations of petitioner, no evidence of fraud was ever
presented.

It is a settled doctrine that the omission of some relatives does not affect
the due execution of a will.[16] That the testator was tricked into signing
it was not sufficiently established by the fact that he had instituted his
wife, who was more than fifty... years his junior, as the sole beneficiary;
and disregarded petitioner and her family, who were the ones who had
taken "the cudgels of taking care of [the testator] in his twilight years."[...
petitioner failed to substantiate her claim of a "grand conspiracy" in the
commission of a fraud. There was no showing that the witnesses of the
proponent stood to receive any benefit from the allowance of the will. The
testimonies of the three subscribing witnesses and... the notary are
credible evidence of its due execution.

the Petition is DENIED, and the assailed Decision and Resolution of the
Court of Appeals are AFFIRMED. Costs against petitioner.

Principles:

Fraud "is a trick, secret device, false statement, or pretense, by which


the subject of it is cheated. It may be of such character that the testator
is misled or deceived as to the nature or contents of the document which
he executes, or it may relate to... some extrinsic fact, in consequence of
the deception regarding which the testator is led to make a certain will
which, but for the fraud, he would not have made."[... the conflict
between the dates appearing on the will does not invalidate the
document, "because the law does not even require that a [notarial] will x
x x be executed and acknowledged on the same occasion."

(Allowance/Disallowance of Wills) Case Citation: Ortega v. Valmonte | G.R. No. 157451 Date: Dec 16,
2005 Petitioners: Leticia Valmonte Ortega Respondents: Josefina C. Valmonte Doctrine: According to
Article 799, the three things that the testator must have the ability to know to be considered of sound
mind are as follows: (1) the nature of the estate to be disposed of, (2) the proper objects of the
testator's bounty, and (3) the character of the testamentary act. Antecedent Facts: Placido toiled and
lived for a long time in the United States until he finally reached retirement. In 1980, Placido finally came
home to stay in the Philippines. Two years after his arrival from the United States and at the age of 80
he wed Josefina who was then 28 years old, on February 5, 1982. But in a little more than two years of
wedded bliss, Placido died on October 8, 1984, of a cause written down as COR PULMONALE. "Placido
executed a notarial last will and testament written in English and consisting of two (2) pages, and dated
June 15, 1983 but acknowledged only on August 9, 1983. The allowance to probate of the will was
opposed by Leticia. Respondent’s contention: The oppositor Leticia declared that Josefina should not
inherit alone because aside from her there are other children from the siblings of Placido who are just as
entitled to inherit from him. She attacked the mental capacity of the testator, declaring that at the time
of the execution of the notarial will the testator was already 83 years old and was no longer of sound
mind. She knew whereof she spoke because in 1983 Placido lived in the Makati residence and asked
Leticia's family to live with him and they took care of him. During that time, the testator's physical and
mental condition showed deterioration, aberrations and senility. This was corroborated by her daughter
Mary Jane Ortega for whom Placido took a fancy and wanted to marry. RTC Ruling: The lower court
disallowed the will. CA Ruling: Reversing the trial court, the appellate court admitted the will of Placido
Valmonte to probate. The CA upheld the credibility of the notary public and the subscribing witnesses
who had acknowledged the due execution of the will. Moreover, it held that the testator had
testamentary capacity at the time of the execution of the will. It added that his "sexual exhibitionism
and unhygienic, crude and impolite ways" did not make him a person of unsound mind. Issues:  WON
the signature of Placido Valmonte in the subject will was procured by fraud or trickery (NO)  WON
Valmonte had testamentary capacity at the time he executed the will (YES) SC Ruling: Article 839 of the
Civil Code states the instances when a will may be disallowed, as follows: (1) If the formalities required
by law have not been complied with; (2) If the testator was insane, or otherwise mentally incapable of
making a will, at the time of its execution; (3) If it was executed through force or under duress, or the
influence of fear, or threats; (4) If it was procured by undue and improper pressure and influence, on the
part of the beneficiary or of some other person; (5) If the signature of the testator was procured by
fraud; (6) If the testator acted by mistake or did not intend that the instrument he signed should be his
will at the time of affixing his signature thereto. Existence of Fraud in the Execution of a Will Petitioner
contends that it was "highly dubious for a woman at the prime of her young life [to] almost immediately
plunge into marriage with a man who [was] thrice her age x x x and who happened to be [a] Fil-
American pensionado," thus casting doubt on the intention of respondent in seeking the probate of the
will. Moreover, it supposedly "defies human reason, logic and common experience" for an old man with
a severe psychological condition to have willingly signed a last will and testament. We are not
convinced. Fraud "is a trick, secret device, false statement, or pretense, by which the subject of it is
cheated. It may be of such character that the testator is misled or deceived as to the nature or contents
of the document which he executes, or it may relate to some extrinsic fact, in consequence of the
deception regarding which the testator is led to make a certain will which, but for the fraud, he would
not have made." Unfortunately, in this case, other than the self-serving allegations of petitioner, no
evidence of fraud was ever presented. It is a settled doctrine that the omission of some relatives does
not affect the due execution of a will. Capacity to make a will According to Article 799, the three things
that the testator must have the ability to know to be considered of sound mind are as follows: (1) the
nature of the estate to be disposed of, (2) the proper objects of the testator's bounty, and (3) the
character of the testamentary act. Applying this test to the present case, we find that the appellate
court was correct in holding that Placido had testamentary capacity at the time of the execution of his
will. It must be noted that despite his advanced age, he was still able to identify accurately the kinds of
property he owned, the extent of his shares in them and even their locations. As regards the proper
objects of his bounty, it was sufficient that he identified his wife as sole beneficiary. As we have stated
earlier, the omission of some relatives from the will did not affect its formal validity. There being no
showing of fraud in its execution, intent in its disposition becomes irrelevant.

LETICIA VALMONTE ORTEGA v. JOSEFINA C. VALMONTE, GR NO. 157451,


2005-12-16

Facts:
This is a case of the probate of the last will and testament of Placido
Valmonte. Placido lived and worked in the United States and retired in the
US. So after the retirement Placido went back to the Philippines. When he
age of 80 he wed Josefina who was then 28 years old, in a ceremony
solemnized by Judge

Perfecto Laguio, Jr. on February 5, 1982. But in a little more than two
years of wedded bliss, Placido died on October 8, 1984 of a cause written
down as COR PULMONALE.

"Placido executed a notarial last will and testament written in English


and consisting of two (2) pages

"The allowance to probate of this will was opposed by Leticia on the


grounds that:

Petitioner failed to allege all assets of the testator, especially those


found in the USA;

Petitioner failed to state the names, ages, and residences of the heirs of
the testator; or to give them proper notice pursuant to law;

Will was not executed and attested as required by law and legal
solemnities and formalities were not complied with;

Testator was mentally incapable to make a will at the time of the alleged
execution he being in an advance sate of senility;

Will was executed under duress, or the influence of fear or threats;

Will was procured by undue and improper influence and pressure on the
part of the petitioner and/or her agents and/or assistants; and/or

Signature of testator was procured by fraud, or trick, and he did not


intend that the instrument should be his will at the time of affixing his
signature thereto;"

Josefina said she had no... knowledge of the existence of the last will and
testament of her husband, but just serendipitously found it in his attache
case after his death. It was only then that she learned that the testator
bequeathed to her his properties and she was named the executrix in the
said will.

"Notary Public Floro Sarmiento, the notary public who notarized the
testator's will, testified that it was in the first week of June 1983 when
the testator together with the three witnesses of the will went to his
house cum law office and requested him to prepare his last will... and
testament. After the testator instructed him on the terms and dispositions
he wanted on the will, the notary public told them to come back

He likewise explained that though it appears that the will was signed by
the testator and his witnesses on June 15, 1983, the day when it should
have been executed... had he not gone out of town, the formal execution
was actually on August 9, 1983. He reasoned that he no longer changed
the typewritten date of June 15, 1983 because he did not like the
document to appear dirty. The notary public also testified that to his
observation the... testator was physically and mentally capable at the
time he affixed his signature on the will.

"The attesting witnesses to the will corroborated the testimony of the


notary public,... "It then found these grounds extant and proven, and
accordingly disallowed probate."[... appellate court admitted the will of
Placido Valmonte to probate. The CA upheld the credibility of the notary
public and the subscribing witnesses who had acknowledged the due
execution of the will. Moreover, it held that the testator had...
testamentary capacity at the time of the execution of the will. It added
that his "sexual exhibitionism and unhygienic, crude and impolite
ways"[6] did not make him a person of unsound mind.

Issues:

Whether or not the signature of Placido Valmonte in the subject will was
procured by fraud or trickery, and that Placido Valmonte never intended
that the instrument should be his last will and testament.

In short, petitioner assails the CA's allowance of the probate of the will of
Placido Valmonte.

Ruling:

The Petition has no merit.

We stress that the party challenging the will bears the burden of proving
the existence of fraud at the time of its execution.[14] The burden to
show otherwise shifts to the proponent of the will only upon a showing of
credible evidence of fraud.[15] Unfortunately in this case, other than the
self-serving allegations of petitioner, no evidence of fraud was ever
presented.

It is a settled doctrine that the omission of some relatives does not affect
the due execution of a will.[16] That the testator was tricked into signing
it was not sufficiently established by the fact that he had instituted his
wife, who was more than fifty... years his junior, as the sole beneficiary;
and disregarded petitioner and her family, who were the ones who had
taken "the cudgels of taking care of [the testator] in his twilight years."[...
petitioner failed to substantiate her claim of a "grand conspiracy" in the
commission of a fraud. There was no showing that the witnesses of the
proponent stood to receive any benefit from the allowance of the will. The
testimonies of the three subscribing witnesses and... the notary are
credible evidence of its due execution.

the Petition is DENIED, and the assailed Decision and Resolution of the
Court of Appeals are AFFIRMED. Costs against petitioner.

Principles:

Fraud "is a trick, secret device, false statement, or pretense, by which


the subject of it is cheated. It may be of such character that the testator
is misled or deceived as to the nature or contents of the document which
he executes, or it may relate to... some extrinsic fact, in consequence of
the deception regarding which the testator is led to make a certain will
which, but for the fraud, he would not have made."[... the conflict
between the dates appearing on the will does not invalidate the
document, "because the law does not even require that a [notarial] will x
x x be executed and acknowledged on the same occasion."

(Allowance/Disallowance of Wills) Case Citation: Ortega v. Valmonte | G.R. No. 157451 Date: Dec 16,
2005 Petitioners: Leticia Valmonte Ortega Respondents: Josefina C. Valmonte Doctrine: According to
Article 799, the three things that the testator must have the ability to know to be considered of sound
mind are as follows: (1) the nature of the estate to be disposed of, (2) the proper objects of the
testator's bounty, and (3) the character of the testamentary act. Antecedent Facts: Placido toiled and
lived for a long time in the United States until he finally reached retirement. In 1980, Placido finally came
home to stay in the Philippines. Two years after his arrival from the United States and at the age of 80
he wed Josefina who was then 28 years old, on February 5, 1982. But in a little more than two years of
wedded bliss, Placido died on October 8, 1984, of a cause written down as COR PULMONALE. "Placido
executed a notarial last will and testament written in English and consisting of two (2) pages, and dated
June 15, 1983 but acknowledged only on August 9, 1983. The allowance to probate of the will was
opposed by Leticia. Respondent’s contention: The oppositor Leticia declared that Josefina should not
inherit alone because aside from her there are other children from the siblings of Placido who are just as
entitled to inherit from him. She attacked the mental capacity of the testator, declaring that at the time
of the execution of the notarial will the testator was already 83 years old and was no longer of sound
mind. She knew whereof she spoke because in 1983 Placido lived in the Makati residence and asked
Leticia's family to live with him and they took care of him. During that time, the testator's physical and
mental condition showed deterioration, aberrations and senility. This was corroborated by her daughter
Mary Jane Ortega for whom Placido took a fancy and wanted to marry. RTC Ruling: The lower court
disallowed the will. CA Ruling: Reversing the trial court, the appellate court admitted the will of Placido
Valmonte to probate. The CA upheld the credibility of the notary public and the subscribing witnesses
who had acknowledged the due execution of the will. Moreover, it held that the testator had
testamentary capacity at the time of the execution of the will. It added that his "sexual exhibitionism
and unhygienic, crude and impolite ways" did not make him a person of unsound mind. Issues:  WON
the signature of Placido Valmonte in the subject will was procured by fraud or trickery (NO)  WON
Valmonte had testamentary capacity at the time he executed the will (YES) SC Ruling: Article 839 of the
Civil Code states the instances when a will may be disallowed, as follows: (1) If the formalities required
by law have not been complied with; (2) If the testator was insane, or otherwise mentally incapable of
making a will, at the time of its execution; (3) If it was executed through force or under duress, or the
influence of fear, or threats; (4) If it was procured by undue and improper pressure and influence, on the
part of the beneficiary or of some other person; (5) If the signature of the testator was procured by
fraud; (6) If the testator acted by mistake or did not intend that the instrument he signed should be his
will at the time of affixing his signature thereto. Existence of Fraud in the Execution of a Will Petitioner
contends that it was "highly dubious for a woman at the prime of her young life [to] almost immediately
plunge into marriage with a man who [was] thrice her age x x x and who happened to be [a] Fil-
American pensionado," thus casting doubt on the intention of respondent in seeking the probate of the
will. Moreover, it supposedly "defies human reason, logic and common experience" for an old man with
a severe psychological condition to have willingly signed a last will and testament. We are not
convinced. Fraud "is a trick, secret device, false statement, or pretense, by which the subject of it is
cheated. It may be of such character that the testator is misled or deceived as to the nature or contents
of the document which he executes, or it may relate to some extrinsic fact, in consequence of the
deception regarding which the testator is led to make a certain will which, but for the fraud, he would
not have made." Unfortunately, in this case, other than the self-serving allegations of petitioner, no
evidence of fraud was ever presented. It is a settled doctrine that the omission of some relatives does
not affect the due execution of a will. Capacity to make a will According to Article 799, the three things
that the testator must have the ability to know to be considered of sound mind are as follows: (1) the
nature of the estate to be disposed of, (2) the proper objects of the testator's bounty, and (3) the
character of the testamentary act. Applying this test to the present case, we find that the appellate
court was correct in holding that Placido had testamentary capacity at the time of the execution of his
will. It must be noted that despite his advanced age, he was still able to identify accurately the kinds of
property he owned, the extent of his shares in them and even their locations. As regards the proper
objects of his bounty, it was sufficient that he identified his wife as sole beneficiary. As we have stated
earlier, the omission of some relatives from the will did not affect its formal validity. There being no
showing of fraud in its execution, intent in its disposition becomes irrelevant.

G.R. No. L-35993 December 19, 1932

In re Estate of the deceased Gregorio Tolentino. ADELAIDA TOLENTINO, petitioner-appellee,


vs.
NATALIA FRANCISCO, ET AL., oppositors-appellants.

Laurel, Del Rosario & Lualhati for appellants.


Eduardo Gutierrez Repide for appellee.
STREET, J.:

This petition was filed in the Court of First Instance of Manila by Adelaida Tolentino de Concepcion,
for the purpose of procuring probate of the will of Gregorio Tolentino, deceased, who died at the
hand of an assassin, in his home, No. 2541 Lico Street, in the District of Santa Cruz, Manila, on
November 9, 1930. In the inception of the proceedings Eugene de Mitkiewicz was appointed special
coadministrator, and he joined as coplaintiff in the petition. Opposition was made to the probate of
the will by Ciriaco Francisco, Natalia Francisco, and Gervasia Francisco, all cousins of the deceased
and residents of the City of Manila. Upon hearing the cause the trial court overruled the opposition,
declared the will to have been properly executed, and allowed the probate thereof. From this order
the three opponents appealed.

At the time of his death on November 9, 1930, Gregorio Tolentino was sixty-six years of age. During
the more vigorous years of his life he had been married to Benita Francisco, but she predeceased
him years ago. By their industry and frugality the two had accumulated a very considerable estate
which does not appear to have suffered any material diminution in the years of Tolentino's
widowhood. The pair had no children, and the generous instincts of the survivor prompted him to
gather around him in his comfortable and commodious home a number of his wife's kin; and by him
various younger members of the connection were supported and educated. At one time Tolentino
contemplated leaving his property mainly to these kin of his wife, of the surname Francisco; and for
several years prior to his death, he had kept a will indicating this desire. However, in October, 1930,
strained relations, resulting from grave disagreements, developed between Tolentino and the
Francisco relations and he determined to make a new will in which, apart from certain legacies in
favor of a few individuals, the bulk of his estate, worth probably about P150,000, should be given to
Adelaida Tolentino de Concepcion, as his universal heir.

To this end, on October 17, 1930, Tolentino went to the office of Eduardo Gutierrez Repide, an
attorney at 97 General Luna, Manila, and informed him that he wanted to make a new will and
desired Repide to draft it for him. After the necessary preliminary inquiries had been made, the
attorney suggested to Tolentino to return later, bringing a copy of the will previously made.
Accordingly, on the second day thereafter, Tolentino again appeared in Repide's office with the prior
will; and the attorney proceeded to reduce the new will to proper form. As the instrument was taking
shape Tolentino stated that he wanted the will to be signed in Repide's office, with Repide himself as
one of the attesting witnesses. For the other two witnesses Tolentino requested that two attorneys
attached to the office, namely, Leoncio B. Monzon and Ramon L. Sunico, should serve. For this
reason, in the draft of the will, as it at first stood, the names of the three above mentioned were
inserted as the names of the three attesting witnesses.

When the instrument had been reduced to proper form it was placed in the hands of Tolentino, the
testator, in order that he might take it home to reflect over its provisions and consider whether it
conformed in all respects to his wishes. On the morning of October 21 he again appeared in
Repide's office and returned to him the draft of the will with certain corrections. Among the changes
thus made was the suppression of the names of Monzon, Sunico, and Repide as attesting
witnesses, these names being substituted by the names of Jose Syyap, Agustin Vergel de Dios, and
Vicente Legarda. The explanation given by the testator for desiring this change was that he had met
Jose Syyap on the Escolta, the day before, and had committed the indiscretion of communicating
the fact that he (Tolentino) was having a new will made in which Monzon, Sunico, and Repide would
appear as the attesting witnesses. Now Syyap had been the draftsman of the former will of
Tolentino, and in this same will the name of Syyap appeared as one of the attesting witnesses, the
other two being Vicente Legarda and Vergel de Dios. When, therefore, Syyap learned that a new will
was being drawn up without his intervention, he showed profound disappointment, saying to
Tolentino that he considered it a gross offense that he, Legarda, and Vergel de Dios should be
eliminated as witnesses to the new will. Upon this manifestation of feeling by Syyap, Tolentino
decided to avail himself of Syyap, Legarda, and Vergel de Dios as witnesses to this will also, and he
therefore requested Repide to change the names of the attesting witnesses. After this point had
been settled Tolentino stated that he would request Syyap, Legarda, and Vergel de Dios to appear
at the office of Repide for the purpose of signing the will. To this end Tolentino went away but
returned later saying that he had spoken to Syyap about it and that the latter strenuously objected,
observing that the will should be signed at a chop-suey restaurant ( panciteria). Tolentino further
stated to his attorney in this conversation that he had arranged with Syyap and the other two
intending witnesses to meet at five o'clock in the afternoon of the next day, which was October 22,
for the purpose of executing the will.

Pursuant to these instructions Repide made the desired changes in the will; and just before twelve
o'clock noon of the next day Tolentino returned to Repide's office and received from him the criminal
document with a carbon copy thereof. Repide advised the testator that the copy should be executed
with the same formality as the original in order that the intention of the testator should not be
frustrated by the possible loss or destruction of the original.
lawphil.net

It is a custom in the office of Repide not to number the consecutive pages of a will, on the typewriting
machine, the duty of numbering the pages being left to the testator himself. This precaution appears
to have been born of experience, and has been adopted by Repide to prevent the possible
destruction of a will by the mere erasure of the figures or letters indicating the pagination, — a
disaster which, in Repide's experience, had occurred in at least one case. Accordingly, upon
delivering the completed will and carbon copy to the testator, Repide took particular pains to instruct
the testator to write the consecutive paging of both original and duplicate before signing the
instrument.

At his interview the testator suggested to Repide that the latter should also go to the place where the
will was to be executed, so that he might be present at the formality. The attorney replied that it was
impossible for him to do so as he had another engagement for the hour indicated, which would
prevent his attendance.

At about 4:30 p. m. on the same day, which was October 22, Tolentino started in his car to pick up
Syyap and Vergel de Dios at their respective homes on Antipolo and Benavides streets. He then
caused his chauffeur to drive with the three to La Previsora Filipina, on Rizal Avenue, where Vicente
Legarda, the third intending witness was to be found. Arriving at this place, the three entered the
office of Legarda, who was manager of the establishment, and they were invited to take seats, which
they did. Tolentino then suggested that the three should go as his guests to a panciteria, where they
could take refreshments and the will could be executed. Legarda replied that he must decline the
invitation for he had an engagement to go to the Cosmos Club the same afternoon. Upon this
Tolentino asked Legarda to permit the will to be signed in his office, and to this request Legarda
acceded.

Tolentino thereupon drew two documents from his pocket saving that it was his last will and
testament, done in duplicate, and he proceeded to read the original to the witnesses. After this had
been completed, Legarda himself took the will in hand and read it himself. He then returned it to
Tolentino, who thereupon proceeded, with pen and ink, to number the pages of the will thus, "Pagina
Primera", "Pagina Segunda", etc. He then paged the duplicate copy of the will in the same way. He
next proceeded to sign the original will and each of its pages by writing his name "G. Tolentino" in
the proper places. Following this, each of the three witnesses signed their own respective names at
the end of the will, at the end of the attesting clause, and in the left margin of each page of the
instrument. During this ceremony all of the persons concerned in the act of attestation were present
together, and all fully advertent to the solemnity that engaged their attention.

After the original of the will had been executed in the manner just stated, the testator expressed his
desire that the duplicate should be executed in the same manner. To this Syyap objected, on the
ground that it was unnecessary; and in this view he was supported by Vergel de Dios, with the result
that the wishes of the testator on this point could not be carried out. As the party was about to break
up Tolentino used these words: "For God's sake, as a favor, I request you not to let any one know
the contents of this will." The meeting then broke up and Tolentino returned Syyap and Vergel de
Dios to their homes in his car. He then proceeded to the law office of Repide, arriving about 6:15 p.
m. After preliminary explanations had been made, Tolentino requested Repide to keep the will
overnight in his safe, as it was already too late to place it in the compartment which Tolentino was
then renting in the Oriental Safe Deposit, in the Kneedler Building. In this connection the testator
stated that he did not wish to take the will to his home, as he knew that his relatives were watching
him and would take advantage of any carelessness on his part to pry into his papers. Also, in this
conversation Tolentino informed Repide of the refusal of Syyap to execute the duplicate of the will.

After a good part of an hour had thus been spent at Repide's office by the testator and after the
original of the will had been deposited in Repide's safe, Tolentino took the attorney to the latter's
residence in Ermita, and then returned to his own home, where he remained without again going out
that night. But promptly at nine o'clock the next morning Tolentino presented himself at Repide's
office for the purpose of securing the will. Repide happened to be out and Tolentino went away, but
again returned the next day and received the will. With the instrument thus in his possession he
proceeded at once to the Oriental Safe Deposit and there left the instrument in his private
compartment, No. 333, in which place it remained until withdrawn some two weeks later by order of
the court.

On the morning of November 9, 1930, Gregorio Tolentino was found dead in his bed, having
perished by the hands of an assassin.

The peculiarity of this case is that, upon the trial of this proceeding for the probate of the will of the
decedent, two of the attesting witnesses, Jose Syyap and Vergel de Dios, repudiated their
participation in the execution of the will at the time and place stated; and while admitting the
genuineness of their signatures to the will, pretended that they had severally signed the instrument,
at the request of the testator, at different places. Thus Syyap, testifying as a witness, claimed that
the testator brought the will to Syyap's house on the afternoon of October 21 — a time, be it
remembered, when the will had not yet left the hands of the draftsman — and upon learning that
Syyap could not be present at the time and place then being arranged for the execution of the will,
he requested Syyap, as a mere matter of complaisance, to sign the will then, which Syyap did.
Vergel de Dios has another story to tell of isolated action, claiming that he signed the will in the
evening of October 22 at the Hospital of San Juan de Dios in Intramuros.

We are unable to give any credence to the testimony of these two witnesses on this point, the same
being an evident fabrication designed for the purpose of defeating the will. In the first place, the
affirmative proof showing that the will was properly executed is adequate, consistent, and
convincing, consisting of the testimony of the third attesting witness, Vicente Legarda, corroborated
by Miguel Legarda and Urbana Rivera, two disinterested individuals, employees of La Previsora
Filipina, who were present in Legarda's office when the will was executed and who lent a discerning
attention to what was being done. In the second place, each of the seven signatures affixed to his
will by Syyap appear to the natural eye to have been made by using the same pen and ink that was
used by Legarda in signing the will. The same is also probably true of the seven signatures made by
Vergel de Dios. This could hardly have happened if the signatures of Syyap and Vergel de Dios had
been affixed, as they now pretend, at different times and places. In the third place, Both Syyap and
Vergel de Dios are impeached by proof of contradictory statements made by them on different
occasions prior to their appearance as witnesses in this case. In this connection we note that, after
the murder of Gregorio Tolentino, and while the police authorities were investigating his death,
Nemesio Alferez, a detective, sent for Syyap and questioned him concerning his relations with the
deceased. Upon this occasion Syyap stated that Gregorio Tolentino had lately made a will, that it
had been executed at the office of La Previsora Filipina under the circumstances already stated, and
that he himself had served as one of the attesting witnesses.

With respect to Vergel de Dios we have the following fact: On the day that Gregorio Tolentino was
buried, Ramon Llorente, a member of the city police force, was sent out to the cemetery in order that
he might be present and observe the demeanor on that occasion of such Tolentino's kin as might be
present. Llorente arrived before the funeral cortege, having been taken out to the cemetery by
Repide. While the two were waiting at the cemetery, Llorente noted the presence of Vergel de Dios,
he requested the policeman to introduce him. In the conversation that ensued Vergel de Dios stated
with considerable detail that Gregorio Tolentino had made a will just before his death, that it was
executed at La Previsora Filipina, and that he was one of the witnesses who attested the instrument
at that time and place.

Again, on a certain occasion subsequent to the death of Gregorio Tolentino, Juan Concepcion the
husband of Adelaida Tolentino, accompanied by Genoveva de Mendoza, called upon Vergel de
Dios, and in the conversation that resulted Vergel de Dios told them that the will was properly
executed, that he was one of the attesting witnesses, and that it had been signed by all of them in
the office of La Previsora Filipina.

These circumstances and other incidents revealed in the proof leave no room for doubt in our mind
that Syyap and Vergel de Dios have entered into a conspiracy between themselves, and in concert
with the opponents, to defeat the will of Gregorio Tolentino although they are well aware that said
will was in all respects properly executed; and the trial court, in our opinion, committed no error in
admitting the will to probate.

When a will is contested it is the duty of the proponent to call all of the attesting witnesses, if
available but the validity of the will in no wise depends upon the united support of the will by all of
those witnesses. A will may be admitted to probate notwithstanding the fact that one or more of the
subscribing witnesses do not unite with the other, or others, in proving all the facts upon which the
validity of the will rests. (Fernandez vs. Tantoco, 49 Phil., 380.) It is sufficient if the court is satisfied
from all the proof that the will was executed and attested in the manner required by law. In this case
we feel well assured that the contested will was properly executed and the order admitting to it
probate was entirely proper.

The order appealed from will therefore be affirmed, with costs against the appellants. So ordered.
TOLENTINO VS. FRANCISCO

Gregorio Tolentino, a deceased had a will and a probate was filed by Adelaida Tolentino, the petitioner.

You might also like